Download DUET Master 2018 DU MSc Anthropology Question Paper With Answer Key

Download DUET (Delhi University Entrance Test conducted by the NTA) 2018 DU MSc Anthropology Question Paper With Solution Key

1)
2)
3)
4)
5)
DU MSc Anthropology
Topic:- DU_J18_MSC_ANTHRO
Increased labour investment in agriculture with no increase in per capita productivity is called as
[Question ID = 18814]
1. Agricultural recession [Option ID = 45250]
2. Agricultural stagnation [Option ID = 45247]
3. Agricultural relegation [Option ID = 45248]
4. Agricultural involution [Option ID = 45249]
Correct Answer :-
Agricultural involution [Option ID = 45249]
Which of the following is a hominid?
[Question ID = 18857]
1. Chimpanzee [Option ID = 45421]
2. Human [Option ID = 45422]
3. Macaque [Option ID = 45419]
4. Baboon [Option ID = 45420]
Correct Answer :-
Human [Option ID = 45422]
Which rule says that the best suited human body for a hot, tropical region would be one with long limbs and short trunk?
[Question ID = 18827]
1. Allen?s rule [Option ID = 45299]
2. Thompson?s rule [Option ID = 45302]
3. Gloger?s rule [Option ID = 45301]
4. Bergman?s rule [Option ID = 45300]
Correct Answer :-
Allen?s rule [Option ID = 45299]
Which Australopithecus fossil was nicknamed as Lucy?
[Question ID = 18867]
1. Australopithecus africanus [Option ID = 45459]
2. Australopithecus afarensis [Option ID = 45462]
3. Australopithecus anamensis [Option ID = 45460]
4. Australopithecus robustus [Option ID = 45461]
Correct Answer :-
Australopithecus afarensis [Option ID = 45462]
Which among the following does not constitute the elementary form of exchange?
[Question ID = 18820]
1. Exchange of goods [Option ID = 45274]
2. Exchange of women [Option ID = 45271]
3. Exchange of money [Option ID = 45272]
4. Exchange of messages [Option ID = 45273]
Correct Answer :-
Exchange of goods [Option ID = 45274]
FirstRanker.com - FirstRanker's Choice
1)
2)
3)
4)
5)
DU MSc Anthropology
Topic:- DU_J18_MSC_ANTHRO
Increased labour investment in agriculture with no increase in per capita productivity is called as
[Question ID = 18814]
1. Agricultural recession [Option ID = 45250]
2. Agricultural stagnation [Option ID = 45247]
3. Agricultural relegation [Option ID = 45248]
4. Agricultural involution [Option ID = 45249]
Correct Answer :-
Agricultural involution [Option ID = 45249]
Which of the following is a hominid?
[Question ID = 18857]
1. Chimpanzee [Option ID = 45421]
2. Human [Option ID = 45422]
3. Macaque [Option ID = 45419]
4. Baboon [Option ID = 45420]
Correct Answer :-
Human [Option ID = 45422]
Which rule says that the best suited human body for a hot, tropical region would be one with long limbs and short trunk?
[Question ID = 18827]
1. Allen?s rule [Option ID = 45299]
2. Thompson?s rule [Option ID = 45302]
3. Gloger?s rule [Option ID = 45301]
4. Bergman?s rule [Option ID = 45300]
Correct Answer :-
Allen?s rule [Option ID = 45299]
Which Australopithecus fossil was nicknamed as Lucy?
[Question ID = 18867]
1. Australopithecus africanus [Option ID = 45459]
2. Australopithecus afarensis [Option ID = 45462]
3. Australopithecus anamensis [Option ID = 45460]
4. Australopithecus robustus [Option ID = 45461]
Correct Answer :-
Australopithecus afarensis [Option ID = 45462]
Which among the following does not constitute the elementary form of exchange?
[Question ID = 18820]
1. Exchange of goods [Option ID = 45274]
2. Exchange of women [Option ID = 45271]
3. Exchange of money [Option ID = 45272]
4. Exchange of messages [Option ID = 45273]
Correct Answer :-
Exchange of goods [Option ID = 45274]
6)
7)
8)
9)
10)
11)
12)
Which prehistoric site is known as Mode I Industry?
[Question ID = 18835]
1. Pirro Nord [Option ID = 45333]
2. Hunsgi [Option ID = 45331]
3. Tautavel [Option ID = 45332]
4. Somme valley [Option ID = 45334]
Correct Answer :-
Pirro Nord [Option ID = 45333]
Any quantitative increase in body size is known as [Question ID = 18788]
1. Development [Option ID = 45144]
2. Maturation [Option ID = 45146]
3. Differentiation [Option ID = 45145]
4. Growth [Option ID = 45143]
Correct Answer :-
Growth [Option ID = 45143]
The highest point on the head is known as [Question ID = 18807]
1. Opisthocranion [Option ID = 45222]
2. Bregma [Option ID = 45220]
3. Vertex [Option ID = 45221]
4. Lambda [Option ID = 45219]
Correct Answer :-
Vertex [Option ID = 45221]
Autosomes are
[Question ID = 18826]
1. Normal sex chromosomes [Option ID = 45296]
2. Abnormal sex chromosomes [Option ID = 45298]
3. Abnormal chromosomes [Option ID = 45297]
4. All chromosomes other than the sex chromosomes [Option ID = 45295]
Correct Answer :-
All chromosomes other than the sex chromosomes [Option ID = 45295]
?Puberty spurt? occurs in which one of the following stages of human growth [Question ID = 18798]
1. Infancy [Option ID = 45183]
2. Childhood [Option ID = 45185]
3. Adolescence [Option ID = 45186]
4. Adulthood [Option ID = 45184]
Correct Answer :-
Adolescence [Option ID = 45186]
Potlatch is associated with [Question ID = 18802]
1. Iroquois [Option ID = 45201]
2. Kwakiutl [Option ID = 45199]
3. Shoshonean [Option ID = 45202]
4. Arapesh [Option ID = 45200]
Correct Answer :-
Kwakiutl [Option ID = 45199]
Oldowan culture is dominated by
[Question ID = 18824]
1. Blade tools [Option ID = 45288]
2. Flake tools [Option ID = 45289]
FirstRanker.com - FirstRanker's Choice
1)
2)
3)
4)
5)
DU MSc Anthropology
Topic:- DU_J18_MSC_ANTHRO
Increased labour investment in agriculture with no increase in per capita productivity is called as
[Question ID = 18814]
1. Agricultural recession [Option ID = 45250]
2. Agricultural stagnation [Option ID = 45247]
3. Agricultural relegation [Option ID = 45248]
4. Agricultural involution [Option ID = 45249]
Correct Answer :-
Agricultural involution [Option ID = 45249]
Which of the following is a hominid?
[Question ID = 18857]
1. Chimpanzee [Option ID = 45421]
2. Human [Option ID = 45422]
3. Macaque [Option ID = 45419]
4. Baboon [Option ID = 45420]
Correct Answer :-
Human [Option ID = 45422]
Which rule says that the best suited human body for a hot, tropical region would be one with long limbs and short trunk?
[Question ID = 18827]
1. Allen?s rule [Option ID = 45299]
2. Thompson?s rule [Option ID = 45302]
3. Gloger?s rule [Option ID = 45301]
4. Bergman?s rule [Option ID = 45300]
Correct Answer :-
Allen?s rule [Option ID = 45299]
Which Australopithecus fossil was nicknamed as Lucy?
[Question ID = 18867]
1. Australopithecus africanus [Option ID = 45459]
2. Australopithecus afarensis [Option ID = 45462]
3. Australopithecus anamensis [Option ID = 45460]
4. Australopithecus robustus [Option ID = 45461]
Correct Answer :-
Australopithecus afarensis [Option ID = 45462]
Which among the following does not constitute the elementary form of exchange?
[Question ID = 18820]
1. Exchange of goods [Option ID = 45274]
2. Exchange of women [Option ID = 45271]
3. Exchange of money [Option ID = 45272]
4. Exchange of messages [Option ID = 45273]
Correct Answer :-
Exchange of goods [Option ID = 45274]
6)
7)
8)
9)
10)
11)
12)
Which prehistoric site is known as Mode I Industry?
[Question ID = 18835]
1. Pirro Nord [Option ID = 45333]
2. Hunsgi [Option ID = 45331]
3. Tautavel [Option ID = 45332]
4. Somme valley [Option ID = 45334]
Correct Answer :-
Pirro Nord [Option ID = 45333]
Any quantitative increase in body size is known as [Question ID = 18788]
1. Development [Option ID = 45144]
2. Maturation [Option ID = 45146]
3. Differentiation [Option ID = 45145]
4. Growth [Option ID = 45143]
Correct Answer :-
Growth [Option ID = 45143]
The highest point on the head is known as [Question ID = 18807]
1. Opisthocranion [Option ID = 45222]
2. Bregma [Option ID = 45220]
3. Vertex [Option ID = 45221]
4. Lambda [Option ID = 45219]
Correct Answer :-
Vertex [Option ID = 45221]
Autosomes are
[Question ID = 18826]
1. Normal sex chromosomes [Option ID = 45296]
2. Abnormal sex chromosomes [Option ID = 45298]
3. Abnormal chromosomes [Option ID = 45297]
4. All chromosomes other than the sex chromosomes [Option ID = 45295]
Correct Answer :-
All chromosomes other than the sex chromosomes [Option ID = 45295]
?Puberty spurt? occurs in which one of the following stages of human growth [Question ID = 18798]
1. Infancy [Option ID = 45183]
2. Childhood [Option ID = 45185]
3. Adolescence [Option ID = 45186]
4. Adulthood [Option ID = 45184]
Correct Answer :-
Adolescence [Option ID = 45186]
Potlatch is associated with [Question ID = 18802]
1. Iroquois [Option ID = 45201]
2. Kwakiutl [Option ID = 45199]
3. Shoshonean [Option ID = 45202]
4. Arapesh [Option ID = 45200]
Correct Answer :-
Kwakiutl [Option ID = 45199]
Oldowan culture is dominated by
[Question ID = 18824]
1. Blade tools [Option ID = 45288]
2. Flake tools [Option ID = 45289]
13)
14)
15)
16)
17)
18)
3. Pebble tools [Option ID = 45287]
4. Microlithic tools [Option ID = 45290]
Correct Answer :-
Pebble tools [Option ID = 45287]
A higher fortified part of a city is called
[Question ID = 18823]
1. metropolis [Option ID = 45284]
2. megapolis [Option ID = 45285]
3. acropolis [Option ID = 45283]
4. necropolis [Option ID = 45286]
Correct Answer :-
acropolis [Option ID = 45283]
Yale Cambridge expedition to the Sohan Valley was led by
[Question ID = 18795]
1. R. B. Foote and V.D. Krishnaswamy [Option ID = 45171]
2. G. R. Sharma and V.D. Mishra [Option ID = 45174]
3. H. D. Sankalia and B. Subba Rao [Option ID = 45173]
4. H. De. Terra and T. T. Paterson [Option ID = 45172]
Correct Answer :-
H. De. Terra and T. T. Paterson [Option ID = 45172]
Two nuclear families in adjacent generations with one son/husband or daughter/wife who is a member of both the families is called a
[Question ID = 18870]
1. lineal family [Option ID = 45471]
2. supplemented nuclear family [Option ID = 45474]
3. stem family [Option ID = 45473]
4. compound family [Option ID = 45472]
Correct Answer :-
stem family [Option ID = 45473]
The lack, excess or imbalance of nutrients in the diet leading to ill health is known as [Question ID = 18790]
1. Malnutrition [Option ID = 45153]
2. Over nutrition [Option ID = 45151]
3. Under nutrition [Option ID = 45152]
4. Marasmus [Option ID = 45154]
Correct Answer :-
Malnutrition [Option ID = 45153]
Holocene starts around
[Question ID = 18834]
1. 20,000 B.P. [Option ID = 45329]
2. 12,000 B.P. [Option ID = 45328]
3. 14,000 B.P. [Option ID = 45330]
4. 10,000 B.P. [Option ID = 45327]
Correct Answer :-
10,000 B.P. [Option ID = 45327]
According to Levi-Strauss which of the following constitute one of the ?atoms of kinship? relationship?
[Question ID = 18879]
FirstRanker.com - FirstRanker's Choice
1)
2)
3)
4)
5)
DU MSc Anthropology
Topic:- DU_J18_MSC_ANTHRO
Increased labour investment in agriculture with no increase in per capita productivity is called as
[Question ID = 18814]
1. Agricultural recession [Option ID = 45250]
2. Agricultural stagnation [Option ID = 45247]
3. Agricultural relegation [Option ID = 45248]
4. Agricultural involution [Option ID = 45249]
Correct Answer :-
Agricultural involution [Option ID = 45249]
Which of the following is a hominid?
[Question ID = 18857]
1. Chimpanzee [Option ID = 45421]
2. Human [Option ID = 45422]
3. Macaque [Option ID = 45419]
4. Baboon [Option ID = 45420]
Correct Answer :-
Human [Option ID = 45422]
Which rule says that the best suited human body for a hot, tropical region would be one with long limbs and short trunk?
[Question ID = 18827]
1. Allen?s rule [Option ID = 45299]
2. Thompson?s rule [Option ID = 45302]
3. Gloger?s rule [Option ID = 45301]
4. Bergman?s rule [Option ID = 45300]
Correct Answer :-
Allen?s rule [Option ID = 45299]
Which Australopithecus fossil was nicknamed as Lucy?
[Question ID = 18867]
1. Australopithecus africanus [Option ID = 45459]
2. Australopithecus afarensis [Option ID = 45462]
3. Australopithecus anamensis [Option ID = 45460]
4. Australopithecus robustus [Option ID = 45461]
Correct Answer :-
Australopithecus afarensis [Option ID = 45462]
Which among the following does not constitute the elementary form of exchange?
[Question ID = 18820]
1. Exchange of goods [Option ID = 45274]
2. Exchange of women [Option ID = 45271]
3. Exchange of money [Option ID = 45272]
4. Exchange of messages [Option ID = 45273]
Correct Answer :-
Exchange of goods [Option ID = 45274]
6)
7)
8)
9)
10)
11)
12)
Which prehistoric site is known as Mode I Industry?
[Question ID = 18835]
1. Pirro Nord [Option ID = 45333]
2. Hunsgi [Option ID = 45331]
3. Tautavel [Option ID = 45332]
4. Somme valley [Option ID = 45334]
Correct Answer :-
Pirro Nord [Option ID = 45333]
Any quantitative increase in body size is known as [Question ID = 18788]
1. Development [Option ID = 45144]
2. Maturation [Option ID = 45146]
3. Differentiation [Option ID = 45145]
4. Growth [Option ID = 45143]
Correct Answer :-
Growth [Option ID = 45143]
The highest point on the head is known as [Question ID = 18807]
1. Opisthocranion [Option ID = 45222]
2. Bregma [Option ID = 45220]
3. Vertex [Option ID = 45221]
4. Lambda [Option ID = 45219]
Correct Answer :-
Vertex [Option ID = 45221]
Autosomes are
[Question ID = 18826]
1. Normal sex chromosomes [Option ID = 45296]
2. Abnormal sex chromosomes [Option ID = 45298]
3. Abnormal chromosomes [Option ID = 45297]
4. All chromosomes other than the sex chromosomes [Option ID = 45295]
Correct Answer :-
All chromosomes other than the sex chromosomes [Option ID = 45295]
?Puberty spurt? occurs in which one of the following stages of human growth [Question ID = 18798]
1. Infancy [Option ID = 45183]
2. Childhood [Option ID = 45185]
3. Adolescence [Option ID = 45186]
4. Adulthood [Option ID = 45184]
Correct Answer :-
Adolescence [Option ID = 45186]
Potlatch is associated with [Question ID = 18802]
1. Iroquois [Option ID = 45201]
2. Kwakiutl [Option ID = 45199]
3. Shoshonean [Option ID = 45202]
4. Arapesh [Option ID = 45200]
Correct Answer :-
Kwakiutl [Option ID = 45199]
Oldowan culture is dominated by
[Question ID = 18824]
1. Blade tools [Option ID = 45288]
2. Flake tools [Option ID = 45289]
13)
14)
15)
16)
17)
18)
3. Pebble tools [Option ID = 45287]
4. Microlithic tools [Option ID = 45290]
Correct Answer :-
Pebble tools [Option ID = 45287]
A higher fortified part of a city is called
[Question ID = 18823]
1. metropolis [Option ID = 45284]
2. megapolis [Option ID = 45285]
3. acropolis [Option ID = 45283]
4. necropolis [Option ID = 45286]
Correct Answer :-
acropolis [Option ID = 45283]
Yale Cambridge expedition to the Sohan Valley was led by
[Question ID = 18795]
1. R. B. Foote and V.D. Krishnaswamy [Option ID = 45171]
2. G. R. Sharma and V.D. Mishra [Option ID = 45174]
3. H. D. Sankalia and B. Subba Rao [Option ID = 45173]
4. H. De. Terra and T. T. Paterson [Option ID = 45172]
Correct Answer :-
H. De. Terra and T. T. Paterson [Option ID = 45172]
Two nuclear families in adjacent generations with one son/husband or daughter/wife who is a member of both the families is called a
[Question ID = 18870]
1. lineal family [Option ID = 45471]
2. supplemented nuclear family [Option ID = 45474]
3. stem family [Option ID = 45473]
4. compound family [Option ID = 45472]
Correct Answer :-
stem family [Option ID = 45473]
The lack, excess or imbalance of nutrients in the diet leading to ill health is known as [Question ID = 18790]
1. Malnutrition [Option ID = 45153]
2. Over nutrition [Option ID = 45151]
3. Under nutrition [Option ID = 45152]
4. Marasmus [Option ID = 45154]
Correct Answer :-
Malnutrition [Option ID = 45153]
Holocene starts around
[Question ID = 18834]
1. 20,000 B.P. [Option ID = 45329]
2. 12,000 B.P. [Option ID = 45328]
3. 14,000 B.P. [Option ID = 45330]
4. 10,000 B.P. [Option ID = 45327]
Correct Answer :-
10,000 B.P. [Option ID = 45327]
According to Levi-Strauss which of the following constitute one of the ?atoms of kinship? relationship?
[Question ID = 18879]
19)
20)
21)
22)
23)
24)
1. mother-son [Option ID = 45507]
2. mother-daughter [Option ID = 45508]
3. father-son [Option ID = 45509]
4. father-daughter [Option ID = 45510]
Correct Answer :-
mother-son [Option ID = 45507]
A karyotype is a:
[Question ID = 18819]
1. Picture of an individual's chromosomes arranged in a standardized way [Option ID = 45269]
2. Type of abnormal chromosome that is associated with Down's syndrome [Option ID = 45268]
3. General term for any type of chromosome [Option ID = 45267]
4. General term for sex chromosome [Option ID = 45270]
Correct Answer :-
Picture of an individual's chromosomes arranged in a standardized way [Option ID = 45269]
Theodicy is an
[Question ID = 18882]
1. attempt to explain theocracy [Option ID = 45521]
2. attempt to explain good [Option ID = 45519]
3. attempt to explain evil [Option ID = 45520]
4. attempt to explain lie and deceit [Option ID = 45522]
Correct Answer :-
attempt to explain evil [Option ID = 45520]
The word ?Somatotype? was coined by [Question ID = 18797]
1. Faulkner [Option ID = 45182]
2. Sheldon [Option ID = 45180]
3. Parnell [Option ID = 45181]
4. Heath and Carter [Option ID = 45179]
Correct Answer :-
Heath and Carter [Option ID = 45179]
Substantivism as an approach in economic anthropology was given by
[Question ID = 18853]
1. A. L. Kroeber [Option ID = 45406]
2. Karl Polanyi [Option ID = 45405]
3. Oscar Lewis [Option ID = 45403]
4. Robert Redfield [Option ID = 62620]
Correct Answer :-
Karl Polanyi [Option ID = 45405]
?Genetics and the Origin of Species? was written by :
[Question ID = 18859]
1. Thomas Malthus. [Option ID = 45430]
2. Theodosius Dobzhansky [Option ID = 45428]
3. Charles Darwin [Option ID = 45427]
4. T. H. Morgan [Option ID = 45429]
Correct Answer :-
Theodosius Dobzhansky [Option ID = 45428]
The ability of the living organisms to survive in a particular ecological set up is called [Question ID = 18809]
FirstRanker.com - FirstRanker's Choice
1)
2)
3)
4)
5)
DU MSc Anthropology
Topic:- DU_J18_MSC_ANTHRO
Increased labour investment in agriculture with no increase in per capita productivity is called as
[Question ID = 18814]
1. Agricultural recession [Option ID = 45250]
2. Agricultural stagnation [Option ID = 45247]
3. Agricultural relegation [Option ID = 45248]
4. Agricultural involution [Option ID = 45249]
Correct Answer :-
Agricultural involution [Option ID = 45249]
Which of the following is a hominid?
[Question ID = 18857]
1. Chimpanzee [Option ID = 45421]
2. Human [Option ID = 45422]
3. Macaque [Option ID = 45419]
4. Baboon [Option ID = 45420]
Correct Answer :-
Human [Option ID = 45422]
Which rule says that the best suited human body for a hot, tropical region would be one with long limbs and short trunk?
[Question ID = 18827]
1. Allen?s rule [Option ID = 45299]
2. Thompson?s rule [Option ID = 45302]
3. Gloger?s rule [Option ID = 45301]
4. Bergman?s rule [Option ID = 45300]
Correct Answer :-
Allen?s rule [Option ID = 45299]
Which Australopithecus fossil was nicknamed as Lucy?
[Question ID = 18867]
1. Australopithecus africanus [Option ID = 45459]
2. Australopithecus afarensis [Option ID = 45462]
3. Australopithecus anamensis [Option ID = 45460]
4. Australopithecus robustus [Option ID = 45461]
Correct Answer :-
Australopithecus afarensis [Option ID = 45462]
Which among the following does not constitute the elementary form of exchange?
[Question ID = 18820]
1. Exchange of goods [Option ID = 45274]
2. Exchange of women [Option ID = 45271]
3. Exchange of money [Option ID = 45272]
4. Exchange of messages [Option ID = 45273]
Correct Answer :-
Exchange of goods [Option ID = 45274]
6)
7)
8)
9)
10)
11)
12)
Which prehistoric site is known as Mode I Industry?
[Question ID = 18835]
1. Pirro Nord [Option ID = 45333]
2. Hunsgi [Option ID = 45331]
3. Tautavel [Option ID = 45332]
4. Somme valley [Option ID = 45334]
Correct Answer :-
Pirro Nord [Option ID = 45333]
Any quantitative increase in body size is known as [Question ID = 18788]
1. Development [Option ID = 45144]
2. Maturation [Option ID = 45146]
3. Differentiation [Option ID = 45145]
4. Growth [Option ID = 45143]
Correct Answer :-
Growth [Option ID = 45143]
The highest point on the head is known as [Question ID = 18807]
1. Opisthocranion [Option ID = 45222]
2. Bregma [Option ID = 45220]
3. Vertex [Option ID = 45221]
4. Lambda [Option ID = 45219]
Correct Answer :-
Vertex [Option ID = 45221]
Autosomes are
[Question ID = 18826]
1. Normal sex chromosomes [Option ID = 45296]
2. Abnormal sex chromosomes [Option ID = 45298]
3. Abnormal chromosomes [Option ID = 45297]
4. All chromosomes other than the sex chromosomes [Option ID = 45295]
Correct Answer :-
All chromosomes other than the sex chromosomes [Option ID = 45295]
?Puberty spurt? occurs in which one of the following stages of human growth [Question ID = 18798]
1. Infancy [Option ID = 45183]
2. Childhood [Option ID = 45185]
3. Adolescence [Option ID = 45186]
4. Adulthood [Option ID = 45184]
Correct Answer :-
Adolescence [Option ID = 45186]
Potlatch is associated with [Question ID = 18802]
1. Iroquois [Option ID = 45201]
2. Kwakiutl [Option ID = 45199]
3. Shoshonean [Option ID = 45202]
4. Arapesh [Option ID = 45200]
Correct Answer :-
Kwakiutl [Option ID = 45199]
Oldowan culture is dominated by
[Question ID = 18824]
1. Blade tools [Option ID = 45288]
2. Flake tools [Option ID = 45289]
13)
14)
15)
16)
17)
18)
3. Pebble tools [Option ID = 45287]
4. Microlithic tools [Option ID = 45290]
Correct Answer :-
Pebble tools [Option ID = 45287]
A higher fortified part of a city is called
[Question ID = 18823]
1. metropolis [Option ID = 45284]
2. megapolis [Option ID = 45285]
3. acropolis [Option ID = 45283]
4. necropolis [Option ID = 45286]
Correct Answer :-
acropolis [Option ID = 45283]
Yale Cambridge expedition to the Sohan Valley was led by
[Question ID = 18795]
1. R. B. Foote and V.D. Krishnaswamy [Option ID = 45171]
2. G. R. Sharma and V.D. Mishra [Option ID = 45174]
3. H. D. Sankalia and B. Subba Rao [Option ID = 45173]
4. H. De. Terra and T. T. Paterson [Option ID = 45172]
Correct Answer :-
H. De. Terra and T. T. Paterson [Option ID = 45172]
Two nuclear families in adjacent generations with one son/husband or daughter/wife who is a member of both the families is called a
[Question ID = 18870]
1. lineal family [Option ID = 45471]
2. supplemented nuclear family [Option ID = 45474]
3. stem family [Option ID = 45473]
4. compound family [Option ID = 45472]
Correct Answer :-
stem family [Option ID = 45473]
The lack, excess or imbalance of nutrients in the diet leading to ill health is known as [Question ID = 18790]
1. Malnutrition [Option ID = 45153]
2. Over nutrition [Option ID = 45151]
3. Under nutrition [Option ID = 45152]
4. Marasmus [Option ID = 45154]
Correct Answer :-
Malnutrition [Option ID = 45153]
Holocene starts around
[Question ID = 18834]
1. 20,000 B.P. [Option ID = 45329]
2. 12,000 B.P. [Option ID = 45328]
3. 14,000 B.P. [Option ID = 45330]
4. 10,000 B.P. [Option ID = 45327]
Correct Answer :-
10,000 B.P. [Option ID = 45327]
According to Levi-Strauss which of the following constitute one of the ?atoms of kinship? relationship?
[Question ID = 18879]
19)
20)
21)
22)
23)
24)
1. mother-son [Option ID = 45507]
2. mother-daughter [Option ID = 45508]
3. father-son [Option ID = 45509]
4. father-daughter [Option ID = 45510]
Correct Answer :-
mother-son [Option ID = 45507]
A karyotype is a:
[Question ID = 18819]
1. Picture of an individual's chromosomes arranged in a standardized way [Option ID = 45269]
2. Type of abnormal chromosome that is associated with Down's syndrome [Option ID = 45268]
3. General term for any type of chromosome [Option ID = 45267]
4. General term for sex chromosome [Option ID = 45270]
Correct Answer :-
Picture of an individual's chromosomes arranged in a standardized way [Option ID = 45269]
Theodicy is an
[Question ID = 18882]
1. attempt to explain theocracy [Option ID = 45521]
2. attempt to explain good [Option ID = 45519]
3. attempt to explain evil [Option ID = 45520]
4. attempt to explain lie and deceit [Option ID = 45522]
Correct Answer :-
attempt to explain evil [Option ID = 45520]
The word ?Somatotype? was coined by [Question ID = 18797]
1. Faulkner [Option ID = 45182]
2. Sheldon [Option ID = 45180]
3. Parnell [Option ID = 45181]
4. Heath and Carter [Option ID = 45179]
Correct Answer :-
Heath and Carter [Option ID = 45179]
Substantivism as an approach in economic anthropology was given by
[Question ID = 18853]
1. A. L. Kroeber [Option ID = 45406]
2. Karl Polanyi [Option ID = 45405]
3. Oscar Lewis [Option ID = 45403]
4. Robert Redfield [Option ID = 62620]
Correct Answer :-
Karl Polanyi [Option ID = 45405]
?Genetics and the Origin of Species? was written by :
[Question ID = 18859]
1. Thomas Malthus. [Option ID = 45430]
2. Theodosius Dobzhansky [Option ID = 45428]
3. Charles Darwin [Option ID = 45427]
4. T. H. Morgan [Option ID = 45429]
Correct Answer :-
Theodosius Dobzhansky [Option ID = 45428]
The ability of the living organisms to survive in a particular ecological set up is called [Question ID = 18809]
25)
26)
27)
28)
29)
30)
1. Acclimation [Option ID = 45228]
2. Acclimatisation [Option ID = 45229]
3. Adaptation [Option ID = 45227]
4. Selection [Option ID = 45230]
Correct Answer :-
Adaptation [Option ID = 45227]
___________is the physiological ability to reproduce an offspring.
[Question ID = 18876]
1. Fecundity [Option ID = 45496]
2. Fertility [Option ID = 45495]
3. Virility [Option ID = 45498]
4. Natality [Option ID = 45497]
Correct Answer :-
Fecundity [Option ID = 45496]
Charles Darwin's ideas concerning the causes of evolution were probably formulated in his mind
[Question ID = 18817]
1. During the late 1880's [Option ID = 45262]
2. During his voyage on H.M.S. Beagle, especially after he reached the Gal?pagos Islands [Option ID = 45261]
3. Before he began his voyage of exploration around the world on H.M.S. Beagle [Option ID = 45260]
4. While he was still a student at Cambridge University [Option ID = 45259]
Correct Answer :-
During his voyage on H.M.S. Beagle, especially after he reached the Gal?pagos Islands [Option ID = 45261]
First Himalayan Glaciation is known as
[Question ID = 18884]
1. Terrace I [Option ID = 45530]
2. Pinjaur [Option ID = 45527]
3. Boulder conglomerate [Option ID = 45529]
4. Tatrot [Option ID = 45528]
Correct Answer :-
Tatrot [Option ID = 45528]
The author of the book Hindus of the Himalayas is [Question ID = 18812]
1. S.C. Dube [Option ID = 45241]
2. D.N. Majumdar [Option ID = 45242]
3. B. K. Roy-Burman [Option ID = 45240]
4. G.D. Berreman [Option ID = 45239]
Correct Answer :-
G.D. Berreman [Option ID = 45239]
Who proposed the ?Three Age System? in pre-history?
[Question ID = 18883]
1. C. J. Thomsen [Option ID = 45525]
2. B. M. Fagan [Option ID = 45523]
3. H.D.Sankalia [Option ID = 45524]
4. F. Borde [Option ID = 45526]
Correct Answer :-
C. J. Thomsen [Option ID = 45525]
Who among the following worked on the complicity between colonial agencies and Anthropologists?
FirstRanker.com - FirstRanker's Choice
1)
2)
3)
4)
5)
DU MSc Anthropology
Topic:- DU_J18_MSC_ANTHRO
Increased labour investment in agriculture with no increase in per capita productivity is called as
[Question ID = 18814]
1. Agricultural recession [Option ID = 45250]
2. Agricultural stagnation [Option ID = 45247]
3. Agricultural relegation [Option ID = 45248]
4. Agricultural involution [Option ID = 45249]
Correct Answer :-
Agricultural involution [Option ID = 45249]
Which of the following is a hominid?
[Question ID = 18857]
1. Chimpanzee [Option ID = 45421]
2. Human [Option ID = 45422]
3. Macaque [Option ID = 45419]
4. Baboon [Option ID = 45420]
Correct Answer :-
Human [Option ID = 45422]
Which rule says that the best suited human body for a hot, tropical region would be one with long limbs and short trunk?
[Question ID = 18827]
1. Allen?s rule [Option ID = 45299]
2. Thompson?s rule [Option ID = 45302]
3. Gloger?s rule [Option ID = 45301]
4. Bergman?s rule [Option ID = 45300]
Correct Answer :-
Allen?s rule [Option ID = 45299]
Which Australopithecus fossil was nicknamed as Lucy?
[Question ID = 18867]
1. Australopithecus africanus [Option ID = 45459]
2. Australopithecus afarensis [Option ID = 45462]
3. Australopithecus anamensis [Option ID = 45460]
4. Australopithecus robustus [Option ID = 45461]
Correct Answer :-
Australopithecus afarensis [Option ID = 45462]
Which among the following does not constitute the elementary form of exchange?
[Question ID = 18820]
1. Exchange of goods [Option ID = 45274]
2. Exchange of women [Option ID = 45271]
3. Exchange of money [Option ID = 45272]
4. Exchange of messages [Option ID = 45273]
Correct Answer :-
Exchange of goods [Option ID = 45274]
6)
7)
8)
9)
10)
11)
12)
Which prehistoric site is known as Mode I Industry?
[Question ID = 18835]
1. Pirro Nord [Option ID = 45333]
2. Hunsgi [Option ID = 45331]
3. Tautavel [Option ID = 45332]
4. Somme valley [Option ID = 45334]
Correct Answer :-
Pirro Nord [Option ID = 45333]
Any quantitative increase in body size is known as [Question ID = 18788]
1. Development [Option ID = 45144]
2. Maturation [Option ID = 45146]
3. Differentiation [Option ID = 45145]
4. Growth [Option ID = 45143]
Correct Answer :-
Growth [Option ID = 45143]
The highest point on the head is known as [Question ID = 18807]
1. Opisthocranion [Option ID = 45222]
2. Bregma [Option ID = 45220]
3. Vertex [Option ID = 45221]
4. Lambda [Option ID = 45219]
Correct Answer :-
Vertex [Option ID = 45221]
Autosomes are
[Question ID = 18826]
1. Normal sex chromosomes [Option ID = 45296]
2. Abnormal sex chromosomes [Option ID = 45298]
3. Abnormal chromosomes [Option ID = 45297]
4. All chromosomes other than the sex chromosomes [Option ID = 45295]
Correct Answer :-
All chromosomes other than the sex chromosomes [Option ID = 45295]
?Puberty spurt? occurs in which one of the following stages of human growth [Question ID = 18798]
1. Infancy [Option ID = 45183]
2. Childhood [Option ID = 45185]
3. Adolescence [Option ID = 45186]
4. Adulthood [Option ID = 45184]
Correct Answer :-
Adolescence [Option ID = 45186]
Potlatch is associated with [Question ID = 18802]
1. Iroquois [Option ID = 45201]
2. Kwakiutl [Option ID = 45199]
3. Shoshonean [Option ID = 45202]
4. Arapesh [Option ID = 45200]
Correct Answer :-
Kwakiutl [Option ID = 45199]
Oldowan culture is dominated by
[Question ID = 18824]
1. Blade tools [Option ID = 45288]
2. Flake tools [Option ID = 45289]
13)
14)
15)
16)
17)
18)
3. Pebble tools [Option ID = 45287]
4. Microlithic tools [Option ID = 45290]
Correct Answer :-
Pebble tools [Option ID = 45287]
A higher fortified part of a city is called
[Question ID = 18823]
1. metropolis [Option ID = 45284]
2. megapolis [Option ID = 45285]
3. acropolis [Option ID = 45283]
4. necropolis [Option ID = 45286]
Correct Answer :-
acropolis [Option ID = 45283]
Yale Cambridge expedition to the Sohan Valley was led by
[Question ID = 18795]
1. R. B. Foote and V.D. Krishnaswamy [Option ID = 45171]
2. G. R. Sharma and V.D. Mishra [Option ID = 45174]
3. H. D. Sankalia and B. Subba Rao [Option ID = 45173]
4. H. De. Terra and T. T. Paterson [Option ID = 45172]
Correct Answer :-
H. De. Terra and T. T. Paterson [Option ID = 45172]
Two nuclear families in adjacent generations with one son/husband or daughter/wife who is a member of both the families is called a
[Question ID = 18870]
1. lineal family [Option ID = 45471]
2. supplemented nuclear family [Option ID = 45474]
3. stem family [Option ID = 45473]
4. compound family [Option ID = 45472]
Correct Answer :-
stem family [Option ID = 45473]
The lack, excess or imbalance of nutrients in the diet leading to ill health is known as [Question ID = 18790]
1. Malnutrition [Option ID = 45153]
2. Over nutrition [Option ID = 45151]
3. Under nutrition [Option ID = 45152]
4. Marasmus [Option ID = 45154]
Correct Answer :-
Malnutrition [Option ID = 45153]
Holocene starts around
[Question ID = 18834]
1. 20,000 B.P. [Option ID = 45329]
2. 12,000 B.P. [Option ID = 45328]
3. 14,000 B.P. [Option ID = 45330]
4. 10,000 B.P. [Option ID = 45327]
Correct Answer :-
10,000 B.P. [Option ID = 45327]
According to Levi-Strauss which of the following constitute one of the ?atoms of kinship? relationship?
[Question ID = 18879]
19)
20)
21)
22)
23)
24)
1. mother-son [Option ID = 45507]
2. mother-daughter [Option ID = 45508]
3. father-son [Option ID = 45509]
4. father-daughter [Option ID = 45510]
Correct Answer :-
mother-son [Option ID = 45507]
A karyotype is a:
[Question ID = 18819]
1. Picture of an individual's chromosomes arranged in a standardized way [Option ID = 45269]
2. Type of abnormal chromosome that is associated with Down's syndrome [Option ID = 45268]
3. General term for any type of chromosome [Option ID = 45267]
4. General term for sex chromosome [Option ID = 45270]
Correct Answer :-
Picture of an individual's chromosomes arranged in a standardized way [Option ID = 45269]
Theodicy is an
[Question ID = 18882]
1. attempt to explain theocracy [Option ID = 45521]
2. attempt to explain good [Option ID = 45519]
3. attempt to explain evil [Option ID = 45520]
4. attempt to explain lie and deceit [Option ID = 45522]
Correct Answer :-
attempt to explain evil [Option ID = 45520]
The word ?Somatotype? was coined by [Question ID = 18797]
1. Faulkner [Option ID = 45182]
2. Sheldon [Option ID = 45180]
3. Parnell [Option ID = 45181]
4. Heath and Carter [Option ID = 45179]
Correct Answer :-
Heath and Carter [Option ID = 45179]
Substantivism as an approach in economic anthropology was given by
[Question ID = 18853]
1. A. L. Kroeber [Option ID = 45406]
2. Karl Polanyi [Option ID = 45405]
3. Oscar Lewis [Option ID = 45403]
4. Robert Redfield [Option ID = 62620]
Correct Answer :-
Karl Polanyi [Option ID = 45405]
?Genetics and the Origin of Species? was written by :
[Question ID = 18859]
1. Thomas Malthus. [Option ID = 45430]
2. Theodosius Dobzhansky [Option ID = 45428]
3. Charles Darwin [Option ID = 45427]
4. T. H. Morgan [Option ID = 45429]
Correct Answer :-
Theodosius Dobzhansky [Option ID = 45428]
The ability of the living organisms to survive in a particular ecological set up is called [Question ID = 18809]
25)
26)
27)
28)
29)
30)
1. Acclimation [Option ID = 45228]
2. Acclimatisation [Option ID = 45229]
3. Adaptation [Option ID = 45227]
4. Selection [Option ID = 45230]
Correct Answer :-
Adaptation [Option ID = 45227]
___________is the physiological ability to reproduce an offspring.
[Question ID = 18876]
1. Fecundity [Option ID = 45496]
2. Fertility [Option ID = 45495]
3. Virility [Option ID = 45498]
4. Natality [Option ID = 45497]
Correct Answer :-
Fecundity [Option ID = 45496]
Charles Darwin's ideas concerning the causes of evolution were probably formulated in his mind
[Question ID = 18817]
1. During the late 1880's [Option ID = 45262]
2. During his voyage on H.M.S. Beagle, especially after he reached the Gal?pagos Islands [Option ID = 45261]
3. Before he began his voyage of exploration around the world on H.M.S. Beagle [Option ID = 45260]
4. While he was still a student at Cambridge University [Option ID = 45259]
Correct Answer :-
During his voyage on H.M.S. Beagle, especially after he reached the Gal?pagos Islands [Option ID = 45261]
First Himalayan Glaciation is known as
[Question ID = 18884]
1. Terrace I [Option ID = 45530]
2. Pinjaur [Option ID = 45527]
3. Boulder conglomerate [Option ID = 45529]
4. Tatrot [Option ID = 45528]
Correct Answer :-
Tatrot [Option ID = 45528]
The author of the book Hindus of the Himalayas is [Question ID = 18812]
1. S.C. Dube [Option ID = 45241]
2. D.N. Majumdar [Option ID = 45242]
3. B. K. Roy-Burman [Option ID = 45240]
4. G.D. Berreman [Option ID = 45239]
Correct Answer :-
G.D. Berreman [Option ID = 45239]
Who proposed the ?Three Age System? in pre-history?
[Question ID = 18883]
1. C. J. Thomsen [Option ID = 45525]
2. B. M. Fagan [Option ID = 45523]
3. H.D.Sankalia [Option ID = 45524]
4. F. Borde [Option ID = 45526]
Correct Answer :-
C. J. Thomsen [Option ID = 45525]
Who among the following worked on the complicity between colonial agencies and Anthropologists?
31)
32)
33)
34)
35)
36)
[Question ID = 18801]
1. Ibn Khaldun [Option ID = 45196]
2. Akbar Ahmed [Option ID = 45197]
3. Ralf Linto [Option ID = 45198]
4. Talal Asad [Option ID = 45195]
Correct Answer :-
Talal Asad [Option ID = 45195]
Who is the author of the book ?The Old Stone Age??
[Question ID = 18854]
1. M.C. Burkitt [Option ID = 45407]
2. J. N. Pal [Option ID = 45408]
3. P.C. Pant [Option ID = 45409]
4. H. De Lumley [Option ID = 45410]
Correct Answer :-
M.C. Burkitt [Option ID = 45407]
Who included the Proto-Australoid racial element in the classification of Indian races?
[Question ID = 18866]
1. S.S. Sarkar [Option ID = 45456]
2. H.H. Risley [Option ID = 45455]
3. B.S. Guha [Option ID = 45458]
4. A.C. Haddon [Option ID = 45457]
Correct Answer :-
B.S. Guha [Option ID = 45458]
In which State is the site Isampur located?
[Question ID = 18815]
1. Karnataka [Option ID = 45253]
2. Uttar Pradesh [Option ID = 45254]
3. Madhya Pradesh [Option ID = 45252]
4. Bihar [Option ID = 45251]
Correct Answer :-
Karnataka [Option ID = 45253]
The view of the skull from above is [Question ID = 18800]
1. Norma Basalis [Option ID = 45193]
2. Norma Occipitalis [Option ID = 45194]
3. Norma Frontalis [Option ID = 45192]
4. Norma Verticalis [Option ID = 45191]
Correct Answer :-
Norma Verticalis [Option ID = 45191]
?Childhood? is peculiar to [Question ID = 18789]
1. Nonhuman primates [Option ID = 45149]
2. Human and nonhuman primates [Option ID = 45147]
3. Humans [Option ID = 45148]
4. Mammals [Option ID = 45150]
Correct Answer :-
Humans [Option ID = 45148]
Children subjected to starvation for short periods may recover completely on getting adequate diet. This is referred to as
FirstRanker.com - FirstRanker's Choice
1)
2)
3)
4)
5)
DU MSc Anthropology
Topic:- DU_J18_MSC_ANTHRO
Increased labour investment in agriculture with no increase in per capita productivity is called as
[Question ID = 18814]
1. Agricultural recession [Option ID = 45250]
2. Agricultural stagnation [Option ID = 45247]
3. Agricultural relegation [Option ID = 45248]
4. Agricultural involution [Option ID = 45249]
Correct Answer :-
Agricultural involution [Option ID = 45249]
Which of the following is a hominid?
[Question ID = 18857]
1. Chimpanzee [Option ID = 45421]
2. Human [Option ID = 45422]
3. Macaque [Option ID = 45419]
4. Baboon [Option ID = 45420]
Correct Answer :-
Human [Option ID = 45422]
Which rule says that the best suited human body for a hot, tropical region would be one with long limbs and short trunk?
[Question ID = 18827]
1. Allen?s rule [Option ID = 45299]
2. Thompson?s rule [Option ID = 45302]
3. Gloger?s rule [Option ID = 45301]
4. Bergman?s rule [Option ID = 45300]
Correct Answer :-
Allen?s rule [Option ID = 45299]
Which Australopithecus fossil was nicknamed as Lucy?
[Question ID = 18867]
1. Australopithecus africanus [Option ID = 45459]
2. Australopithecus afarensis [Option ID = 45462]
3. Australopithecus anamensis [Option ID = 45460]
4. Australopithecus robustus [Option ID = 45461]
Correct Answer :-
Australopithecus afarensis [Option ID = 45462]
Which among the following does not constitute the elementary form of exchange?
[Question ID = 18820]
1. Exchange of goods [Option ID = 45274]
2. Exchange of women [Option ID = 45271]
3. Exchange of money [Option ID = 45272]
4. Exchange of messages [Option ID = 45273]
Correct Answer :-
Exchange of goods [Option ID = 45274]
6)
7)
8)
9)
10)
11)
12)
Which prehistoric site is known as Mode I Industry?
[Question ID = 18835]
1. Pirro Nord [Option ID = 45333]
2. Hunsgi [Option ID = 45331]
3. Tautavel [Option ID = 45332]
4. Somme valley [Option ID = 45334]
Correct Answer :-
Pirro Nord [Option ID = 45333]
Any quantitative increase in body size is known as [Question ID = 18788]
1. Development [Option ID = 45144]
2. Maturation [Option ID = 45146]
3. Differentiation [Option ID = 45145]
4. Growth [Option ID = 45143]
Correct Answer :-
Growth [Option ID = 45143]
The highest point on the head is known as [Question ID = 18807]
1. Opisthocranion [Option ID = 45222]
2. Bregma [Option ID = 45220]
3. Vertex [Option ID = 45221]
4. Lambda [Option ID = 45219]
Correct Answer :-
Vertex [Option ID = 45221]
Autosomes are
[Question ID = 18826]
1. Normal sex chromosomes [Option ID = 45296]
2. Abnormal sex chromosomes [Option ID = 45298]
3. Abnormal chromosomes [Option ID = 45297]
4. All chromosomes other than the sex chromosomes [Option ID = 45295]
Correct Answer :-
All chromosomes other than the sex chromosomes [Option ID = 45295]
?Puberty spurt? occurs in which one of the following stages of human growth [Question ID = 18798]
1. Infancy [Option ID = 45183]
2. Childhood [Option ID = 45185]
3. Adolescence [Option ID = 45186]
4. Adulthood [Option ID = 45184]
Correct Answer :-
Adolescence [Option ID = 45186]
Potlatch is associated with [Question ID = 18802]
1. Iroquois [Option ID = 45201]
2. Kwakiutl [Option ID = 45199]
3. Shoshonean [Option ID = 45202]
4. Arapesh [Option ID = 45200]
Correct Answer :-
Kwakiutl [Option ID = 45199]
Oldowan culture is dominated by
[Question ID = 18824]
1. Blade tools [Option ID = 45288]
2. Flake tools [Option ID = 45289]
13)
14)
15)
16)
17)
18)
3. Pebble tools [Option ID = 45287]
4. Microlithic tools [Option ID = 45290]
Correct Answer :-
Pebble tools [Option ID = 45287]
A higher fortified part of a city is called
[Question ID = 18823]
1. metropolis [Option ID = 45284]
2. megapolis [Option ID = 45285]
3. acropolis [Option ID = 45283]
4. necropolis [Option ID = 45286]
Correct Answer :-
acropolis [Option ID = 45283]
Yale Cambridge expedition to the Sohan Valley was led by
[Question ID = 18795]
1. R. B. Foote and V.D. Krishnaswamy [Option ID = 45171]
2. G. R. Sharma and V.D. Mishra [Option ID = 45174]
3. H. D. Sankalia and B. Subba Rao [Option ID = 45173]
4. H. De. Terra and T. T. Paterson [Option ID = 45172]
Correct Answer :-
H. De. Terra and T. T. Paterson [Option ID = 45172]
Two nuclear families in adjacent generations with one son/husband or daughter/wife who is a member of both the families is called a
[Question ID = 18870]
1. lineal family [Option ID = 45471]
2. supplemented nuclear family [Option ID = 45474]
3. stem family [Option ID = 45473]
4. compound family [Option ID = 45472]
Correct Answer :-
stem family [Option ID = 45473]
The lack, excess or imbalance of nutrients in the diet leading to ill health is known as [Question ID = 18790]
1. Malnutrition [Option ID = 45153]
2. Over nutrition [Option ID = 45151]
3. Under nutrition [Option ID = 45152]
4. Marasmus [Option ID = 45154]
Correct Answer :-
Malnutrition [Option ID = 45153]
Holocene starts around
[Question ID = 18834]
1. 20,000 B.P. [Option ID = 45329]
2. 12,000 B.P. [Option ID = 45328]
3. 14,000 B.P. [Option ID = 45330]
4. 10,000 B.P. [Option ID = 45327]
Correct Answer :-
10,000 B.P. [Option ID = 45327]
According to Levi-Strauss which of the following constitute one of the ?atoms of kinship? relationship?
[Question ID = 18879]
19)
20)
21)
22)
23)
24)
1. mother-son [Option ID = 45507]
2. mother-daughter [Option ID = 45508]
3. father-son [Option ID = 45509]
4. father-daughter [Option ID = 45510]
Correct Answer :-
mother-son [Option ID = 45507]
A karyotype is a:
[Question ID = 18819]
1. Picture of an individual's chromosomes arranged in a standardized way [Option ID = 45269]
2. Type of abnormal chromosome that is associated with Down's syndrome [Option ID = 45268]
3. General term for any type of chromosome [Option ID = 45267]
4. General term for sex chromosome [Option ID = 45270]
Correct Answer :-
Picture of an individual's chromosomes arranged in a standardized way [Option ID = 45269]
Theodicy is an
[Question ID = 18882]
1. attempt to explain theocracy [Option ID = 45521]
2. attempt to explain good [Option ID = 45519]
3. attempt to explain evil [Option ID = 45520]
4. attempt to explain lie and deceit [Option ID = 45522]
Correct Answer :-
attempt to explain evil [Option ID = 45520]
The word ?Somatotype? was coined by [Question ID = 18797]
1. Faulkner [Option ID = 45182]
2. Sheldon [Option ID = 45180]
3. Parnell [Option ID = 45181]
4. Heath and Carter [Option ID = 45179]
Correct Answer :-
Heath and Carter [Option ID = 45179]
Substantivism as an approach in economic anthropology was given by
[Question ID = 18853]
1. A. L. Kroeber [Option ID = 45406]
2. Karl Polanyi [Option ID = 45405]
3. Oscar Lewis [Option ID = 45403]
4. Robert Redfield [Option ID = 62620]
Correct Answer :-
Karl Polanyi [Option ID = 45405]
?Genetics and the Origin of Species? was written by :
[Question ID = 18859]
1. Thomas Malthus. [Option ID = 45430]
2. Theodosius Dobzhansky [Option ID = 45428]
3. Charles Darwin [Option ID = 45427]
4. T. H. Morgan [Option ID = 45429]
Correct Answer :-
Theodosius Dobzhansky [Option ID = 45428]
The ability of the living organisms to survive in a particular ecological set up is called [Question ID = 18809]
25)
26)
27)
28)
29)
30)
1. Acclimation [Option ID = 45228]
2. Acclimatisation [Option ID = 45229]
3. Adaptation [Option ID = 45227]
4. Selection [Option ID = 45230]
Correct Answer :-
Adaptation [Option ID = 45227]
___________is the physiological ability to reproduce an offspring.
[Question ID = 18876]
1. Fecundity [Option ID = 45496]
2. Fertility [Option ID = 45495]
3. Virility [Option ID = 45498]
4. Natality [Option ID = 45497]
Correct Answer :-
Fecundity [Option ID = 45496]
Charles Darwin's ideas concerning the causes of evolution were probably formulated in his mind
[Question ID = 18817]
1. During the late 1880's [Option ID = 45262]
2. During his voyage on H.M.S. Beagle, especially after he reached the Gal?pagos Islands [Option ID = 45261]
3. Before he began his voyage of exploration around the world on H.M.S. Beagle [Option ID = 45260]
4. While he was still a student at Cambridge University [Option ID = 45259]
Correct Answer :-
During his voyage on H.M.S. Beagle, especially after he reached the Gal?pagos Islands [Option ID = 45261]
First Himalayan Glaciation is known as
[Question ID = 18884]
1. Terrace I [Option ID = 45530]
2. Pinjaur [Option ID = 45527]
3. Boulder conglomerate [Option ID = 45529]
4. Tatrot [Option ID = 45528]
Correct Answer :-
Tatrot [Option ID = 45528]
The author of the book Hindus of the Himalayas is [Question ID = 18812]
1. S.C. Dube [Option ID = 45241]
2. D.N. Majumdar [Option ID = 45242]
3. B. K. Roy-Burman [Option ID = 45240]
4. G.D. Berreman [Option ID = 45239]
Correct Answer :-
G.D. Berreman [Option ID = 45239]
Who proposed the ?Three Age System? in pre-history?
[Question ID = 18883]
1. C. J. Thomsen [Option ID = 45525]
2. B. M. Fagan [Option ID = 45523]
3. H.D.Sankalia [Option ID = 45524]
4. F. Borde [Option ID = 45526]
Correct Answer :-
C. J. Thomsen [Option ID = 45525]
Who among the following worked on the complicity between colonial agencies and Anthropologists?
31)
32)
33)
34)
35)
36)
[Question ID = 18801]
1. Ibn Khaldun [Option ID = 45196]
2. Akbar Ahmed [Option ID = 45197]
3. Ralf Linto [Option ID = 45198]
4. Talal Asad [Option ID = 45195]
Correct Answer :-
Talal Asad [Option ID = 45195]
Who is the author of the book ?The Old Stone Age??
[Question ID = 18854]
1. M.C. Burkitt [Option ID = 45407]
2. J. N. Pal [Option ID = 45408]
3. P.C. Pant [Option ID = 45409]
4. H. De Lumley [Option ID = 45410]
Correct Answer :-
M.C. Burkitt [Option ID = 45407]
Who included the Proto-Australoid racial element in the classification of Indian races?
[Question ID = 18866]
1. S.S. Sarkar [Option ID = 45456]
2. H.H. Risley [Option ID = 45455]
3. B.S. Guha [Option ID = 45458]
4. A.C. Haddon [Option ID = 45457]
Correct Answer :-
B.S. Guha [Option ID = 45458]
In which State is the site Isampur located?
[Question ID = 18815]
1. Karnataka [Option ID = 45253]
2. Uttar Pradesh [Option ID = 45254]
3. Madhya Pradesh [Option ID = 45252]
4. Bihar [Option ID = 45251]
Correct Answer :-
Karnataka [Option ID = 45253]
The view of the skull from above is [Question ID = 18800]
1. Norma Basalis [Option ID = 45193]
2. Norma Occipitalis [Option ID = 45194]
3. Norma Frontalis [Option ID = 45192]
4. Norma Verticalis [Option ID = 45191]
Correct Answer :-
Norma Verticalis [Option ID = 45191]
?Childhood? is peculiar to [Question ID = 18789]
1. Nonhuman primates [Option ID = 45149]
2. Human and nonhuman primates [Option ID = 45147]
3. Humans [Option ID = 45148]
4. Mammals [Option ID = 45150]
Correct Answer :-
Humans [Option ID = 45148]
Children subjected to starvation for short periods may recover completely on getting adequate diet. This is referred to as
37)
38)
39)
40)
41)
[Question ID = 18838]
1. Growth spurt [Option ID = 45346]
2. Catch down growth [Option ID = 45345]
3. Catch-up growth [Option ID = 45343]
4. Canalization [Option ID = 45344]
Correct Answer :-
Catch-up growth [Option ID = 45343]
Attirampakkam is [Question ID = 18805]
1. Lower Paleolithic site [Option ID = 45211]
2. Mesolithic site [Option ID = 45214]
3. Indus Valley Civilization site [Option ID = 45213]
4. Upper Paleolithic site [Option ID = 45212]
Correct Answer :-
Lower Paleolithic site [Option ID = 45211]
Shivering
[Question ID = 18828]
1. Has no effect on body heat [Option ID = 45306]
2. Indirectly produces more body heat [Option ID = 45304]
3. Speeds up the loss of body heat [Option ID = 45303]
4. Delays up the loss of body heat [Option ID = 45305]
Correct Answer :-
Indirectly produces more body heat [Option ID = 45304]
A deficiency disease of the skeletal system caused by a lack of Vitamin D or Calcium or both, and often resulting in bone deformities is
[Question ID = 18849]
1. Arthritis [Option ID = 45390]
2. Scurvy [Option ID = 45387]
3. Rickets [Option ID = 45389]
4. Osteoporosis [Option ID = 45388]
Correct Answer :-
Rickets [Option ID = 45389]
A deficiency disease caused due to severe lack of protein in diet among children is
[Question ID = 18856]
1. Anaemia [Option ID = 45418]
2. Kwashiorkor [Option ID = 45417]
3. Scurvy [Option ID = 45416]
4. Rickets [Option ID = 45415]
Correct Answer :-
Kwashiorkor [Option ID = 45417]
Samples of Dendrochronology is taken from which of the following?
[Question ID = 18873]
1. Lake [Option ID = 45483]
2. Tree [Option ID = 45486]
3. River [Option ID = 45484]
4. sea [Option ID = 45485]
Correct Answer :-
Tree [Option ID = 45486]
FirstRanker.com - FirstRanker's Choice
1)
2)
3)
4)
5)
DU MSc Anthropology
Topic:- DU_J18_MSC_ANTHRO
Increased labour investment in agriculture with no increase in per capita productivity is called as
[Question ID = 18814]
1. Agricultural recession [Option ID = 45250]
2. Agricultural stagnation [Option ID = 45247]
3. Agricultural relegation [Option ID = 45248]
4. Agricultural involution [Option ID = 45249]
Correct Answer :-
Agricultural involution [Option ID = 45249]
Which of the following is a hominid?
[Question ID = 18857]
1. Chimpanzee [Option ID = 45421]
2. Human [Option ID = 45422]
3. Macaque [Option ID = 45419]
4. Baboon [Option ID = 45420]
Correct Answer :-
Human [Option ID = 45422]
Which rule says that the best suited human body for a hot, tropical region would be one with long limbs and short trunk?
[Question ID = 18827]
1. Allen?s rule [Option ID = 45299]
2. Thompson?s rule [Option ID = 45302]
3. Gloger?s rule [Option ID = 45301]
4. Bergman?s rule [Option ID = 45300]
Correct Answer :-
Allen?s rule [Option ID = 45299]
Which Australopithecus fossil was nicknamed as Lucy?
[Question ID = 18867]
1. Australopithecus africanus [Option ID = 45459]
2. Australopithecus afarensis [Option ID = 45462]
3. Australopithecus anamensis [Option ID = 45460]
4. Australopithecus robustus [Option ID = 45461]
Correct Answer :-
Australopithecus afarensis [Option ID = 45462]
Which among the following does not constitute the elementary form of exchange?
[Question ID = 18820]
1. Exchange of goods [Option ID = 45274]
2. Exchange of women [Option ID = 45271]
3. Exchange of money [Option ID = 45272]
4. Exchange of messages [Option ID = 45273]
Correct Answer :-
Exchange of goods [Option ID = 45274]
6)
7)
8)
9)
10)
11)
12)
Which prehistoric site is known as Mode I Industry?
[Question ID = 18835]
1. Pirro Nord [Option ID = 45333]
2. Hunsgi [Option ID = 45331]
3. Tautavel [Option ID = 45332]
4. Somme valley [Option ID = 45334]
Correct Answer :-
Pirro Nord [Option ID = 45333]
Any quantitative increase in body size is known as [Question ID = 18788]
1. Development [Option ID = 45144]
2. Maturation [Option ID = 45146]
3. Differentiation [Option ID = 45145]
4. Growth [Option ID = 45143]
Correct Answer :-
Growth [Option ID = 45143]
The highest point on the head is known as [Question ID = 18807]
1. Opisthocranion [Option ID = 45222]
2. Bregma [Option ID = 45220]
3. Vertex [Option ID = 45221]
4. Lambda [Option ID = 45219]
Correct Answer :-
Vertex [Option ID = 45221]
Autosomes are
[Question ID = 18826]
1. Normal sex chromosomes [Option ID = 45296]
2. Abnormal sex chromosomes [Option ID = 45298]
3. Abnormal chromosomes [Option ID = 45297]
4. All chromosomes other than the sex chromosomes [Option ID = 45295]
Correct Answer :-
All chromosomes other than the sex chromosomes [Option ID = 45295]
?Puberty spurt? occurs in which one of the following stages of human growth [Question ID = 18798]
1. Infancy [Option ID = 45183]
2. Childhood [Option ID = 45185]
3. Adolescence [Option ID = 45186]
4. Adulthood [Option ID = 45184]
Correct Answer :-
Adolescence [Option ID = 45186]
Potlatch is associated with [Question ID = 18802]
1. Iroquois [Option ID = 45201]
2. Kwakiutl [Option ID = 45199]
3. Shoshonean [Option ID = 45202]
4. Arapesh [Option ID = 45200]
Correct Answer :-
Kwakiutl [Option ID = 45199]
Oldowan culture is dominated by
[Question ID = 18824]
1. Blade tools [Option ID = 45288]
2. Flake tools [Option ID = 45289]
13)
14)
15)
16)
17)
18)
3. Pebble tools [Option ID = 45287]
4. Microlithic tools [Option ID = 45290]
Correct Answer :-
Pebble tools [Option ID = 45287]
A higher fortified part of a city is called
[Question ID = 18823]
1. metropolis [Option ID = 45284]
2. megapolis [Option ID = 45285]
3. acropolis [Option ID = 45283]
4. necropolis [Option ID = 45286]
Correct Answer :-
acropolis [Option ID = 45283]
Yale Cambridge expedition to the Sohan Valley was led by
[Question ID = 18795]
1. R. B. Foote and V.D. Krishnaswamy [Option ID = 45171]
2. G. R. Sharma and V.D. Mishra [Option ID = 45174]
3. H. D. Sankalia and B. Subba Rao [Option ID = 45173]
4. H. De. Terra and T. T. Paterson [Option ID = 45172]
Correct Answer :-
H. De. Terra and T. T. Paterson [Option ID = 45172]
Two nuclear families in adjacent generations with one son/husband or daughter/wife who is a member of both the families is called a
[Question ID = 18870]
1. lineal family [Option ID = 45471]
2. supplemented nuclear family [Option ID = 45474]
3. stem family [Option ID = 45473]
4. compound family [Option ID = 45472]
Correct Answer :-
stem family [Option ID = 45473]
The lack, excess or imbalance of nutrients in the diet leading to ill health is known as [Question ID = 18790]
1. Malnutrition [Option ID = 45153]
2. Over nutrition [Option ID = 45151]
3. Under nutrition [Option ID = 45152]
4. Marasmus [Option ID = 45154]
Correct Answer :-
Malnutrition [Option ID = 45153]
Holocene starts around
[Question ID = 18834]
1. 20,000 B.P. [Option ID = 45329]
2. 12,000 B.P. [Option ID = 45328]
3. 14,000 B.P. [Option ID = 45330]
4. 10,000 B.P. [Option ID = 45327]
Correct Answer :-
10,000 B.P. [Option ID = 45327]
According to Levi-Strauss which of the following constitute one of the ?atoms of kinship? relationship?
[Question ID = 18879]
19)
20)
21)
22)
23)
24)
1. mother-son [Option ID = 45507]
2. mother-daughter [Option ID = 45508]
3. father-son [Option ID = 45509]
4. father-daughter [Option ID = 45510]
Correct Answer :-
mother-son [Option ID = 45507]
A karyotype is a:
[Question ID = 18819]
1. Picture of an individual's chromosomes arranged in a standardized way [Option ID = 45269]
2. Type of abnormal chromosome that is associated with Down's syndrome [Option ID = 45268]
3. General term for any type of chromosome [Option ID = 45267]
4. General term for sex chromosome [Option ID = 45270]
Correct Answer :-
Picture of an individual's chromosomes arranged in a standardized way [Option ID = 45269]
Theodicy is an
[Question ID = 18882]
1. attempt to explain theocracy [Option ID = 45521]
2. attempt to explain good [Option ID = 45519]
3. attempt to explain evil [Option ID = 45520]
4. attempt to explain lie and deceit [Option ID = 45522]
Correct Answer :-
attempt to explain evil [Option ID = 45520]
The word ?Somatotype? was coined by [Question ID = 18797]
1. Faulkner [Option ID = 45182]
2. Sheldon [Option ID = 45180]
3. Parnell [Option ID = 45181]
4. Heath and Carter [Option ID = 45179]
Correct Answer :-
Heath and Carter [Option ID = 45179]
Substantivism as an approach in economic anthropology was given by
[Question ID = 18853]
1. A. L. Kroeber [Option ID = 45406]
2. Karl Polanyi [Option ID = 45405]
3. Oscar Lewis [Option ID = 45403]
4. Robert Redfield [Option ID = 62620]
Correct Answer :-
Karl Polanyi [Option ID = 45405]
?Genetics and the Origin of Species? was written by :
[Question ID = 18859]
1. Thomas Malthus. [Option ID = 45430]
2. Theodosius Dobzhansky [Option ID = 45428]
3. Charles Darwin [Option ID = 45427]
4. T. H. Morgan [Option ID = 45429]
Correct Answer :-
Theodosius Dobzhansky [Option ID = 45428]
The ability of the living organisms to survive in a particular ecological set up is called [Question ID = 18809]
25)
26)
27)
28)
29)
30)
1. Acclimation [Option ID = 45228]
2. Acclimatisation [Option ID = 45229]
3. Adaptation [Option ID = 45227]
4. Selection [Option ID = 45230]
Correct Answer :-
Adaptation [Option ID = 45227]
___________is the physiological ability to reproduce an offspring.
[Question ID = 18876]
1. Fecundity [Option ID = 45496]
2. Fertility [Option ID = 45495]
3. Virility [Option ID = 45498]
4. Natality [Option ID = 45497]
Correct Answer :-
Fecundity [Option ID = 45496]
Charles Darwin's ideas concerning the causes of evolution were probably formulated in his mind
[Question ID = 18817]
1. During the late 1880's [Option ID = 45262]
2. During his voyage on H.M.S. Beagle, especially after he reached the Gal?pagos Islands [Option ID = 45261]
3. Before he began his voyage of exploration around the world on H.M.S. Beagle [Option ID = 45260]
4. While he was still a student at Cambridge University [Option ID = 45259]
Correct Answer :-
During his voyage on H.M.S. Beagle, especially after he reached the Gal?pagos Islands [Option ID = 45261]
First Himalayan Glaciation is known as
[Question ID = 18884]
1. Terrace I [Option ID = 45530]
2. Pinjaur [Option ID = 45527]
3. Boulder conglomerate [Option ID = 45529]
4. Tatrot [Option ID = 45528]
Correct Answer :-
Tatrot [Option ID = 45528]
The author of the book Hindus of the Himalayas is [Question ID = 18812]
1. S.C. Dube [Option ID = 45241]
2. D.N. Majumdar [Option ID = 45242]
3. B. K. Roy-Burman [Option ID = 45240]
4. G.D. Berreman [Option ID = 45239]
Correct Answer :-
G.D. Berreman [Option ID = 45239]
Who proposed the ?Three Age System? in pre-history?
[Question ID = 18883]
1. C. J. Thomsen [Option ID = 45525]
2. B. M. Fagan [Option ID = 45523]
3. H.D.Sankalia [Option ID = 45524]
4. F. Borde [Option ID = 45526]
Correct Answer :-
C. J. Thomsen [Option ID = 45525]
Who among the following worked on the complicity between colonial agencies and Anthropologists?
31)
32)
33)
34)
35)
36)
[Question ID = 18801]
1. Ibn Khaldun [Option ID = 45196]
2. Akbar Ahmed [Option ID = 45197]
3. Ralf Linto [Option ID = 45198]
4. Talal Asad [Option ID = 45195]
Correct Answer :-
Talal Asad [Option ID = 45195]
Who is the author of the book ?The Old Stone Age??
[Question ID = 18854]
1. M.C. Burkitt [Option ID = 45407]
2. J. N. Pal [Option ID = 45408]
3. P.C. Pant [Option ID = 45409]
4. H. De Lumley [Option ID = 45410]
Correct Answer :-
M.C. Burkitt [Option ID = 45407]
Who included the Proto-Australoid racial element in the classification of Indian races?
[Question ID = 18866]
1. S.S. Sarkar [Option ID = 45456]
2. H.H. Risley [Option ID = 45455]
3. B.S. Guha [Option ID = 45458]
4. A.C. Haddon [Option ID = 45457]
Correct Answer :-
B.S. Guha [Option ID = 45458]
In which State is the site Isampur located?
[Question ID = 18815]
1. Karnataka [Option ID = 45253]
2. Uttar Pradesh [Option ID = 45254]
3. Madhya Pradesh [Option ID = 45252]
4. Bihar [Option ID = 45251]
Correct Answer :-
Karnataka [Option ID = 45253]
The view of the skull from above is [Question ID = 18800]
1. Norma Basalis [Option ID = 45193]
2. Norma Occipitalis [Option ID = 45194]
3. Norma Frontalis [Option ID = 45192]
4. Norma Verticalis [Option ID = 45191]
Correct Answer :-
Norma Verticalis [Option ID = 45191]
?Childhood? is peculiar to [Question ID = 18789]
1. Nonhuman primates [Option ID = 45149]
2. Human and nonhuman primates [Option ID = 45147]
3. Humans [Option ID = 45148]
4. Mammals [Option ID = 45150]
Correct Answer :-
Humans [Option ID = 45148]
Children subjected to starvation for short periods may recover completely on getting adequate diet. This is referred to as
37)
38)
39)
40)
41)
[Question ID = 18838]
1. Growth spurt [Option ID = 45346]
2. Catch down growth [Option ID = 45345]
3. Catch-up growth [Option ID = 45343]
4. Canalization [Option ID = 45344]
Correct Answer :-
Catch-up growth [Option ID = 45343]
Attirampakkam is [Question ID = 18805]
1. Lower Paleolithic site [Option ID = 45211]
2. Mesolithic site [Option ID = 45214]
3. Indus Valley Civilization site [Option ID = 45213]
4. Upper Paleolithic site [Option ID = 45212]
Correct Answer :-
Lower Paleolithic site [Option ID = 45211]
Shivering
[Question ID = 18828]
1. Has no effect on body heat [Option ID = 45306]
2. Indirectly produces more body heat [Option ID = 45304]
3. Speeds up the loss of body heat [Option ID = 45303]
4. Delays up the loss of body heat [Option ID = 45305]
Correct Answer :-
Indirectly produces more body heat [Option ID = 45304]
A deficiency disease of the skeletal system caused by a lack of Vitamin D or Calcium or both, and often resulting in bone deformities is
[Question ID = 18849]
1. Arthritis [Option ID = 45390]
2. Scurvy [Option ID = 45387]
3. Rickets [Option ID = 45389]
4. Osteoporosis [Option ID = 45388]
Correct Answer :-
Rickets [Option ID = 45389]
A deficiency disease caused due to severe lack of protein in diet among children is
[Question ID = 18856]
1. Anaemia [Option ID = 45418]
2. Kwashiorkor [Option ID = 45417]
3. Scurvy [Option ID = 45416]
4. Rickets [Option ID = 45415]
Correct Answer :-
Kwashiorkor [Option ID = 45417]
Samples of Dendrochronology is taken from which of the following?
[Question ID = 18873]
1. Lake [Option ID = 45483]
2. Tree [Option ID = 45486]
3. River [Option ID = 45484]
4. sea [Option ID = 45485]
Correct Answer :-
Tree [Option ID = 45486]
42)
43)
44)
45)
46)
47)
48)
Swiddening is a form of
[Question ID = 18822]
1. terrace cultivation [Option ID = 45281]
2. intensive cultivation [Option ID = 45280]
3. commercial cultivation [Option ID = 45282]
4. slash and burn cultivation [Option ID = 45279]
Correct Answer :-
slash and burn cultivation [Option ID = 45279]
Osteology is the study of [Question ID = 18810]
1. The human skeleton [Option ID = 45234]
2. The human brain [Option ID = 45233]
3. Human survival strategies [Option ID = 45231]
4. Human languages [Option ID = 45232]
Correct Answer :-
The human skeleton [Option ID = 45234]
Extended Case Study method in anthropology is associated with the work of [Question ID = 18813]
1. Paul Radin [Option ID = 45246]
2. Julian Steward [Option ID = 45244]
3. Anthony Walker [Option ID = 45245]
4. Max Gluckman [Option ID = 45243]
Correct Answer :-
Max Gluckman [Option ID = 45243]
Celts are prepared by
[Question ID = 18863]
1. Levalloisian technique [Option ID = 45443]
2. Clactonian technique [Option ID = 45444]
3. Pressure technique [Option ID = 45446]
4. Grinding and polishing technique [Option ID = 45445]
Correct Answer :-
Grinding and polishing technique [Option ID = 45445]
Khallu is a form : [Question ID = 18792]
1. Fishing [Option ID = 45160]
2. Hunting [Option ID = 45159]
3. Cropping [Option ID = 45161]
4. Weaving [Option ID = 45162]
Correct Answer :-
Cropping [Option ID = 45161]
Lascaux is
[Question ID = 18844]
1. Cave art site [Option ID = 45367]
2. Mesolithic site [Option ID = 45368]
3. Chalcolithic site [Option ID = 45369]
4. Neolithic site [Option ID = 45370]
Correct Answer :-
Cave art site [Option ID = 45367]
A social fact is not characterized by
[Question ID = 18862]
FirstRanker.com - FirstRanker's Choice
1)
2)
3)
4)
5)
DU MSc Anthropology
Topic:- DU_J18_MSC_ANTHRO
Increased labour investment in agriculture with no increase in per capita productivity is called as
[Question ID = 18814]
1. Agricultural recession [Option ID = 45250]
2. Agricultural stagnation [Option ID = 45247]
3. Agricultural relegation [Option ID = 45248]
4. Agricultural involution [Option ID = 45249]
Correct Answer :-
Agricultural involution [Option ID = 45249]
Which of the following is a hominid?
[Question ID = 18857]
1. Chimpanzee [Option ID = 45421]
2. Human [Option ID = 45422]
3. Macaque [Option ID = 45419]
4. Baboon [Option ID = 45420]
Correct Answer :-
Human [Option ID = 45422]
Which rule says that the best suited human body for a hot, tropical region would be one with long limbs and short trunk?
[Question ID = 18827]
1. Allen?s rule [Option ID = 45299]
2. Thompson?s rule [Option ID = 45302]
3. Gloger?s rule [Option ID = 45301]
4. Bergman?s rule [Option ID = 45300]
Correct Answer :-
Allen?s rule [Option ID = 45299]
Which Australopithecus fossil was nicknamed as Lucy?
[Question ID = 18867]
1. Australopithecus africanus [Option ID = 45459]
2. Australopithecus afarensis [Option ID = 45462]
3. Australopithecus anamensis [Option ID = 45460]
4. Australopithecus robustus [Option ID = 45461]
Correct Answer :-
Australopithecus afarensis [Option ID = 45462]
Which among the following does not constitute the elementary form of exchange?
[Question ID = 18820]
1. Exchange of goods [Option ID = 45274]
2. Exchange of women [Option ID = 45271]
3. Exchange of money [Option ID = 45272]
4. Exchange of messages [Option ID = 45273]
Correct Answer :-
Exchange of goods [Option ID = 45274]
6)
7)
8)
9)
10)
11)
12)
Which prehistoric site is known as Mode I Industry?
[Question ID = 18835]
1. Pirro Nord [Option ID = 45333]
2. Hunsgi [Option ID = 45331]
3. Tautavel [Option ID = 45332]
4. Somme valley [Option ID = 45334]
Correct Answer :-
Pirro Nord [Option ID = 45333]
Any quantitative increase in body size is known as [Question ID = 18788]
1. Development [Option ID = 45144]
2. Maturation [Option ID = 45146]
3. Differentiation [Option ID = 45145]
4. Growth [Option ID = 45143]
Correct Answer :-
Growth [Option ID = 45143]
The highest point on the head is known as [Question ID = 18807]
1. Opisthocranion [Option ID = 45222]
2. Bregma [Option ID = 45220]
3. Vertex [Option ID = 45221]
4. Lambda [Option ID = 45219]
Correct Answer :-
Vertex [Option ID = 45221]
Autosomes are
[Question ID = 18826]
1. Normal sex chromosomes [Option ID = 45296]
2. Abnormal sex chromosomes [Option ID = 45298]
3. Abnormal chromosomes [Option ID = 45297]
4. All chromosomes other than the sex chromosomes [Option ID = 45295]
Correct Answer :-
All chromosomes other than the sex chromosomes [Option ID = 45295]
?Puberty spurt? occurs in which one of the following stages of human growth [Question ID = 18798]
1. Infancy [Option ID = 45183]
2. Childhood [Option ID = 45185]
3. Adolescence [Option ID = 45186]
4. Adulthood [Option ID = 45184]
Correct Answer :-
Adolescence [Option ID = 45186]
Potlatch is associated with [Question ID = 18802]
1. Iroquois [Option ID = 45201]
2. Kwakiutl [Option ID = 45199]
3. Shoshonean [Option ID = 45202]
4. Arapesh [Option ID = 45200]
Correct Answer :-
Kwakiutl [Option ID = 45199]
Oldowan culture is dominated by
[Question ID = 18824]
1. Blade tools [Option ID = 45288]
2. Flake tools [Option ID = 45289]
13)
14)
15)
16)
17)
18)
3. Pebble tools [Option ID = 45287]
4. Microlithic tools [Option ID = 45290]
Correct Answer :-
Pebble tools [Option ID = 45287]
A higher fortified part of a city is called
[Question ID = 18823]
1. metropolis [Option ID = 45284]
2. megapolis [Option ID = 45285]
3. acropolis [Option ID = 45283]
4. necropolis [Option ID = 45286]
Correct Answer :-
acropolis [Option ID = 45283]
Yale Cambridge expedition to the Sohan Valley was led by
[Question ID = 18795]
1. R. B. Foote and V.D. Krishnaswamy [Option ID = 45171]
2. G. R. Sharma and V.D. Mishra [Option ID = 45174]
3. H. D. Sankalia and B. Subba Rao [Option ID = 45173]
4. H. De. Terra and T. T. Paterson [Option ID = 45172]
Correct Answer :-
H. De. Terra and T. T. Paterson [Option ID = 45172]
Two nuclear families in adjacent generations with one son/husband or daughter/wife who is a member of both the families is called a
[Question ID = 18870]
1. lineal family [Option ID = 45471]
2. supplemented nuclear family [Option ID = 45474]
3. stem family [Option ID = 45473]
4. compound family [Option ID = 45472]
Correct Answer :-
stem family [Option ID = 45473]
The lack, excess or imbalance of nutrients in the diet leading to ill health is known as [Question ID = 18790]
1. Malnutrition [Option ID = 45153]
2. Over nutrition [Option ID = 45151]
3. Under nutrition [Option ID = 45152]
4. Marasmus [Option ID = 45154]
Correct Answer :-
Malnutrition [Option ID = 45153]
Holocene starts around
[Question ID = 18834]
1. 20,000 B.P. [Option ID = 45329]
2. 12,000 B.P. [Option ID = 45328]
3. 14,000 B.P. [Option ID = 45330]
4. 10,000 B.P. [Option ID = 45327]
Correct Answer :-
10,000 B.P. [Option ID = 45327]
According to Levi-Strauss which of the following constitute one of the ?atoms of kinship? relationship?
[Question ID = 18879]
19)
20)
21)
22)
23)
24)
1. mother-son [Option ID = 45507]
2. mother-daughter [Option ID = 45508]
3. father-son [Option ID = 45509]
4. father-daughter [Option ID = 45510]
Correct Answer :-
mother-son [Option ID = 45507]
A karyotype is a:
[Question ID = 18819]
1. Picture of an individual's chromosomes arranged in a standardized way [Option ID = 45269]
2. Type of abnormal chromosome that is associated with Down's syndrome [Option ID = 45268]
3. General term for any type of chromosome [Option ID = 45267]
4. General term for sex chromosome [Option ID = 45270]
Correct Answer :-
Picture of an individual's chromosomes arranged in a standardized way [Option ID = 45269]
Theodicy is an
[Question ID = 18882]
1. attempt to explain theocracy [Option ID = 45521]
2. attempt to explain good [Option ID = 45519]
3. attempt to explain evil [Option ID = 45520]
4. attempt to explain lie and deceit [Option ID = 45522]
Correct Answer :-
attempt to explain evil [Option ID = 45520]
The word ?Somatotype? was coined by [Question ID = 18797]
1. Faulkner [Option ID = 45182]
2. Sheldon [Option ID = 45180]
3. Parnell [Option ID = 45181]
4. Heath and Carter [Option ID = 45179]
Correct Answer :-
Heath and Carter [Option ID = 45179]
Substantivism as an approach in economic anthropology was given by
[Question ID = 18853]
1. A. L. Kroeber [Option ID = 45406]
2. Karl Polanyi [Option ID = 45405]
3. Oscar Lewis [Option ID = 45403]
4. Robert Redfield [Option ID = 62620]
Correct Answer :-
Karl Polanyi [Option ID = 45405]
?Genetics and the Origin of Species? was written by :
[Question ID = 18859]
1. Thomas Malthus. [Option ID = 45430]
2. Theodosius Dobzhansky [Option ID = 45428]
3. Charles Darwin [Option ID = 45427]
4. T. H. Morgan [Option ID = 45429]
Correct Answer :-
Theodosius Dobzhansky [Option ID = 45428]
The ability of the living organisms to survive in a particular ecological set up is called [Question ID = 18809]
25)
26)
27)
28)
29)
30)
1. Acclimation [Option ID = 45228]
2. Acclimatisation [Option ID = 45229]
3. Adaptation [Option ID = 45227]
4. Selection [Option ID = 45230]
Correct Answer :-
Adaptation [Option ID = 45227]
___________is the physiological ability to reproduce an offspring.
[Question ID = 18876]
1. Fecundity [Option ID = 45496]
2. Fertility [Option ID = 45495]
3. Virility [Option ID = 45498]
4. Natality [Option ID = 45497]
Correct Answer :-
Fecundity [Option ID = 45496]
Charles Darwin's ideas concerning the causes of evolution were probably formulated in his mind
[Question ID = 18817]
1. During the late 1880's [Option ID = 45262]
2. During his voyage on H.M.S. Beagle, especially after he reached the Gal?pagos Islands [Option ID = 45261]
3. Before he began his voyage of exploration around the world on H.M.S. Beagle [Option ID = 45260]
4. While he was still a student at Cambridge University [Option ID = 45259]
Correct Answer :-
During his voyage on H.M.S. Beagle, especially after he reached the Gal?pagos Islands [Option ID = 45261]
First Himalayan Glaciation is known as
[Question ID = 18884]
1. Terrace I [Option ID = 45530]
2. Pinjaur [Option ID = 45527]
3. Boulder conglomerate [Option ID = 45529]
4. Tatrot [Option ID = 45528]
Correct Answer :-
Tatrot [Option ID = 45528]
The author of the book Hindus of the Himalayas is [Question ID = 18812]
1. S.C. Dube [Option ID = 45241]
2. D.N. Majumdar [Option ID = 45242]
3. B. K. Roy-Burman [Option ID = 45240]
4. G.D. Berreman [Option ID = 45239]
Correct Answer :-
G.D. Berreman [Option ID = 45239]
Who proposed the ?Three Age System? in pre-history?
[Question ID = 18883]
1. C. J. Thomsen [Option ID = 45525]
2. B. M. Fagan [Option ID = 45523]
3. H.D.Sankalia [Option ID = 45524]
4. F. Borde [Option ID = 45526]
Correct Answer :-
C. J. Thomsen [Option ID = 45525]
Who among the following worked on the complicity between colonial agencies and Anthropologists?
31)
32)
33)
34)
35)
36)
[Question ID = 18801]
1. Ibn Khaldun [Option ID = 45196]
2. Akbar Ahmed [Option ID = 45197]
3. Ralf Linto [Option ID = 45198]
4. Talal Asad [Option ID = 45195]
Correct Answer :-
Talal Asad [Option ID = 45195]
Who is the author of the book ?The Old Stone Age??
[Question ID = 18854]
1. M.C. Burkitt [Option ID = 45407]
2. J. N. Pal [Option ID = 45408]
3. P.C. Pant [Option ID = 45409]
4. H. De Lumley [Option ID = 45410]
Correct Answer :-
M.C. Burkitt [Option ID = 45407]
Who included the Proto-Australoid racial element in the classification of Indian races?
[Question ID = 18866]
1. S.S. Sarkar [Option ID = 45456]
2. H.H. Risley [Option ID = 45455]
3. B.S. Guha [Option ID = 45458]
4. A.C. Haddon [Option ID = 45457]
Correct Answer :-
B.S. Guha [Option ID = 45458]
In which State is the site Isampur located?
[Question ID = 18815]
1. Karnataka [Option ID = 45253]
2. Uttar Pradesh [Option ID = 45254]
3. Madhya Pradesh [Option ID = 45252]
4. Bihar [Option ID = 45251]
Correct Answer :-
Karnataka [Option ID = 45253]
The view of the skull from above is [Question ID = 18800]
1. Norma Basalis [Option ID = 45193]
2. Norma Occipitalis [Option ID = 45194]
3. Norma Frontalis [Option ID = 45192]
4. Norma Verticalis [Option ID = 45191]
Correct Answer :-
Norma Verticalis [Option ID = 45191]
?Childhood? is peculiar to [Question ID = 18789]
1. Nonhuman primates [Option ID = 45149]
2. Human and nonhuman primates [Option ID = 45147]
3. Humans [Option ID = 45148]
4. Mammals [Option ID = 45150]
Correct Answer :-
Humans [Option ID = 45148]
Children subjected to starvation for short periods may recover completely on getting adequate diet. This is referred to as
37)
38)
39)
40)
41)
[Question ID = 18838]
1. Growth spurt [Option ID = 45346]
2. Catch down growth [Option ID = 45345]
3. Catch-up growth [Option ID = 45343]
4. Canalization [Option ID = 45344]
Correct Answer :-
Catch-up growth [Option ID = 45343]
Attirampakkam is [Question ID = 18805]
1. Lower Paleolithic site [Option ID = 45211]
2. Mesolithic site [Option ID = 45214]
3. Indus Valley Civilization site [Option ID = 45213]
4. Upper Paleolithic site [Option ID = 45212]
Correct Answer :-
Lower Paleolithic site [Option ID = 45211]
Shivering
[Question ID = 18828]
1. Has no effect on body heat [Option ID = 45306]
2. Indirectly produces more body heat [Option ID = 45304]
3. Speeds up the loss of body heat [Option ID = 45303]
4. Delays up the loss of body heat [Option ID = 45305]
Correct Answer :-
Indirectly produces more body heat [Option ID = 45304]
A deficiency disease of the skeletal system caused by a lack of Vitamin D or Calcium or both, and often resulting in bone deformities is
[Question ID = 18849]
1. Arthritis [Option ID = 45390]
2. Scurvy [Option ID = 45387]
3. Rickets [Option ID = 45389]
4. Osteoporosis [Option ID = 45388]
Correct Answer :-
Rickets [Option ID = 45389]
A deficiency disease caused due to severe lack of protein in diet among children is
[Question ID = 18856]
1. Anaemia [Option ID = 45418]
2. Kwashiorkor [Option ID = 45417]
3. Scurvy [Option ID = 45416]
4. Rickets [Option ID = 45415]
Correct Answer :-
Kwashiorkor [Option ID = 45417]
Samples of Dendrochronology is taken from which of the following?
[Question ID = 18873]
1. Lake [Option ID = 45483]
2. Tree [Option ID = 45486]
3. River [Option ID = 45484]
4. sea [Option ID = 45485]
Correct Answer :-
Tree [Option ID = 45486]
42)
43)
44)
45)
46)
47)
48)
Swiddening is a form of
[Question ID = 18822]
1. terrace cultivation [Option ID = 45281]
2. intensive cultivation [Option ID = 45280]
3. commercial cultivation [Option ID = 45282]
4. slash and burn cultivation [Option ID = 45279]
Correct Answer :-
slash and burn cultivation [Option ID = 45279]
Osteology is the study of [Question ID = 18810]
1. The human skeleton [Option ID = 45234]
2. The human brain [Option ID = 45233]
3. Human survival strategies [Option ID = 45231]
4. Human languages [Option ID = 45232]
Correct Answer :-
The human skeleton [Option ID = 45234]
Extended Case Study method in anthropology is associated with the work of [Question ID = 18813]
1. Paul Radin [Option ID = 45246]
2. Julian Steward [Option ID = 45244]
3. Anthony Walker [Option ID = 45245]
4. Max Gluckman [Option ID = 45243]
Correct Answer :-
Max Gluckman [Option ID = 45243]
Celts are prepared by
[Question ID = 18863]
1. Levalloisian technique [Option ID = 45443]
2. Clactonian technique [Option ID = 45444]
3. Pressure technique [Option ID = 45446]
4. Grinding and polishing technique [Option ID = 45445]
Correct Answer :-
Grinding and polishing technique [Option ID = 45445]
Khallu is a form : [Question ID = 18792]
1. Fishing [Option ID = 45160]
2. Hunting [Option ID = 45159]
3. Cropping [Option ID = 45161]
4. Weaving [Option ID = 45162]
Correct Answer :-
Cropping [Option ID = 45161]
Lascaux is
[Question ID = 18844]
1. Cave art site [Option ID = 45367]
2. Mesolithic site [Option ID = 45368]
3. Chalcolithic site [Option ID = 45369]
4. Neolithic site [Option ID = 45370]
Correct Answer :-
Cave art site [Option ID = 45367]
A social fact is not characterized by
[Question ID = 18862]
49)
50)
51)
52)
53)
54)
1. Inheritance [Option ID = 45441]
2. Independent existence [Option ID = 45439]
3. Constraints [Option ID = 45442]
4. Exteriority [Option ID = 45440]
Correct Answer :-
Inheritance [Option ID = 45441]
Chopper and Chopping tool are
[Question ID = 18864]
1. blade tool [Option ID = 45447]
2. flake tool [Option ID = 45449]
3. core tool [Option ID = 45448]
4. Neolithic tools [Option ID = 45450]
Correct Answer :-
core tool [Option ID = 45448]
The theory of evolution by natural selection was independently developed by: [Question ID = 19246]
1. Darwin and Lamarck [Option ID = 46977]
2. Darwin and Wallace [Option ID = 46976]
3. Lyell and Darwin [Option ID = 46975]
4. Lyell and Hutton [Option ID = 46978]
Correct Answer :-
Darwin and Lamarck [Option ID = 46977]
? Linea aspera? responsible for the erect posture in man is present in
[Question ID = 18846]
1. Anterior side of tibia [Option ID = 45376]
2. Anterior side of femur [Option ID = 45378]
3. Posterior side of femur [Option ID = 45375]
4. Medial side of fibula [Option ID = 45377]
Correct Answer :-
Posterior side of femur [Option ID = 45375]
?Purity and Danger? is associated with the work of
[Question ID = 18842]
1. M.N. Srinivas [Option ID = 45362]
2. Mary Douglas [Option ID = 45360]
3. Victor Turner [Option ID = 45359]
4. Edmund R. Leach [Option ID = 45361]
Correct Answer :-
Mary Douglas [Option ID = 45360]
?Swaddling Hypothesis? was used by the
[Question ID = 18832]
1. Evolutionist [Option ID = 45320]
2. Conflict Theorist [Option ID = 45321]
3. Cultural Ecologist [Option ID = 45319]
4. Cultural Personality Thinker [Option ID = 45322]
Correct Answer :-
Cultural Personality Thinker [Option ID = 45322]
Trapeze is [Question ID = 18806]
FirstRanker.com - FirstRanker's Choice
1)
2)
3)
4)
5)
DU MSc Anthropology
Topic:- DU_J18_MSC_ANTHRO
Increased labour investment in agriculture with no increase in per capita productivity is called as
[Question ID = 18814]
1. Agricultural recession [Option ID = 45250]
2. Agricultural stagnation [Option ID = 45247]
3. Agricultural relegation [Option ID = 45248]
4. Agricultural involution [Option ID = 45249]
Correct Answer :-
Agricultural involution [Option ID = 45249]
Which of the following is a hominid?
[Question ID = 18857]
1. Chimpanzee [Option ID = 45421]
2. Human [Option ID = 45422]
3. Macaque [Option ID = 45419]
4. Baboon [Option ID = 45420]
Correct Answer :-
Human [Option ID = 45422]
Which rule says that the best suited human body for a hot, tropical region would be one with long limbs and short trunk?
[Question ID = 18827]
1. Allen?s rule [Option ID = 45299]
2. Thompson?s rule [Option ID = 45302]
3. Gloger?s rule [Option ID = 45301]
4. Bergman?s rule [Option ID = 45300]
Correct Answer :-
Allen?s rule [Option ID = 45299]
Which Australopithecus fossil was nicknamed as Lucy?
[Question ID = 18867]
1. Australopithecus africanus [Option ID = 45459]
2. Australopithecus afarensis [Option ID = 45462]
3. Australopithecus anamensis [Option ID = 45460]
4. Australopithecus robustus [Option ID = 45461]
Correct Answer :-
Australopithecus afarensis [Option ID = 45462]
Which among the following does not constitute the elementary form of exchange?
[Question ID = 18820]
1. Exchange of goods [Option ID = 45274]
2. Exchange of women [Option ID = 45271]
3. Exchange of money [Option ID = 45272]
4. Exchange of messages [Option ID = 45273]
Correct Answer :-
Exchange of goods [Option ID = 45274]
6)
7)
8)
9)
10)
11)
12)
Which prehistoric site is known as Mode I Industry?
[Question ID = 18835]
1. Pirro Nord [Option ID = 45333]
2. Hunsgi [Option ID = 45331]
3. Tautavel [Option ID = 45332]
4. Somme valley [Option ID = 45334]
Correct Answer :-
Pirro Nord [Option ID = 45333]
Any quantitative increase in body size is known as [Question ID = 18788]
1. Development [Option ID = 45144]
2. Maturation [Option ID = 45146]
3. Differentiation [Option ID = 45145]
4. Growth [Option ID = 45143]
Correct Answer :-
Growth [Option ID = 45143]
The highest point on the head is known as [Question ID = 18807]
1. Opisthocranion [Option ID = 45222]
2. Bregma [Option ID = 45220]
3. Vertex [Option ID = 45221]
4. Lambda [Option ID = 45219]
Correct Answer :-
Vertex [Option ID = 45221]
Autosomes are
[Question ID = 18826]
1. Normal sex chromosomes [Option ID = 45296]
2. Abnormal sex chromosomes [Option ID = 45298]
3. Abnormal chromosomes [Option ID = 45297]
4. All chromosomes other than the sex chromosomes [Option ID = 45295]
Correct Answer :-
All chromosomes other than the sex chromosomes [Option ID = 45295]
?Puberty spurt? occurs in which one of the following stages of human growth [Question ID = 18798]
1. Infancy [Option ID = 45183]
2. Childhood [Option ID = 45185]
3. Adolescence [Option ID = 45186]
4. Adulthood [Option ID = 45184]
Correct Answer :-
Adolescence [Option ID = 45186]
Potlatch is associated with [Question ID = 18802]
1. Iroquois [Option ID = 45201]
2. Kwakiutl [Option ID = 45199]
3. Shoshonean [Option ID = 45202]
4. Arapesh [Option ID = 45200]
Correct Answer :-
Kwakiutl [Option ID = 45199]
Oldowan culture is dominated by
[Question ID = 18824]
1. Blade tools [Option ID = 45288]
2. Flake tools [Option ID = 45289]
13)
14)
15)
16)
17)
18)
3. Pebble tools [Option ID = 45287]
4. Microlithic tools [Option ID = 45290]
Correct Answer :-
Pebble tools [Option ID = 45287]
A higher fortified part of a city is called
[Question ID = 18823]
1. metropolis [Option ID = 45284]
2. megapolis [Option ID = 45285]
3. acropolis [Option ID = 45283]
4. necropolis [Option ID = 45286]
Correct Answer :-
acropolis [Option ID = 45283]
Yale Cambridge expedition to the Sohan Valley was led by
[Question ID = 18795]
1. R. B. Foote and V.D. Krishnaswamy [Option ID = 45171]
2. G. R. Sharma and V.D. Mishra [Option ID = 45174]
3. H. D. Sankalia and B. Subba Rao [Option ID = 45173]
4. H. De. Terra and T. T. Paterson [Option ID = 45172]
Correct Answer :-
H. De. Terra and T. T. Paterson [Option ID = 45172]
Two nuclear families in adjacent generations with one son/husband or daughter/wife who is a member of both the families is called a
[Question ID = 18870]
1. lineal family [Option ID = 45471]
2. supplemented nuclear family [Option ID = 45474]
3. stem family [Option ID = 45473]
4. compound family [Option ID = 45472]
Correct Answer :-
stem family [Option ID = 45473]
The lack, excess or imbalance of nutrients in the diet leading to ill health is known as [Question ID = 18790]
1. Malnutrition [Option ID = 45153]
2. Over nutrition [Option ID = 45151]
3. Under nutrition [Option ID = 45152]
4. Marasmus [Option ID = 45154]
Correct Answer :-
Malnutrition [Option ID = 45153]
Holocene starts around
[Question ID = 18834]
1. 20,000 B.P. [Option ID = 45329]
2. 12,000 B.P. [Option ID = 45328]
3. 14,000 B.P. [Option ID = 45330]
4. 10,000 B.P. [Option ID = 45327]
Correct Answer :-
10,000 B.P. [Option ID = 45327]
According to Levi-Strauss which of the following constitute one of the ?atoms of kinship? relationship?
[Question ID = 18879]
19)
20)
21)
22)
23)
24)
1. mother-son [Option ID = 45507]
2. mother-daughter [Option ID = 45508]
3. father-son [Option ID = 45509]
4. father-daughter [Option ID = 45510]
Correct Answer :-
mother-son [Option ID = 45507]
A karyotype is a:
[Question ID = 18819]
1. Picture of an individual's chromosomes arranged in a standardized way [Option ID = 45269]
2. Type of abnormal chromosome that is associated with Down's syndrome [Option ID = 45268]
3. General term for any type of chromosome [Option ID = 45267]
4. General term for sex chromosome [Option ID = 45270]
Correct Answer :-
Picture of an individual's chromosomes arranged in a standardized way [Option ID = 45269]
Theodicy is an
[Question ID = 18882]
1. attempt to explain theocracy [Option ID = 45521]
2. attempt to explain good [Option ID = 45519]
3. attempt to explain evil [Option ID = 45520]
4. attempt to explain lie and deceit [Option ID = 45522]
Correct Answer :-
attempt to explain evil [Option ID = 45520]
The word ?Somatotype? was coined by [Question ID = 18797]
1. Faulkner [Option ID = 45182]
2. Sheldon [Option ID = 45180]
3. Parnell [Option ID = 45181]
4. Heath and Carter [Option ID = 45179]
Correct Answer :-
Heath and Carter [Option ID = 45179]
Substantivism as an approach in economic anthropology was given by
[Question ID = 18853]
1. A. L. Kroeber [Option ID = 45406]
2. Karl Polanyi [Option ID = 45405]
3. Oscar Lewis [Option ID = 45403]
4. Robert Redfield [Option ID = 62620]
Correct Answer :-
Karl Polanyi [Option ID = 45405]
?Genetics and the Origin of Species? was written by :
[Question ID = 18859]
1. Thomas Malthus. [Option ID = 45430]
2. Theodosius Dobzhansky [Option ID = 45428]
3. Charles Darwin [Option ID = 45427]
4. T. H. Morgan [Option ID = 45429]
Correct Answer :-
Theodosius Dobzhansky [Option ID = 45428]
The ability of the living organisms to survive in a particular ecological set up is called [Question ID = 18809]
25)
26)
27)
28)
29)
30)
1. Acclimation [Option ID = 45228]
2. Acclimatisation [Option ID = 45229]
3. Adaptation [Option ID = 45227]
4. Selection [Option ID = 45230]
Correct Answer :-
Adaptation [Option ID = 45227]
___________is the physiological ability to reproduce an offspring.
[Question ID = 18876]
1. Fecundity [Option ID = 45496]
2. Fertility [Option ID = 45495]
3. Virility [Option ID = 45498]
4. Natality [Option ID = 45497]
Correct Answer :-
Fecundity [Option ID = 45496]
Charles Darwin's ideas concerning the causes of evolution were probably formulated in his mind
[Question ID = 18817]
1. During the late 1880's [Option ID = 45262]
2. During his voyage on H.M.S. Beagle, especially after he reached the Gal?pagos Islands [Option ID = 45261]
3. Before he began his voyage of exploration around the world on H.M.S. Beagle [Option ID = 45260]
4. While he was still a student at Cambridge University [Option ID = 45259]
Correct Answer :-
During his voyage on H.M.S. Beagle, especially after he reached the Gal?pagos Islands [Option ID = 45261]
First Himalayan Glaciation is known as
[Question ID = 18884]
1. Terrace I [Option ID = 45530]
2. Pinjaur [Option ID = 45527]
3. Boulder conglomerate [Option ID = 45529]
4. Tatrot [Option ID = 45528]
Correct Answer :-
Tatrot [Option ID = 45528]
The author of the book Hindus of the Himalayas is [Question ID = 18812]
1. S.C. Dube [Option ID = 45241]
2. D.N. Majumdar [Option ID = 45242]
3. B. K. Roy-Burman [Option ID = 45240]
4. G.D. Berreman [Option ID = 45239]
Correct Answer :-
G.D. Berreman [Option ID = 45239]
Who proposed the ?Three Age System? in pre-history?
[Question ID = 18883]
1. C. J. Thomsen [Option ID = 45525]
2. B. M. Fagan [Option ID = 45523]
3. H.D.Sankalia [Option ID = 45524]
4. F. Borde [Option ID = 45526]
Correct Answer :-
C. J. Thomsen [Option ID = 45525]
Who among the following worked on the complicity between colonial agencies and Anthropologists?
31)
32)
33)
34)
35)
36)
[Question ID = 18801]
1. Ibn Khaldun [Option ID = 45196]
2. Akbar Ahmed [Option ID = 45197]
3. Ralf Linto [Option ID = 45198]
4. Talal Asad [Option ID = 45195]
Correct Answer :-
Talal Asad [Option ID = 45195]
Who is the author of the book ?The Old Stone Age??
[Question ID = 18854]
1. M.C. Burkitt [Option ID = 45407]
2. J. N. Pal [Option ID = 45408]
3. P.C. Pant [Option ID = 45409]
4. H. De Lumley [Option ID = 45410]
Correct Answer :-
M.C. Burkitt [Option ID = 45407]
Who included the Proto-Australoid racial element in the classification of Indian races?
[Question ID = 18866]
1. S.S. Sarkar [Option ID = 45456]
2. H.H. Risley [Option ID = 45455]
3. B.S. Guha [Option ID = 45458]
4. A.C. Haddon [Option ID = 45457]
Correct Answer :-
B.S. Guha [Option ID = 45458]
In which State is the site Isampur located?
[Question ID = 18815]
1. Karnataka [Option ID = 45253]
2. Uttar Pradesh [Option ID = 45254]
3. Madhya Pradesh [Option ID = 45252]
4. Bihar [Option ID = 45251]
Correct Answer :-
Karnataka [Option ID = 45253]
The view of the skull from above is [Question ID = 18800]
1. Norma Basalis [Option ID = 45193]
2. Norma Occipitalis [Option ID = 45194]
3. Norma Frontalis [Option ID = 45192]
4. Norma Verticalis [Option ID = 45191]
Correct Answer :-
Norma Verticalis [Option ID = 45191]
?Childhood? is peculiar to [Question ID = 18789]
1. Nonhuman primates [Option ID = 45149]
2. Human and nonhuman primates [Option ID = 45147]
3. Humans [Option ID = 45148]
4. Mammals [Option ID = 45150]
Correct Answer :-
Humans [Option ID = 45148]
Children subjected to starvation for short periods may recover completely on getting adequate diet. This is referred to as
37)
38)
39)
40)
41)
[Question ID = 18838]
1. Growth spurt [Option ID = 45346]
2. Catch down growth [Option ID = 45345]
3. Catch-up growth [Option ID = 45343]
4. Canalization [Option ID = 45344]
Correct Answer :-
Catch-up growth [Option ID = 45343]
Attirampakkam is [Question ID = 18805]
1. Lower Paleolithic site [Option ID = 45211]
2. Mesolithic site [Option ID = 45214]
3. Indus Valley Civilization site [Option ID = 45213]
4. Upper Paleolithic site [Option ID = 45212]
Correct Answer :-
Lower Paleolithic site [Option ID = 45211]
Shivering
[Question ID = 18828]
1. Has no effect on body heat [Option ID = 45306]
2. Indirectly produces more body heat [Option ID = 45304]
3. Speeds up the loss of body heat [Option ID = 45303]
4. Delays up the loss of body heat [Option ID = 45305]
Correct Answer :-
Indirectly produces more body heat [Option ID = 45304]
A deficiency disease of the skeletal system caused by a lack of Vitamin D or Calcium or both, and often resulting in bone deformities is
[Question ID = 18849]
1. Arthritis [Option ID = 45390]
2. Scurvy [Option ID = 45387]
3. Rickets [Option ID = 45389]
4. Osteoporosis [Option ID = 45388]
Correct Answer :-
Rickets [Option ID = 45389]
A deficiency disease caused due to severe lack of protein in diet among children is
[Question ID = 18856]
1. Anaemia [Option ID = 45418]
2. Kwashiorkor [Option ID = 45417]
3. Scurvy [Option ID = 45416]
4. Rickets [Option ID = 45415]
Correct Answer :-
Kwashiorkor [Option ID = 45417]
Samples of Dendrochronology is taken from which of the following?
[Question ID = 18873]
1. Lake [Option ID = 45483]
2. Tree [Option ID = 45486]
3. River [Option ID = 45484]
4. sea [Option ID = 45485]
Correct Answer :-
Tree [Option ID = 45486]
42)
43)
44)
45)
46)
47)
48)
Swiddening is a form of
[Question ID = 18822]
1. terrace cultivation [Option ID = 45281]
2. intensive cultivation [Option ID = 45280]
3. commercial cultivation [Option ID = 45282]
4. slash and burn cultivation [Option ID = 45279]
Correct Answer :-
slash and burn cultivation [Option ID = 45279]
Osteology is the study of [Question ID = 18810]
1. The human skeleton [Option ID = 45234]
2. The human brain [Option ID = 45233]
3. Human survival strategies [Option ID = 45231]
4. Human languages [Option ID = 45232]
Correct Answer :-
The human skeleton [Option ID = 45234]
Extended Case Study method in anthropology is associated with the work of [Question ID = 18813]
1. Paul Radin [Option ID = 45246]
2. Julian Steward [Option ID = 45244]
3. Anthony Walker [Option ID = 45245]
4. Max Gluckman [Option ID = 45243]
Correct Answer :-
Max Gluckman [Option ID = 45243]
Celts are prepared by
[Question ID = 18863]
1. Levalloisian technique [Option ID = 45443]
2. Clactonian technique [Option ID = 45444]
3. Pressure technique [Option ID = 45446]
4. Grinding and polishing technique [Option ID = 45445]
Correct Answer :-
Grinding and polishing technique [Option ID = 45445]
Khallu is a form : [Question ID = 18792]
1. Fishing [Option ID = 45160]
2. Hunting [Option ID = 45159]
3. Cropping [Option ID = 45161]
4. Weaving [Option ID = 45162]
Correct Answer :-
Cropping [Option ID = 45161]
Lascaux is
[Question ID = 18844]
1. Cave art site [Option ID = 45367]
2. Mesolithic site [Option ID = 45368]
3. Chalcolithic site [Option ID = 45369]
4. Neolithic site [Option ID = 45370]
Correct Answer :-
Cave art site [Option ID = 45367]
A social fact is not characterized by
[Question ID = 18862]
49)
50)
51)
52)
53)
54)
1. Inheritance [Option ID = 45441]
2. Independent existence [Option ID = 45439]
3. Constraints [Option ID = 45442]
4. Exteriority [Option ID = 45440]
Correct Answer :-
Inheritance [Option ID = 45441]
Chopper and Chopping tool are
[Question ID = 18864]
1. blade tool [Option ID = 45447]
2. flake tool [Option ID = 45449]
3. core tool [Option ID = 45448]
4. Neolithic tools [Option ID = 45450]
Correct Answer :-
core tool [Option ID = 45448]
The theory of evolution by natural selection was independently developed by: [Question ID = 19246]
1. Darwin and Lamarck [Option ID = 46977]
2. Darwin and Wallace [Option ID = 46976]
3. Lyell and Darwin [Option ID = 46975]
4. Lyell and Hutton [Option ID = 46978]
Correct Answer :-
Darwin and Lamarck [Option ID = 46977]
? Linea aspera? responsible for the erect posture in man is present in
[Question ID = 18846]
1. Anterior side of tibia [Option ID = 45376]
2. Anterior side of femur [Option ID = 45378]
3. Posterior side of femur [Option ID = 45375]
4. Medial side of fibula [Option ID = 45377]
Correct Answer :-
Posterior side of femur [Option ID = 45375]
?Purity and Danger? is associated with the work of
[Question ID = 18842]
1. M.N. Srinivas [Option ID = 45362]
2. Mary Douglas [Option ID = 45360]
3. Victor Turner [Option ID = 45359]
4. Edmund R. Leach [Option ID = 45361]
Correct Answer :-
Mary Douglas [Option ID = 45360]
?Swaddling Hypothesis? was used by the
[Question ID = 18832]
1. Evolutionist [Option ID = 45320]
2. Conflict Theorist [Option ID = 45321]
3. Cultural Ecologist [Option ID = 45319]
4. Cultural Personality Thinker [Option ID = 45322]
Correct Answer :-
Cultural Personality Thinker [Option ID = 45322]
Trapeze is [Question ID = 18806]
55)
56)
57)
58)
59)
60)
1. Neolithic tool [Option ID = 45215]
2. Core tool [Option ID = 45216]
3. Flake tool [Option ID = 45217]
4. Microlithic tool [Option ID = 45218]
Correct Answer :-
Microlithic tool [Option ID = 45218]
?Melanin? pigment is produced in our [Question ID = 18799]
1. Blood [Option ID = 45188]
2. Pancreas [Option ID = 45190]
3. Liver [Option ID = 45189]
4. Skin [Option ID = 45187]
Correct Answer :-
Skin [Option ID = 45187]
What is the earliest date of Indian Lower Paleolithic culture?
[Question ID = 18825]
1. 2.4 mya [Option ID = 45294]
2. 1.8 mya [Option ID = 45292]
3. 1.51 mya [Option ID = 45293]
4. 1.71 mya [Option ID = 45291]
Correct Answer :-
1.51 mya [Option ID = 45293]
Mutation Theory was proposed by:
[Question ID = 18865]
1. Hugo de Vries [Option ID = 45451]
2. Gregor Mendel. [Option ID = 45454]
3. August Weismann [Option ID = 45453]
4. R. A. Fischer [Option ID = 45452]
Correct Answer :-
Hugo de Vries [Option ID = 45451]
Gonadotropins are responsible for the growth of the-
[Question ID = 18839]
1. Whole body [Option ID = 45350]
2. Brain [Option ID = 45347]
3. Ovaries and testis [Option ID = 45348]
4. Immune system [Option ID = 45349]
Correct Answer :-
Ovaries and testis [Option ID = 45348]
Attempts to account for facts by means of general hypothesis or research questions is known as: [Question ID = 18794]
1. Phenomenology [Option ID = 45169]
2. Reduction [Option ID = 45170]
3. Induction [Option ID = 45167]
4. Deduction [Option ID = 45168]
Correct Answer :-
Deduction [Option ID = 45168]
Someone experiencing physiological problems as a result of oxygen deprivation is
[Question ID = 18818]
FirstRanker.com - FirstRanker's Choice
1)
2)
3)
4)
5)
DU MSc Anthropology
Topic:- DU_J18_MSC_ANTHRO
Increased labour investment in agriculture with no increase in per capita productivity is called as
[Question ID = 18814]
1. Agricultural recession [Option ID = 45250]
2. Agricultural stagnation [Option ID = 45247]
3. Agricultural relegation [Option ID = 45248]
4. Agricultural involution [Option ID = 45249]
Correct Answer :-
Agricultural involution [Option ID = 45249]
Which of the following is a hominid?
[Question ID = 18857]
1. Chimpanzee [Option ID = 45421]
2. Human [Option ID = 45422]
3. Macaque [Option ID = 45419]
4. Baboon [Option ID = 45420]
Correct Answer :-
Human [Option ID = 45422]
Which rule says that the best suited human body for a hot, tropical region would be one with long limbs and short trunk?
[Question ID = 18827]
1. Allen?s rule [Option ID = 45299]
2. Thompson?s rule [Option ID = 45302]
3. Gloger?s rule [Option ID = 45301]
4. Bergman?s rule [Option ID = 45300]
Correct Answer :-
Allen?s rule [Option ID = 45299]
Which Australopithecus fossil was nicknamed as Lucy?
[Question ID = 18867]
1. Australopithecus africanus [Option ID = 45459]
2. Australopithecus afarensis [Option ID = 45462]
3. Australopithecus anamensis [Option ID = 45460]
4. Australopithecus robustus [Option ID = 45461]
Correct Answer :-
Australopithecus afarensis [Option ID = 45462]
Which among the following does not constitute the elementary form of exchange?
[Question ID = 18820]
1. Exchange of goods [Option ID = 45274]
2. Exchange of women [Option ID = 45271]
3. Exchange of money [Option ID = 45272]
4. Exchange of messages [Option ID = 45273]
Correct Answer :-
Exchange of goods [Option ID = 45274]
6)
7)
8)
9)
10)
11)
12)
Which prehistoric site is known as Mode I Industry?
[Question ID = 18835]
1. Pirro Nord [Option ID = 45333]
2. Hunsgi [Option ID = 45331]
3. Tautavel [Option ID = 45332]
4. Somme valley [Option ID = 45334]
Correct Answer :-
Pirro Nord [Option ID = 45333]
Any quantitative increase in body size is known as [Question ID = 18788]
1. Development [Option ID = 45144]
2. Maturation [Option ID = 45146]
3. Differentiation [Option ID = 45145]
4. Growth [Option ID = 45143]
Correct Answer :-
Growth [Option ID = 45143]
The highest point on the head is known as [Question ID = 18807]
1. Opisthocranion [Option ID = 45222]
2. Bregma [Option ID = 45220]
3. Vertex [Option ID = 45221]
4. Lambda [Option ID = 45219]
Correct Answer :-
Vertex [Option ID = 45221]
Autosomes are
[Question ID = 18826]
1. Normal sex chromosomes [Option ID = 45296]
2. Abnormal sex chromosomes [Option ID = 45298]
3. Abnormal chromosomes [Option ID = 45297]
4. All chromosomes other than the sex chromosomes [Option ID = 45295]
Correct Answer :-
All chromosomes other than the sex chromosomes [Option ID = 45295]
?Puberty spurt? occurs in which one of the following stages of human growth [Question ID = 18798]
1. Infancy [Option ID = 45183]
2. Childhood [Option ID = 45185]
3. Adolescence [Option ID = 45186]
4. Adulthood [Option ID = 45184]
Correct Answer :-
Adolescence [Option ID = 45186]
Potlatch is associated with [Question ID = 18802]
1. Iroquois [Option ID = 45201]
2. Kwakiutl [Option ID = 45199]
3. Shoshonean [Option ID = 45202]
4. Arapesh [Option ID = 45200]
Correct Answer :-
Kwakiutl [Option ID = 45199]
Oldowan culture is dominated by
[Question ID = 18824]
1. Blade tools [Option ID = 45288]
2. Flake tools [Option ID = 45289]
13)
14)
15)
16)
17)
18)
3. Pebble tools [Option ID = 45287]
4. Microlithic tools [Option ID = 45290]
Correct Answer :-
Pebble tools [Option ID = 45287]
A higher fortified part of a city is called
[Question ID = 18823]
1. metropolis [Option ID = 45284]
2. megapolis [Option ID = 45285]
3. acropolis [Option ID = 45283]
4. necropolis [Option ID = 45286]
Correct Answer :-
acropolis [Option ID = 45283]
Yale Cambridge expedition to the Sohan Valley was led by
[Question ID = 18795]
1. R. B. Foote and V.D. Krishnaswamy [Option ID = 45171]
2. G. R. Sharma and V.D. Mishra [Option ID = 45174]
3. H. D. Sankalia and B. Subba Rao [Option ID = 45173]
4. H. De. Terra and T. T. Paterson [Option ID = 45172]
Correct Answer :-
H. De. Terra and T. T. Paterson [Option ID = 45172]
Two nuclear families in adjacent generations with one son/husband or daughter/wife who is a member of both the families is called a
[Question ID = 18870]
1. lineal family [Option ID = 45471]
2. supplemented nuclear family [Option ID = 45474]
3. stem family [Option ID = 45473]
4. compound family [Option ID = 45472]
Correct Answer :-
stem family [Option ID = 45473]
The lack, excess or imbalance of nutrients in the diet leading to ill health is known as [Question ID = 18790]
1. Malnutrition [Option ID = 45153]
2. Over nutrition [Option ID = 45151]
3. Under nutrition [Option ID = 45152]
4. Marasmus [Option ID = 45154]
Correct Answer :-
Malnutrition [Option ID = 45153]
Holocene starts around
[Question ID = 18834]
1. 20,000 B.P. [Option ID = 45329]
2. 12,000 B.P. [Option ID = 45328]
3. 14,000 B.P. [Option ID = 45330]
4. 10,000 B.P. [Option ID = 45327]
Correct Answer :-
10,000 B.P. [Option ID = 45327]
According to Levi-Strauss which of the following constitute one of the ?atoms of kinship? relationship?
[Question ID = 18879]
19)
20)
21)
22)
23)
24)
1. mother-son [Option ID = 45507]
2. mother-daughter [Option ID = 45508]
3. father-son [Option ID = 45509]
4. father-daughter [Option ID = 45510]
Correct Answer :-
mother-son [Option ID = 45507]
A karyotype is a:
[Question ID = 18819]
1. Picture of an individual's chromosomes arranged in a standardized way [Option ID = 45269]
2. Type of abnormal chromosome that is associated with Down's syndrome [Option ID = 45268]
3. General term for any type of chromosome [Option ID = 45267]
4. General term for sex chromosome [Option ID = 45270]
Correct Answer :-
Picture of an individual's chromosomes arranged in a standardized way [Option ID = 45269]
Theodicy is an
[Question ID = 18882]
1. attempt to explain theocracy [Option ID = 45521]
2. attempt to explain good [Option ID = 45519]
3. attempt to explain evil [Option ID = 45520]
4. attempt to explain lie and deceit [Option ID = 45522]
Correct Answer :-
attempt to explain evil [Option ID = 45520]
The word ?Somatotype? was coined by [Question ID = 18797]
1. Faulkner [Option ID = 45182]
2. Sheldon [Option ID = 45180]
3. Parnell [Option ID = 45181]
4. Heath and Carter [Option ID = 45179]
Correct Answer :-
Heath and Carter [Option ID = 45179]
Substantivism as an approach in economic anthropology was given by
[Question ID = 18853]
1. A. L. Kroeber [Option ID = 45406]
2. Karl Polanyi [Option ID = 45405]
3. Oscar Lewis [Option ID = 45403]
4. Robert Redfield [Option ID = 62620]
Correct Answer :-
Karl Polanyi [Option ID = 45405]
?Genetics and the Origin of Species? was written by :
[Question ID = 18859]
1. Thomas Malthus. [Option ID = 45430]
2. Theodosius Dobzhansky [Option ID = 45428]
3. Charles Darwin [Option ID = 45427]
4. T. H. Morgan [Option ID = 45429]
Correct Answer :-
Theodosius Dobzhansky [Option ID = 45428]
The ability of the living organisms to survive in a particular ecological set up is called [Question ID = 18809]
25)
26)
27)
28)
29)
30)
1. Acclimation [Option ID = 45228]
2. Acclimatisation [Option ID = 45229]
3. Adaptation [Option ID = 45227]
4. Selection [Option ID = 45230]
Correct Answer :-
Adaptation [Option ID = 45227]
___________is the physiological ability to reproduce an offspring.
[Question ID = 18876]
1. Fecundity [Option ID = 45496]
2. Fertility [Option ID = 45495]
3. Virility [Option ID = 45498]
4. Natality [Option ID = 45497]
Correct Answer :-
Fecundity [Option ID = 45496]
Charles Darwin's ideas concerning the causes of evolution were probably formulated in his mind
[Question ID = 18817]
1. During the late 1880's [Option ID = 45262]
2. During his voyage on H.M.S. Beagle, especially after he reached the Gal?pagos Islands [Option ID = 45261]
3. Before he began his voyage of exploration around the world on H.M.S. Beagle [Option ID = 45260]
4. While he was still a student at Cambridge University [Option ID = 45259]
Correct Answer :-
During his voyage on H.M.S. Beagle, especially after he reached the Gal?pagos Islands [Option ID = 45261]
First Himalayan Glaciation is known as
[Question ID = 18884]
1. Terrace I [Option ID = 45530]
2. Pinjaur [Option ID = 45527]
3. Boulder conglomerate [Option ID = 45529]
4. Tatrot [Option ID = 45528]
Correct Answer :-
Tatrot [Option ID = 45528]
The author of the book Hindus of the Himalayas is [Question ID = 18812]
1. S.C. Dube [Option ID = 45241]
2. D.N. Majumdar [Option ID = 45242]
3. B. K. Roy-Burman [Option ID = 45240]
4. G.D. Berreman [Option ID = 45239]
Correct Answer :-
G.D. Berreman [Option ID = 45239]
Who proposed the ?Three Age System? in pre-history?
[Question ID = 18883]
1. C. J. Thomsen [Option ID = 45525]
2. B. M. Fagan [Option ID = 45523]
3. H.D.Sankalia [Option ID = 45524]
4. F. Borde [Option ID = 45526]
Correct Answer :-
C. J. Thomsen [Option ID = 45525]
Who among the following worked on the complicity between colonial agencies and Anthropologists?
31)
32)
33)
34)
35)
36)
[Question ID = 18801]
1. Ibn Khaldun [Option ID = 45196]
2. Akbar Ahmed [Option ID = 45197]
3. Ralf Linto [Option ID = 45198]
4. Talal Asad [Option ID = 45195]
Correct Answer :-
Talal Asad [Option ID = 45195]
Who is the author of the book ?The Old Stone Age??
[Question ID = 18854]
1. M.C. Burkitt [Option ID = 45407]
2. J. N. Pal [Option ID = 45408]
3. P.C. Pant [Option ID = 45409]
4. H. De Lumley [Option ID = 45410]
Correct Answer :-
M.C. Burkitt [Option ID = 45407]
Who included the Proto-Australoid racial element in the classification of Indian races?
[Question ID = 18866]
1. S.S. Sarkar [Option ID = 45456]
2. H.H. Risley [Option ID = 45455]
3. B.S. Guha [Option ID = 45458]
4. A.C. Haddon [Option ID = 45457]
Correct Answer :-
B.S. Guha [Option ID = 45458]
In which State is the site Isampur located?
[Question ID = 18815]
1. Karnataka [Option ID = 45253]
2. Uttar Pradesh [Option ID = 45254]
3. Madhya Pradesh [Option ID = 45252]
4. Bihar [Option ID = 45251]
Correct Answer :-
Karnataka [Option ID = 45253]
The view of the skull from above is [Question ID = 18800]
1. Norma Basalis [Option ID = 45193]
2. Norma Occipitalis [Option ID = 45194]
3. Norma Frontalis [Option ID = 45192]
4. Norma Verticalis [Option ID = 45191]
Correct Answer :-
Norma Verticalis [Option ID = 45191]
?Childhood? is peculiar to [Question ID = 18789]
1. Nonhuman primates [Option ID = 45149]
2. Human and nonhuman primates [Option ID = 45147]
3. Humans [Option ID = 45148]
4. Mammals [Option ID = 45150]
Correct Answer :-
Humans [Option ID = 45148]
Children subjected to starvation for short periods may recover completely on getting adequate diet. This is referred to as
37)
38)
39)
40)
41)
[Question ID = 18838]
1. Growth spurt [Option ID = 45346]
2. Catch down growth [Option ID = 45345]
3. Catch-up growth [Option ID = 45343]
4. Canalization [Option ID = 45344]
Correct Answer :-
Catch-up growth [Option ID = 45343]
Attirampakkam is [Question ID = 18805]
1. Lower Paleolithic site [Option ID = 45211]
2. Mesolithic site [Option ID = 45214]
3. Indus Valley Civilization site [Option ID = 45213]
4. Upper Paleolithic site [Option ID = 45212]
Correct Answer :-
Lower Paleolithic site [Option ID = 45211]
Shivering
[Question ID = 18828]
1. Has no effect on body heat [Option ID = 45306]
2. Indirectly produces more body heat [Option ID = 45304]
3. Speeds up the loss of body heat [Option ID = 45303]
4. Delays up the loss of body heat [Option ID = 45305]
Correct Answer :-
Indirectly produces more body heat [Option ID = 45304]
A deficiency disease of the skeletal system caused by a lack of Vitamin D or Calcium or both, and often resulting in bone deformities is
[Question ID = 18849]
1. Arthritis [Option ID = 45390]
2. Scurvy [Option ID = 45387]
3. Rickets [Option ID = 45389]
4. Osteoporosis [Option ID = 45388]
Correct Answer :-
Rickets [Option ID = 45389]
A deficiency disease caused due to severe lack of protein in diet among children is
[Question ID = 18856]
1. Anaemia [Option ID = 45418]
2. Kwashiorkor [Option ID = 45417]
3. Scurvy [Option ID = 45416]
4. Rickets [Option ID = 45415]
Correct Answer :-
Kwashiorkor [Option ID = 45417]
Samples of Dendrochronology is taken from which of the following?
[Question ID = 18873]
1. Lake [Option ID = 45483]
2. Tree [Option ID = 45486]
3. River [Option ID = 45484]
4. sea [Option ID = 45485]
Correct Answer :-
Tree [Option ID = 45486]
42)
43)
44)
45)
46)
47)
48)
Swiddening is a form of
[Question ID = 18822]
1. terrace cultivation [Option ID = 45281]
2. intensive cultivation [Option ID = 45280]
3. commercial cultivation [Option ID = 45282]
4. slash and burn cultivation [Option ID = 45279]
Correct Answer :-
slash and burn cultivation [Option ID = 45279]
Osteology is the study of [Question ID = 18810]
1. The human skeleton [Option ID = 45234]
2. The human brain [Option ID = 45233]
3. Human survival strategies [Option ID = 45231]
4. Human languages [Option ID = 45232]
Correct Answer :-
The human skeleton [Option ID = 45234]
Extended Case Study method in anthropology is associated with the work of [Question ID = 18813]
1. Paul Radin [Option ID = 45246]
2. Julian Steward [Option ID = 45244]
3. Anthony Walker [Option ID = 45245]
4. Max Gluckman [Option ID = 45243]
Correct Answer :-
Max Gluckman [Option ID = 45243]
Celts are prepared by
[Question ID = 18863]
1. Levalloisian technique [Option ID = 45443]
2. Clactonian technique [Option ID = 45444]
3. Pressure technique [Option ID = 45446]
4. Grinding and polishing technique [Option ID = 45445]
Correct Answer :-
Grinding and polishing technique [Option ID = 45445]
Khallu is a form : [Question ID = 18792]
1. Fishing [Option ID = 45160]
2. Hunting [Option ID = 45159]
3. Cropping [Option ID = 45161]
4. Weaving [Option ID = 45162]
Correct Answer :-
Cropping [Option ID = 45161]
Lascaux is
[Question ID = 18844]
1. Cave art site [Option ID = 45367]
2. Mesolithic site [Option ID = 45368]
3. Chalcolithic site [Option ID = 45369]
4. Neolithic site [Option ID = 45370]
Correct Answer :-
Cave art site [Option ID = 45367]
A social fact is not characterized by
[Question ID = 18862]
49)
50)
51)
52)
53)
54)
1. Inheritance [Option ID = 45441]
2. Independent existence [Option ID = 45439]
3. Constraints [Option ID = 45442]
4. Exteriority [Option ID = 45440]
Correct Answer :-
Inheritance [Option ID = 45441]
Chopper and Chopping tool are
[Question ID = 18864]
1. blade tool [Option ID = 45447]
2. flake tool [Option ID = 45449]
3. core tool [Option ID = 45448]
4. Neolithic tools [Option ID = 45450]
Correct Answer :-
core tool [Option ID = 45448]
The theory of evolution by natural selection was independently developed by: [Question ID = 19246]
1. Darwin and Lamarck [Option ID = 46977]
2. Darwin and Wallace [Option ID = 46976]
3. Lyell and Darwin [Option ID = 46975]
4. Lyell and Hutton [Option ID = 46978]
Correct Answer :-
Darwin and Lamarck [Option ID = 46977]
? Linea aspera? responsible for the erect posture in man is present in
[Question ID = 18846]
1. Anterior side of tibia [Option ID = 45376]
2. Anterior side of femur [Option ID = 45378]
3. Posterior side of femur [Option ID = 45375]
4. Medial side of fibula [Option ID = 45377]
Correct Answer :-
Posterior side of femur [Option ID = 45375]
?Purity and Danger? is associated with the work of
[Question ID = 18842]
1. M.N. Srinivas [Option ID = 45362]
2. Mary Douglas [Option ID = 45360]
3. Victor Turner [Option ID = 45359]
4. Edmund R. Leach [Option ID = 45361]
Correct Answer :-
Mary Douglas [Option ID = 45360]
?Swaddling Hypothesis? was used by the
[Question ID = 18832]
1. Evolutionist [Option ID = 45320]
2. Conflict Theorist [Option ID = 45321]
3. Cultural Ecologist [Option ID = 45319]
4. Cultural Personality Thinker [Option ID = 45322]
Correct Answer :-
Cultural Personality Thinker [Option ID = 45322]
Trapeze is [Question ID = 18806]
55)
56)
57)
58)
59)
60)
1. Neolithic tool [Option ID = 45215]
2. Core tool [Option ID = 45216]
3. Flake tool [Option ID = 45217]
4. Microlithic tool [Option ID = 45218]
Correct Answer :-
Microlithic tool [Option ID = 45218]
?Melanin? pigment is produced in our [Question ID = 18799]
1. Blood [Option ID = 45188]
2. Pancreas [Option ID = 45190]
3. Liver [Option ID = 45189]
4. Skin [Option ID = 45187]
Correct Answer :-
Skin [Option ID = 45187]
What is the earliest date of Indian Lower Paleolithic culture?
[Question ID = 18825]
1. 2.4 mya [Option ID = 45294]
2. 1.8 mya [Option ID = 45292]
3. 1.51 mya [Option ID = 45293]
4. 1.71 mya [Option ID = 45291]
Correct Answer :-
1.51 mya [Option ID = 45293]
Mutation Theory was proposed by:
[Question ID = 18865]
1. Hugo de Vries [Option ID = 45451]
2. Gregor Mendel. [Option ID = 45454]
3. August Weismann [Option ID = 45453]
4. R. A. Fischer [Option ID = 45452]
Correct Answer :-
Hugo de Vries [Option ID = 45451]
Gonadotropins are responsible for the growth of the-
[Question ID = 18839]
1. Whole body [Option ID = 45350]
2. Brain [Option ID = 45347]
3. Ovaries and testis [Option ID = 45348]
4. Immune system [Option ID = 45349]
Correct Answer :-
Ovaries and testis [Option ID = 45348]
Attempts to account for facts by means of general hypothesis or research questions is known as: [Question ID = 18794]
1. Phenomenology [Option ID = 45169]
2. Reduction [Option ID = 45170]
3. Induction [Option ID = 45167]
4. Deduction [Option ID = 45168]
Correct Answer :-
Deduction [Option ID = 45168]
Someone experiencing physiological problems as a result of oxygen deprivation is
[Question ID = 18818]
61)
62)
63)
64)
65)
1. Polycythaemia [Option ID = 45263]
2. Hyperoxia [Option ID = 45266]
3. Barometric excess [Option ID = 45264]
4. Hypoxia [Option ID = 45265]
Correct Answer :-
Hypoxia [Option ID = 45265]
Who gave the concept of ?the survival of the fittest?? [Question ID = 18808]
1. Hugo De Vries [Option ID = 45225]
2. Charles Darwin [Option ID = 45223]
3. Carl Correns [Option ID = 45226]
4. Herbert Spencer [Option ID = 45224]
Correct Answer :-
Herbert Spencer [Option ID = 45224]
Prepared core technique is also known as
[Question ID = 18845]
1. Punching technique [Option ID = 45374]
2. Levalloisian technique [Option ID = 45373]
3. Clactonian technique [Option ID = 45371]
4. Cylinder Hammer technique [Option ID = 45372]
Correct Answer :-
Levalloisian technique [Option ID = 45373]
A syndrome characterised by physiological, behavioural and mental defects that are due to the presence of an extra copy of the
chromosome 21 is
[Question ID = 18848]
1. Klinefelter syndrome [Option ID = 45385]
2. Marfan syndrome [Option ID = 45386]
3. Turner syndrome [Option ID = 45383]
4. Down syndrome [Option ID = 45384]
Correct Answer :-
Down syndrome [Option ID = 45384]
The statement "Animals are good to think with" is from
[Question ID = 18804]
1. Sherry B. Ortner [Option ID = 45209]
2. Margaret Mead [Option ID = 45210]
3. Claude L?vi Strauss [Option ID = 45208]
4. Mellisa Liewyn-Davies [Option ID = 45207]
Correct Answer :-
Claude L?vi Strauss [Option ID = 45208]
The physiological process of being or becoming accustomed to a new physical environment is known as:
[Question ID = 18877]
1. Alteration [Option ID = 45502]
2. Adaptation [Option ID = 45501]
3. Acclimatization [Option ID = 45500]
4. Assimilation [Option ID = 45499]
Correct Answer :-
Acclimatization [Option ID = 45500]
FirstRanker.com - FirstRanker's Choice
1)
2)
3)
4)
5)
DU MSc Anthropology
Topic:- DU_J18_MSC_ANTHRO
Increased labour investment in agriculture with no increase in per capita productivity is called as
[Question ID = 18814]
1. Agricultural recession [Option ID = 45250]
2. Agricultural stagnation [Option ID = 45247]
3. Agricultural relegation [Option ID = 45248]
4. Agricultural involution [Option ID = 45249]
Correct Answer :-
Agricultural involution [Option ID = 45249]
Which of the following is a hominid?
[Question ID = 18857]
1. Chimpanzee [Option ID = 45421]
2. Human [Option ID = 45422]
3. Macaque [Option ID = 45419]
4. Baboon [Option ID = 45420]
Correct Answer :-
Human [Option ID = 45422]
Which rule says that the best suited human body for a hot, tropical region would be one with long limbs and short trunk?
[Question ID = 18827]
1. Allen?s rule [Option ID = 45299]
2. Thompson?s rule [Option ID = 45302]
3. Gloger?s rule [Option ID = 45301]
4. Bergman?s rule [Option ID = 45300]
Correct Answer :-
Allen?s rule [Option ID = 45299]
Which Australopithecus fossil was nicknamed as Lucy?
[Question ID = 18867]
1. Australopithecus africanus [Option ID = 45459]
2. Australopithecus afarensis [Option ID = 45462]
3. Australopithecus anamensis [Option ID = 45460]
4. Australopithecus robustus [Option ID = 45461]
Correct Answer :-
Australopithecus afarensis [Option ID = 45462]
Which among the following does not constitute the elementary form of exchange?
[Question ID = 18820]
1. Exchange of goods [Option ID = 45274]
2. Exchange of women [Option ID = 45271]
3. Exchange of money [Option ID = 45272]
4. Exchange of messages [Option ID = 45273]
Correct Answer :-
Exchange of goods [Option ID = 45274]
6)
7)
8)
9)
10)
11)
12)
Which prehistoric site is known as Mode I Industry?
[Question ID = 18835]
1. Pirro Nord [Option ID = 45333]
2. Hunsgi [Option ID = 45331]
3. Tautavel [Option ID = 45332]
4. Somme valley [Option ID = 45334]
Correct Answer :-
Pirro Nord [Option ID = 45333]
Any quantitative increase in body size is known as [Question ID = 18788]
1. Development [Option ID = 45144]
2. Maturation [Option ID = 45146]
3. Differentiation [Option ID = 45145]
4. Growth [Option ID = 45143]
Correct Answer :-
Growth [Option ID = 45143]
The highest point on the head is known as [Question ID = 18807]
1. Opisthocranion [Option ID = 45222]
2. Bregma [Option ID = 45220]
3. Vertex [Option ID = 45221]
4. Lambda [Option ID = 45219]
Correct Answer :-
Vertex [Option ID = 45221]
Autosomes are
[Question ID = 18826]
1. Normal sex chromosomes [Option ID = 45296]
2. Abnormal sex chromosomes [Option ID = 45298]
3. Abnormal chromosomes [Option ID = 45297]
4. All chromosomes other than the sex chromosomes [Option ID = 45295]
Correct Answer :-
All chromosomes other than the sex chromosomes [Option ID = 45295]
?Puberty spurt? occurs in which one of the following stages of human growth [Question ID = 18798]
1. Infancy [Option ID = 45183]
2. Childhood [Option ID = 45185]
3. Adolescence [Option ID = 45186]
4. Adulthood [Option ID = 45184]
Correct Answer :-
Adolescence [Option ID = 45186]
Potlatch is associated with [Question ID = 18802]
1. Iroquois [Option ID = 45201]
2. Kwakiutl [Option ID = 45199]
3. Shoshonean [Option ID = 45202]
4. Arapesh [Option ID = 45200]
Correct Answer :-
Kwakiutl [Option ID = 45199]
Oldowan culture is dominated by
[Question ID = 18824]
1. Blade tools [Option ID = 45288]
2. Flake tools [Option ID = 45289]
13)
14)
15)
16)
17)
18)
3. Pebble tools [Option ID = 45287]
4. Microlithic tools [Option ID = 45290]
Correct Answer :-
Pebble tools [Option ID = 45287]
A higher fortified part of a city is called
[Question ID = 18823]
1. metropolis [Option ID = 45284]
2. megapolis [Option ID = 45285]
3. acropolis [Option ID = 45283]
4. necropolis [Option ID = 45286]
Correct Answer :-
acropolis [Option ID = 45283]
Yale Cambridge expedition to the Sohan Valley was led by
[Question ID = 18795]
1. R. B. Foote and V.D. Krishnaswamy [Option ID = 45171]
2. G. R. Sharma and V.D. Mishra [Option ID = 45174]
3. H. D. Sankalia and B. Subba Rao [Option ID = 45173]
4. H. De. Terra and T. T. Paterson [Option ID = 45172]
Correct Answer :-
H. De. Terra and T. T. Paterson [Option ID = 45172]
Two nuclear families in adjacent generations with one son/husband or daughter/wife who is a member of both the families is called a
[Question ID = 18870]
1. lineal family [Option ID = 45471]
2. supplemented nuclear family [Option ID = 45474]
3. stem family [Option ID = 45473]
4. compound family [Option ID = 45472]
Correct Answer :-
stem family [Option ID = 45473]
The lack, excess or imbalance of nutrients in the diet leading to ill health is known as [Question ID = 18790]
1. Malnutrition [Option ID = 45153]
2. Over nutrition [Option ID = 45151]
3. Under nutrition [Option ID = 45152]
4. Marasmus [Option ID = 45154]
Correct Answer :-
Malnutrition [Option ID = 45153]
Holocene starts around
[Question ID = 18834]
1. 20,000 B.P. [Option ID = 45329]
2. 12,000 B.P. [Option ID = 45328]
3. 14,000 B.P. [Option ID = 45330]
4. 10,000 B.P. [Option ID = 45327]
Correct Answer :-
10,000 B.P. [Option ID = 45327]
According to Levi-Strauss which of the following constitute one of the ?atoms of kinship? relationship?
[Question ID = 18879]
19)
20)
21)
22)
23)
24)
1. mother-son [Option ID = 45507]
2. mother-daughter [Option ID = 45508]
3. father-son [Option ID = 45509]
4. father-daughter [Option ID = 45510]
Correct Answer :-
mother-son [Option ID = 45507]
A karyotype is a:
[Question ID = 18819]
1. Picture of an individual's chromosomes arranged in a standardized way [Option ID = 45269]
2. Type of abnormal chromosome that is associated with Down's syndrome [Option ID = 45268]
3. General term for any type of chromosome [Option ID = 45267]
4. General term for sex chromosome [Option ID = 45270]
Correct Answer :-
Picture of an individual's chromosomes arranged in a standardized way [Option ID = 45269]
Theodicy is an
[Question ID = 18882]
1. attempt to explain theocracy [Option ID = 45521]
2. attempt to explain good [Option ID = 45519]
3. attempt to explain evil [Option ID = 45520]
4. attempt to explain lie and deceit [Option ID = 45522]
Correct Answer :-
attempt to explain evil [Option ID = 45520]
The word ?Somatotype? was coined by [Question ID = 18797]
1. Faulkner [Option ID = 45182]
2. Sheldon [Option ID = 45180]
3. Parnell [Option ID = 45181]
4. Heath and Carter [Option ID = 45179]
Correct Answer :-
Heath and Carter [Option ID = 45179]
Substantivism as an approach in economic anthropology was given by
[Question ID = 18853]
1. A. L. Kroeber [Option ID = 45406]
2. Karl Polanyi [Option ID = 45405]
3. Oscar Lewis [Option ID = 45403]
4. Robert Redfield [Option ID = 62620]
Correct Answer :-
Karl Polanyi [Option ID = 45405]
?Genetics and the Origin of Species? was written by :
[Question ID = 18859]
1. Thomas Malthus. [Option ID = 45430]
2. Theodosius Dobzhansky [Option ID = 45428]
3. Charles Darwin [Option ID = 45427]
4. T. H. Morgan [Option ID = 45429]
Correct Answer :-
Theodosius Dobzhansky [Option ID = 45428]
The ability of the living organisms to survive in a particular ecological set up is called [Question ID = 18809]
25)
26)
27)
28)
29)
30)
1. Acclimation [Option ID = 45228]
2. Acclimatisation [Option ID = 45229]
3. Adaptation [Option ID = 45227]
4. Selection [Option ID = 45230]
Correct Answer :-
Adaptation [Option ID = 45227]
___________is the physiological ability to reproduce an offspring.
[Question ID = 18876]
1. Fecundity [Option ID = 45496]
2. Fertility [Option ID = 45495]
3. Virility [Option ID = 45498]
4. Natality [Option ID = 45497]
Correct Answer :-
Fecundity [Option ID = 45496]
Charles Darwin's ideas concerning the causes of evolution were probably formulated in his mind
[Question ID = 18817]
1. During the late 1880's [Option ID = 45262]
2. During his voyage on H.M.S. Beagle, especially after he reached the Gal?pagos Islands [Option ID = 45261]
3. Before he began his voyage of exploration around the world on H.M.S. Beagle [Option ID = 45260]
4. While he was still a student at Cambridge University [Option ID = 45259]
Correct Answer :-
During his voyage on H.M.S. Beagle, especially after he reached the Gal?pagos Islands [Option ID = 45261]
First Himalayan Glaciation is known as
[Question ID = 18884]
1. Terrace I [Option ID = 45530]
2. Pinjaur [Option ID = 45527]
3. Boulder conglomerate [Option ID = 45529]
4. Tatrot [Option ID = 45528]
Correct Answer :-
Tatrot [Option ID = 45528]
The author of the book Hindus of the Himalayas is [Question ID = 18812]
1. S.C. Dube [Option ID = 45241]
2. D.N. Majumdar [Option ID = 45242]
3. B. K. Roy-Burman [Option ID = 45240]
4. G.D. Berreman [Option ID = 45239]
Correct Answer :-
G.D. Berreman [Option ID = 45239]
Who proposed the ?Three Age System? in pre-history?
[Question ID = 18883]
1. C. J. Thomsen [Option ID = 45525]
2. B. M. Fagan [Option ID = 45523]
3. H.D.Sankalia [Option ID = 45524]
4. F. Borde [Option ID = 45526]
Correct Answer :-
C. J. Thomsen [Option ID = 45525]
Who among the following worked on the complicity between colonial agencies and Anthropologists?
31)
32)
33)
34)
35)
36)
[Question ID = 18801]
1. Ibn Khaldun [Option ID = 45196]
2. Akbar Ahmed [Option ID = 45197]
3. Ralf Linto [Option ID = 45198]
4. Talal Asad [Option ID = 45195]
Correct Answer :-
Talal Asad [Option ID = 45195]
Who is the author of the book ?The Old Stone Age??
[Question ID = 18854]
1. M.C. Burkitt [Option ID = 45407]
2. J. N. Pal [Option ID = 45408]
3. P.C. Pant [Option ID = 45409]
4. H. De Lumley [Option ID = 45410]
Correct Answer :-
M.C. Burkitt [Option ID = 45407]
Who included the Proto-Australoid racial element in the classification of Indian races?
[Question ID = 18866]
1. S.S. Sarkar [Option ID = 45456]
2. H.H. Risley [Option ID = 45455]
3. B.S. Guha [Option ID = 45458]
4. A.C. Haddon [Option ID = 45457]
Correct Answer :-
B.S. Guha [Option ID = 45458]
In which State is the site Isampur located?
[Question ID = 18815]
1. Karnataka [Option ID = 45253]
2. Uttar Pradesh [Option ID = 45254]
3. Madhya Pradesh [Option ID = 45252]
4. Bihar [Option ID = 45251]
Correct Answer :-
Karnataka [Option ID = 45253]
The view of the skull from above is [Question ID = 18800]
1. Norma Basalis [Option ID = 45193]
2. Norma Occipitalis [Option ID = 45194]
3. Norma Frontalis [Option ID = 45192]
4. Norma Verticalis [Option ID = 45191]
Correct Answer :-
Norma Verticalis [Option ID = 45191]
?Childhood? is peculiar to [Question ID = 18789]
1. Nonhuman primates [Option ID = 45149]
2. Human and nonhuman primates [Option ID = 45147]
3. Humans [Option ID = 45148]
4. Mammals [Option ID = 45150]
Correct Answer :-
Humans [Option ID = 45148]
Children subjected to starvation for short periods may recover completely on getting adequate diet. This is referred to as
37)
38)
39)
40)
41)
[Question ID = 18838]
1. Growth spurt [Option ID = 45346]
2. Catch down growth [Option ID = 45345]
3. Catch-up growth [Option ID = 45343]
4. Canalization [Option ID = 45344]
Correct Answer :-
Catch-up growth [Option ID = 45343]
Attirampakkam is [Question ID = 18805]
1. Lower Paleolithic site [Option ID = 45211]
2. Mesolithic site [Option ID = 45214]
3. Indus Valley Civilization site [Option ID = 45213]
4. Upper Paleolithic site [Option ID = 45212]
Correct Answer :-
Lower Paleolithic site [Option ID = 45211]
Shivering
[Question ID = 18828]
1. Has no effect on body heat [Option ID = 45306]
2. Indirectly produces more body heat [Option ID = 45304]
3. Speeds up the loss of body heat [Option ID = 45303]
4. Delays up the loss of body heat [Option ID = 45305]
Correct Answer :-
Indirectly produces more body heat [Option ID = 45304]
A deficiency disease of the skeletal system caused by a lack of Vitamin D or Calcium or both, and often resulting in bone deformities is
[Question ID = 18849]
1. Arthritis [Option ID = 45390]
2. Scurvy [Option ID = 45387]
3. Rickets [Option ID = 45389]
4. Osteoporosis [Option ID = 45388]
Correct Answer :-
Rickets [Option ID = 45389]
A deficiency disease caused due to severe lack of protein in diet among children is
[Question ID = 18856]
1. Anaemia [Option ID = 45418]
2. Kwashiorkor [Option ID = 45417]
3. Scurvy [Option ID = 45416]
4. Rickets [Option ID = 45415]
Correct Answer :-
Kwashiorkor [Option ID = 45417]
Samples of Dendrochronology is taken from which of the following?
[Question ID = 18873]
1. Lake [Option ID = 45483]
2. Tree [Option ID = 45486]
3. River [Option ID = 45484]
4. sea [Option ID = 45485]
Correct Answer :-
Tree [Option ID = 45486]
42)
43)
44)
45)
46)
47)
48)
Swiddening is a form of
[Question ID = 18822]
1. terrace cultivation [Option ID = 45281]
2. intensive cultivation [Option ID = 45280]
3. commercial cultivation [Option ID = 45282]
4. slash and burn cultivation [Option ID = 45279]
Correct Answer :-
slash and burn cultivation [Option ID = 45279]
Osteology is the study of [Question ID = 18810]
1. The human skeleton [Option ID = 45234]
2. The human brain [Option ID = 45233]
3. Human survival strategies [Option ID = 45231]
4. Human languages [Option ID = 45232]
Correct Answer :-
The human skeleton [Option ID = 45234]
Extended Case Study method in anthropology is associated with the work of [Question ID = 18813]
1. Paul Radin [Option ID = 45246]
2. Julian Steward [Option ID = 45244]
3. Anthony Walker [Option ID = 45245]
4. Max Gluckman [Option ID = 45243]
Correct Answer :-
Max Gluckman [Option ID = 45243]
Celts are prepared by
[Question ID = 18863]
1. Levalloisian technique [Option ID = 45443]
2. Clactonian technique [Option ID = 45444]
3. Pressure technique [Option ID = 45446]
4. Grinding and polishing technique [Option ID = 45445]
Correct Answer :-
Grinding and polishing technique [Option ID = 45445]
Khallu is a form : [Question ID = 18792]
1. Fishing [Option ID = 45160]
2. Hunting [Option ID = 45159]
3. Cropping [Option ID = 45161]
4. Weaving [Option ID = 45162]
Correct Answer :-
Cropping [Option ID = 45161]
Lascaux is
[Question ID = 18844]
1. Cave art site [Option ID = 45367]
2. Mesolithic site [Option ID = 45368]
3. Chalcolithic site [Option ID = 45369]
4. Neolithic site [Option ID = 45370]
Correct Answer :-
Cave art site [Option ID = 45367]
A social fact is not characterized by
[Question ID = 18862]
49)
50)
51)
52)
53)
54)
1. Inheritance [Option ID = 45441]
2. Independent existence [Option ID = 45439]
3. Constraints [Option ID = 45442]
4. Exteriority [Option ID = 45440]
Correct Answer :-
Inheritance [Option ID = 45441]
Chopper and Chopping tool are
[Question ID = 18864]
1. blade tool [Option ID = 45447]
2. flake tool [Option ID = 45449]
3. core tool [Option ID = 45448]
4. Neolithic tools [Option ID = 45450]
Correct Answer :-
core tool [Option ID = 45448]
The theory of evolution by natural selection was independently developed by: [Question ID = 19246]
1. Darwin and Lamarck [Option ID = 46977]
2. Darwin and Wallace [Option ID = 46976]
3. Lyell and Darwin [Option ID = 46975]
4. Lyell and Hutton [Option ID = 46978]
Correct Answer :-
Darwin and Lamarck [Option ID = 46977]
? Linea aspera? responsible for the erect posture in man is present in
[Question ID = 18846]
1. Anterior side of tibia [Option ID = 45376]
2. Anterior side of femur [Option ID = 45378]
3. Posterior side of femur [Option ID = 45375]
4. Medial side of fibula [Option ID = 45377]
Correct Answer :-
Posterior side of femur [Option ID = 45375]
?Purity and Danger? is associated with the work of
[Question ID = 18842]
1. M.N. Srinivas [Option ID = 45362]
2. Mary Douglas [Option ID = 45360]
3. Victor Turner [Option ID = 45359]
4. Edmund R. Leach [Option ID = 45361]
Correct Answer :-
Mary Douglas [Option ID = 45360]
?Swaddling Hypothesis? was used by the
[Question ID = 18832]
1. Evolutionist [Option ID = 45320]
2. Conflict Theorist [Option ID = 45321]
3. Cultural Ecologist [Option ID = 45319]
4. Cultural Personality Thinker [Option ID = 45322]
Correct Answer :-
Cultural Personality Thinker [Option ID = 45322]
Trapeze is [Question ID = 18806]
55)
56)
57)
58)
59)
60)
1. Neolithic tool [Option ID = 45215]
2. Core tool [Option ID = 45216]
3. Flake tool [Option ID = 45217]
4. Microlithic tool [Option ID = 45218]
Correct Answer :-
Microlithic tool [Option ID = 45218]
?Melanin? pigment is produced in our [Question ID = 18799]
1. Blood [Option ID = 45188]
2. Pancreas [Option ID = 45190]
3. Liver [Option ID = 45189]
4. Skin [Option ID = 45187]
Correct Answer :-
Skin [Option ID = 45187]
What is the earliest date of Indian Lower Paleolithic culture?
[Question ID = 18825]
1. 2.4 mya [Option ID = 45294]
2. 1.8 mya [Option ID = 45292]
3. 1.51 mya [Option ID = 45293]
4. 1.71 mya [Option ID = 45291]
Correct Answer :-
1.51 mya [Option ID = 45293]
Mutation Theory was proposed by:
[Question ID = 18865]
1. Hugo de Vries [Option ID = 45451]
2. Gregor Mendel. [Option ID = 45454]
3. August Weismann [Option ID = 45453]
4. R. A. Fischer [Option ID = 45452]
Correct Answer :-
Hugo de Vries [Option ID = 45451]
Gonadotropins are responsible for the growth of the-
[Question ID = 18839]
1. Whole body [Option ID = 45350]
2. Brain [Option ID = 45347]
3. Ovaries and testis [Option ID = 45348]
4. Immune system [Option ID = 45349]
Correct Answer :-
Ovaries and testis [Option ID = 45348]
Attempts to account for facts by means of general hypothesis or research questions is known as: [Question ID = 18794]
1. Phenomenology [Option ID = 45169]
2. Reduction [Option ID = 45170]
3. Induction [Option ID = 45167]
4. Deduction [Option ID = 45168]
Correct Answer :-
Deduction [Option ID = 45168]
Someone experiencing physiological problems as a result of oxygen deprivation is
[Question ID = 18818]
61)
62)
63)
64)
65)
1. Polycythaemia [Option ID = 45263]
2. Hyperoxia [Option ID = 45266]
3. Barometric excess [Option ID = 45264]
4. Hypoxia [Option ID = 45265]
Correct Answer :-
Hypoxia [Option ID = 45265]
Who gave the concept of ?the survival of the fittest?? [Question ID = 18808]
1. Hugo De Vries [Option ID = 45225]
2. Charles Darwin [Option ID = 45223]
3. Carl Correns [Option ID = 45226]
4. Herbert Spencer [Option ID = 45224]
Correct Answer :-
Herbert Spencer [Option ID = 45224]
Prepared core technique is also known as
[Question ID = 18845]
1. Punching technique [Option ID = 45374]
2. Levalloisian technique [Option ID = 45373]
3. Clactonian technique [Option ID = 45371]
4. Cylinder Hammer technique [Option ID = 45372]
Correct Answer :-
Levalloisian technique [Option ID = 45373]
A syndrome characterised by physiological, behavioural and mental defects that are due to the presence of an extra copy of the
chromosome 21 is
[Question ID = 18848]
1. Klinefelter syndrome [Option ID = 45385]
2. Marfan syndrome [Option ID = 45386]
3. Turner syndrome [Option ID = 45383]
4. Down syndrome [Option ID = 45384]
Correct Answer :-
Down syndrome [Option ID = 45384]
The statement "Animals are good to think with" is from
[Question ID = 18804]
1. Sherry B. Ortner [Option ID = 45209]
2. Margaret Mead [Option ID = 45210]
3. Claude L?vi Strauss [Option ID = 45208]
4. Mellisa Liewyn-Davies [Option ID = 45207]
Correct Answer :-
Claude L?vi Strauss [Option ID = 45208]
The physiological process of being or becoming accustomed to a new physical environment is known as:
[Question ID = 18877]
1. Alteration [Option ID = 45502]
2. Adaptation [Option ID = 45501]
3. Acclimatization [Option ID = 45500]
4. Assimilation [Option ID = 45499]
Correct Answer :-
Acclimatization [Option ID = 45500]
66)
67)
68)
69)
70)
71)
The famous ?Fox Project? was guided by
[Question ID = 18821]
1. Marvin Harris. [Option ID = 45278]
2. Sol Tax [Option ID = 45275]
3. George P. Murdock [Option ID = 45276]
4. Margaret Mead [Option ID = 45277]
Correct Answer :-
Sol Tax [Option ID = 45275]
The first Director of Anthropological Survey of India was
[Question ID = 18851]
1. S.C. Roy [Option ID = 45396]
2. N. K. Bose. [Option ID = 45395]
3. B.S. Guha [Option ID = 45398]
4. G.S.Ghurye [Option ID = 45397]
Correct Answer :-
B.S. Guha [Option ID = 45398]
The Cross Cultural Cumulative Coding Center (The 5 C?s ) has been developed by
[Question ID = 18833]
1. Ruth Benedict [Option ID = 45323]
2. Franz Boas. [Option ID = 45324]
3. G. P. Murdock [Option ID = 45326]
4. Jack Goody [Option ID = 45325]
Correct Answer :-
G. P. Murdock [Option ID = 45326]
The advisory committee for the revision of the list of Scheduled Castes and Scheduled Tribal is also known as
[Question ID = 18861]
1. Ghurye committee [Option ID = 45438]
2. Shilu Ao committee [Option ID = 45437]
3. Elwin committee [Option ID = 45436]
4. Lokur committee [Option ID = 45435]
Correct Answer :-
Lokur committee [Option ID = 45435]
The period referring to the process of growth from birth up to 28 days of life is-
[Question ID = 18837]
1. Postnatal [Option ID = 45339]
2. Infancy [Option ID = 45340]
3. Neo-natal [Option ID = 45342]
4. Prenatal [Option ID = 45341]
Correct Answer :-
Neo-natal [Option ID = 45342]
The book ?Homo Hierarchicus? is authored by
[Question ID = 18791]
1. Louis Dumont [Option ID = 45155]
2. M.N.Srinivas [Option ID = 45156]
3. G.S. Ghurye [Option ID = 45158]
4. Gerald Berreman [Option ID = 45157]
FirstRanker.com - FirstRanker's Choice
1)
2)
3)
4)
5)
DU MSc Anthropology
Topic:- DU_J18_MSC_ANTHRO
Increased labour investment in agriculture with no increase in per capita productivity is called as
[Question ID = 18814]
1. Agricultural recession [Option ID = 45250]
2. Agricultural stagnation [Option ID = 45247]
3. Agricultural relegation [Option ID = 45248]
4. Agricultural involution [Option ID = 45249]
Correct Answer :-
Agricultural involution [Option ID = 45249]
Which of the following is a hominid?
[Question ID = 18857]
1. Chimpanzee [Option ID = 45421]
2. Human [Option ID = 45422]
3. Macaque [Option ID = 45419]
4. Baboon [Option ID = 45420]
Correct Answer :-
Human [Option ID = 45422]
Which rule says that the best suited human body for a hot, tropical region would be one with long limbs and short trunk?
[Question ID = 18827]
1. Allen?s rule [Option ID = 45299]
2. Thompson?s rule [Option ID = 45302]
3. Gloger?s rule [Option ID = 45301]
4. Bergman?s rule [Option ID = 45300]
Correct Answer :-
Allen?s rule [Option ID = 45299]
Which Australopithecus fossil was nicknamed as Lucy?
[Question ID = 18867]
1. Australopithecus africanus [Option ID = 45459]
2. Australopithecus afarensis [Option ID = 45462]
3. Australopithecus anamensis [Option ID = 45460]
4. Australopithecus robustus [Option ID = 45461]
Correct Answer :-
Australopithecus afarensis [Option ID = 45462]
Which among the following does not constitute the elementary form of exchange?
[Question ID = 18820]
1. Exchange of goods [Option ID = 45274]
2. Exchange of women [Option ID = 45271]
3. Exchange of money [Option ID = 45272]
4. Exchange of messages [Option ID = 45273]
Correct Answer :-
Exchange of goods [Option ID = 45274]
6)
7)
8)
9)
10)
11)
12)
Which prehistoric site is known as Mode I Industry?
[Question ID = 18835]
1. Pirro Nord [Option ID = 45333]
2. Hunsgi [Option ID = 45331]
3. Tautavel [Option ID = 45332]
4. Somme valley [Option ID = 45334]
Correct Answer :-
Pirro Nord [Option ID = 45333]
Any quantitative increase in body size is known as [Question ID = 18788]
1. Development [Option ID = 45144]
2. Maturation [Option ID = 45146]
3. Differentiation [Option ID = 45145]
4. Growth [Option ID = 45143]
Correct Answer :-
Growth [Option ID = 45143]
The highest point on the head is known as [Question ID = 18807]
1. Opisthocranion [Option ID = 45222]
2. Bregma [Option ID = 45220]
3. Vertex [Option ID = 45221]
4. Lambda [Option ID = 45219]
Correct Answer :-
Vertex [Option ID = 45221]
Autosomes are
[Question ID = 18826]
1. Normal sex chromosomes [Option ID = 45296]
2. Abnormal sex chromosomes [Option ID = 45298]
3. Abnormal chromosomes [Option ID = 45297]
4. All chromosomes other than the sex chromosomes [Option ID = 45295]
Correct Answer :-
All chromosomes other than the sex chromosomes [Option ID = 45295]
?Puberty spurt? occurs in which one of the following stages of human growth [Question ID = 18798]
1. Infancy [Option ID = 45183]
2. Childhood [Option ID = 45185]
3. Adolescence [Option ID = 45186]
4. Adulthood [Option ID = 45184]
Correct Answer :-
Adolescence [Option ID = 45186]
Potlatch is associated with [Question ID = 18802]
1. Iroquois [Option ID = 45201]
2. Kwakiutl [Option ID = 45199]
3. Shoshonean [Option ID = 45202]
4. Arapesh [Option ID = 45200]
Correct Answer :-
Kwakiutl [Option ID = 45199]
Oldowan culture is dominated by
[Question ID = 18824]
1. Blade tools [Option ID = 45288]
2. Flake tools [Option ID = 45289]
13)
14)
15)
16)
17)
18)
3. Pebble tools [Option ID = 45287]
4. Microlithic tools [Option ID = 45290]
Correct Answer :-
Pebble tools [Option ID = 45287]
A higher fortified part of a city is called
[Question ID = 18823]
1. metropolis [Option ID = 45284]
2. megapolis [Option ID = 45285]
3. acropolis [Option ID = 45283]
4. necropolis [Option ID = 45286]
Correct Answer :-
acropolis [Option ID = 45283]
Yale Cambridge expedition to the Sohan Valley was led by
[Question ID = 18795]
1. R. B. Foote and V.D. Krishnaswamy [Option ID = 45171]
2. G. R. Sharma and V.D. Mishra [Option ID = 45174]
3. H. D. Sankalia and B. Subba Rao [Option ID = 45173]
4. H. De. Terra and T. T. Paterson [Option ID = 45172]
Correct Answer :-
H. De. Terra and T. T. Paterson [Option ID = 45172]
Two nuclear families in adjacent generations with one son/husband or daughter/wife who is a member of both the families is called a
[Question ID = 18870]
1. lineal family [Option ID = 45471]
2. supplemented nuclear family [Option ID = 45474]
3. stem family [Option ID = 45473]
4. compound family [Option ID = 45472]
Correct Answer :-
stem family [Option ID = 45473]
The lack, excess or imbalance of nutrients in the diet leading to ill health is known as [Question ID = 18790]
1. Malnutrition [Option ID = 45153]
2. Over nutrition [Option ID = 45151]
3. Under nutrition [Option ID = 45152]
4. Marasmus [Option ID = 45154]
Correct Answer :-
Malnutrition [Option ID = 45153]
Holocene starts around
[Question ID = 18834]
1. 20,000 B.P. [Option ID = 45329]
2. 12,000 B.P. [Option ID = 45328]
3. 14,000 B.P. [Option ID = 45330]
4. 10,000 B.P. [Option ID = 45327]
Correct Answer :-
10,000 B.P. [Option ID = 45327]
According to Levi-Strauss which of the following constitute one of the ?atoms of kinship? relationship?
[Question ID = 18879]
19)
20)
21)
22)
23)
24)
1. mother-son [Option ID = 45507]
2. mother-daughter [Option ID = 45508]
3. father-son [Option ID = 45509]
4. father-daughter [Option ID = 45510]
Correct Answer :-
mother-son [Option ID = 45507]
A karyotype is a:
[Question ID = 18819]
1. Picture of an individual's chromosomes arranged in a standardized way [Option ID = 45269]
2. Type of abnormal chromosome that is associated with Down's syndrome [Option ID = 45268]
3. General term for any type of chromosome [Option ID = 45267]
4. General term for sex chromosome [Option ID = 45270]
Correct Answer :-
Picture of an individual's chromosomes arranged in a standardized way [Option ID = 45269]
Theodicy is an
[Question ID = 18882]
1. attempt to explain theocracy [Option ID = 45521]
2. attempt to explain good [Option ID = 45519]
3. attempt to explain evil [Option ID = 45520]
4. attempt to explain lie and deceit [Option ID = 45522]
Correct Answer :-
attempt to explain evil [Option ID = 45520]
The word ?Somatotype? was coined by [Question ID = 18797]
1. Faulkner [Option ID = 45182]
2. Sheldon [Option ID = 45180]
3. Parnell [Option ID = 45181]
4. Heath and Carter [Option ID = 45179]
Correct Answer :-
Heath and Carter [Option ID = 45179]
Substantivism as an approach in economic anthropology was given by
[Question ID = 18853]
1. A. L. Kroeber [Option ID = 45406]
2. Karl Polanyi [Option ID = 45405]
3. Oscar Lewis [Option ID = 45403]
4. Robert Redfield [Option ID = 62620]
Correct Answer :-
Karl Polanyi [Option ID = 45405]
?Genetics and the Origin of Species? was written by :
[Question ID = 18859]
1. Thomas Malthus. [Option ID = 45430]
2. Theodosius Dobzhansky [Option ID = 45428]
3. Charles Darwin [Option ID = 45427]
4. T. H. Morgan [Option ID = 45429]
Correct Answer :-
Theodosius Dobzhansky [Option ID = 45428]
The ability of the living organisms to survive in a particular ecological set up is called [Question ID = 18809]
25)
26)
27)
28)
29)
30)
1. Acclimation [Option ID = 45228]
2. Acclimatisation [Option ID = 45229]
3. Adaptation [Option ID = 45227]
4. Selection [Option ID = 45230]
Correct Answer :-
Adaptation [Option ID = 45227]
___________is the physiological ability to reproduce an offspring.
[Question ID = 18876]
1. Fecundity [Option ID = 45496]
2. Fertility [Option ID = 45495]
3. Virility [Option ID = 45498]
4. Natality [Option ID = 45497]
Correct Answer :-
Fecundity [Option ID = 45496]
Charles Darwin's ideas concerning the causes of evolution were probably formulated in his mind
[Question ID = 18817]
1. During the late 1880's [Option ID = 45262]
2. During his voyage on H.M.S. Beagle, especially after he reached the Gal?pagos Islands [Option ID = 45261]
3. Before he began his voyage of exploration around the world on H.M.S. Beagle [Option ID = 45260]
4. While he was still a student at Cambridge University [Option ID = 45259]
Correct Answer :-
During his voyage on H.M.S. Beagle, especially after he reached the Gal?pagos Islands [Option ID = 45261]
First Himalayan Glaciation is known as
[Question ID = 18884]
1. Terrace I [Option ID = 45530]
2. Pinjaur [Option ID = 45527]
3. Boulder conglomerate [Option ID = 45529]
4. Tatrot [Option ID = 45528]
Correct Answer :-
Tatrot [Option ID = 45528]
The author of the book Hindus of the Himalayas is [Question ID = 18812]
1. S.C. Dube [Option ID = 45241]
2. D.N. Majumdar [Option ID = 45242]
3. B. K. Roy-Burman [Option ID = 45240]
4. G.D. Berreman [Option ID = 45239]
Correct Answer :-
G.D. Berreman [Option ID = 45239]
Who proposed the ?Three Age System? in pre-history?
[Question ID = 18883]
1. C. J. Thomsen [Option ID = 45525]
2. B. M. Fagan [Option ID = 45523]
3. H.D.Sankalia [Option ID = 45524]
4. F. Borde [Option ID = 45526]
Correct Answer :-
C. J. Thomsen [Option ID = 45525]
Who among the following worked on the complicity between colonial agencies and Anthropologists?
31)
32)
33)
34)
35)
36)
[Question ID = 18801]
1. Ibn Khaldun [Option ID = 45196]
2. Akbar Ahmed [Option ID = 45197]
3. Ralf Linto [Option ID = 45198]
4. Talal Asad [Option ID = 45195]
Correct Answer :-
Talal Asad [Option ID = 45195]
Who is the author of the book ?The Old Stone Age??
[Question ID = 18854]
1. M.C. Burkitt [Option ID = 45407]
2. J. N. Pal [Option ID = 45408]
3. P.C. Pant [Option ID = 45409]
4. H. De Lumley [Option ID = 45410]
Correct Answer :-
M.C. Burkitt [Option ID = 45407]
Who included the Proto-Australoid racial element in the classification of Indian races?
[Question ID = 18866]
1. S.S. Sarkar [Option ID = 45456]
2. H.H. Risley [Option ID = 45455]
3. B.S. Guha [Option ID = 45458]
4. A.C. Haddon [Option ID = 45457]
Correct Answer :-
B.S. Guha [Option ID = 45458]
In which State is the site Isampur located?
[Question ID = 18815]
1. Karnataka [Option ID = 45253]
2. Uttar Pradesh [Option ID = 45254]
3. Madhya Pradesh [Option ID = 45252]
4. Bihar [Option ID = 45251]
Correct Answer :-
Karnataka [Option ID = 45253]
The view of the skull from above is [Question ID = 18800]
1. Norma Basalis [Option ID = 45193]
2. Norma Occipitalis [Option ID = 45194]
3. Norma Frontalis [Option ID = 45192]
4. Norma Verticalis [Option ID = 45191]
Correct Answer :-
Norma Verticalis [Option ID = 45191]
?Childhood? is peculiar to [Question ID = 18789]
1. Nonhuman primates [Option ID = 45149]
2. Human and nonhuman primates [Option ID = 45147]
3. Humans [Option ID = 45148]
4. Mammals [Option ID = 45150]
Correct Answer :-
Humans [Option ID = 45148]
Children subjected to starvation for short periods may recover completely on getting adequate diet. This is referred to as
37)
38)
39)
40)
41)
[Question ID = 18838]
1. Growth spurt [Option ID = 45346]
2. Catch down growth [Option ID = 45345]
3. Catch-up growth [Option ID = 45343]
4. Canalization [Option ID = 45344]
Correct Answer :-
Catch-up growth [Option ID = 45343]
Attirampakkam is [Question ID = 18805]
1. Lower Paleolithic site [Option ID = 45211]
2. Mesolithic site [Option ID = 45214]
3. Indus Valley Civilization site [Option ID = 45213]
4. Upper Paleolithic site [Option ID = 45212]
Correct Answer :-
Lower Paleolithic site [Option ID = 45211]
Shivering
[Question ID = 18828]
1. Has no effect on body heat [Option ID = 45306]
2. Indirectly produces more body heat [Option ID = 45304]
3. Speeds up the loss of body heat [Option ID = 45303]
4. Delays up the loss of body heat [Option ID = 45305]
Correct Answer :-
Indirectly produces more body heat [Option ID = 45304]
A deficiency disease of the skeletal system caused by a lack of Vitamin D or Calcium or both, and often resulting in bone deformities is
[Question ID = 18849]
1. Arthritis [Option ID = 45390]
2. Scurvy [Option ID = 45387]
3. Rickets [Option ID = 45389]
4. Osteoporosis [Option ID = 45388]
Correct Answer :-
Rickets [Option ID = 45389]
A deficiency disease caused due to severe lack of protein in diet among children is
[Question ID = 18856]
1. Anaemia [Option ID = 45418]
2. Kwashiorkor [Option ID = 45417]
3. Scurvy [Option ID = 45416]
4. Rickets [Option ID = 45415]
Correct Answer :-
Kwashiorkor [Option ID = 45417]
Samples of Dendrochronology is taken from which of the following?
[Question ID = 18873]
1. Lake [Option ID = 45483]
2. Tree [Option ID = 45486]
3. River [Option ID = 45484]
4. sea [Option ID = 45485]
Correct Answer :-
Tree [Option ID = 45486]
42)
43)
44)
45)
46)
47)
48)
Swiddening is a form of
[Question ID = 18822]
1. terrace cultivation [Option ID = 45281]
2. intensive cultivation [Option ID = 45280]
3. commercial cultivation [Option ID = 45282]
4. slash and burn cultivation [Option ID = 45279]
Correct Answer :-
slash and burn cultivation [Option ID = 45279]
Osteology is the study of [Question ID = 18810]
1. The human skeleton [Option ID = 45234]
2. The human brain [Option ID = 45233]
3. Human survival strategies [Option ID = 45231]
4. Human languages [Option ID = 45232]
Correct Answer :-
The human skeleton [Option ID = 45234]
Extended Case Study method in anthropology is associated with the work of [Question ID = 18813]
1. Paul Radin [Option ID = 45246]
2. Julian Steward [Option ID = 45244]
3. Anthony Walker [Option ID = 45245]
4. Max Gluckman [Option ID = 45243]
Correct Answer :-
Max Gluckman [Option ID = 45243]
Celts are prepared by
[Question ID = 18863]
1. Levalloisian technique [Option ID = 45443]
2. Clactonian technique [Option ID = 45444]
3. Pressure technique [Option ID = 45446]
4. Grinding and polishing technique [Option ID = 45445]
Correct Answer :-
Grinding and polishing technique [Option ID = 45445]
Khallu is a form : [Question ID = 18792]
1. Fishing [Option ID = 45160]
2. Hunting [Option ID = 45159]
3. Cropping [Option ID = 45161]
4. Weaving [Option ID = 45162]
Correct Answer :-
Cropping [Option ID = 45161]
Lascaux is
[Question ID = 18844]
1. Cave art site [Option ID = 45367]
2. Mesolithic site [Option ID = 45368]
3. Chalcolithic site [Option ID = 45369]
4. Neolithic site [Option ID = 45370]
Correct Answer :-
Cave art site [Option ID = 45367]
A social fact is not characterized by
[Question ID = 18862]
49)
50)
51)
52)
53)
54)
1. Inheritance [Option ID = 45441]
2. Independent existence [Option ID = 45439]
3. Constraints [Option ID = 45442]
4. Exteriority [Option ID = 45440]
Correct Answer :-
Inheritance [Option ID = 45441]
Chopper and Chopping tool are
[Question ID = 18864]
1. blade tool [Option ID = 45447]
2. flake tool [Option ID = 45449]
3. core tool [Option ID = 45448]
4. Neolithic tools [Option ID = 45450]
Correct Answer :-
core tool [Option ID = 45448]
The theory of evolution by natural selection was independently developed by: [Question ID = 19246]
1. Darwin and Lamarck [Option ID = 46977]
2. Darwin and Wallace [Option ID = 46976]
3. Lyell and Darwin [Option ID = 46975]
4. Lyell and Hutton [Option ID = 46978]
Correct Answer :-
Darwin and Lamarck [Option ID = 46977]
? Linea aspera? responsible for the erect posture in man is present in
[Question ID = 18846]
1. Anterior side of tibia [Option ID = 45376]
2. Anterior side of femur [Option ID = 45378]
3. Posterior side of femur [Option ID = 45375]
4. Medial side of fibula [Option ID = 45377]
Correct Answer :-
Posterior side of femur [Option ID = 45375]
?Purity and Danger? is associated with the work of
[Question ID = 18842]
1. M.N. Srinivas [Option ID = 45362]
2. Mary Douglas [Option ID = 45360]
3. Victor Turner [Option ID = 45359]
4. Edmund R. Leach [Option ID = 45361]
Correct Answer :-
Mary Douglas [Option ID = 45360]
?Swaddling Hypothesis? was used by the
[Question ID = 18832]
1. Evolutionist [Option ID = 45320]
2. Conflict Theorist [Option ID = 45321]
3. Cultural Ecologist [Option ID = 45319]
4. Cultural Personality Thinker [Option ID = 45322]
Correct Answer :-
Cultural Personality Thinker [Option ID = 45322]
Trapeze is [Question ID = 18806]
55)
56)
57)
58)
59)
60)
1. Neolithic tool [Option ID = 45215]
2. Core tool [Option ID = 45216]
3. Flake tool [Option ID = 45217]
4. Microlithic tool [Option ID = 45218]
Correct Answer :-
Microlithic tool [Option ID = 45218]
?Melanin? pigment is produced in our [Question ID = 18799]
1. Blood [Option ID = 45188]
2. Pancreas [Option ID = 45190]
3. Liver [Option ID = 45189]
4. Skin [Option ID = 45187]
Correct Answer :-
Skin [Option ID = 45187]
What is the earliest date of Indian Lower Paleolithic culture?
[Question ID = 18825]
1. 2.4 mya [Option ID = 45294]
2. 1.8 mya [Option ID = 45292]
3. 1.51 mya [Option ID = 45293]
4. 1.71 mya [Option ID = 45291]
Correct Answer :-
1.51 mya [Option ID = 45293]
Mutation Theory was proposed by:
[Question ID = 18865]
1. Hugo de Vries [Option ID = 45451]
2. Gregor Mendel. [Option ID = 45454]
3. August Weismann [Option ID = 45453]
4. R. A. Fischer [Option ID = 45452]
Correct Answer :-
Hugo de Vries [Option ID = 45451]
Gonadotropins are responsible for the growth of the-
[Question ID = 18839]
1. Whole body [Option ID = 45350]
2. Brain [Option ID = 45347]
3. Ovaries and testis [Option ID = 45348]
4. Immune system [Option ID = 45349]
Correct Answer :-
Ovaries and testis [Option ID = 45348]
Attempts to account for facts by means of general hypothesis or research questions is known as: [Question ID = 18794]
1. Phenomenology [Option ID = 45169]
2. Reduction [Option ID = 45170]
3. Induction [Option ID = 45167]
4. Deduction [Option ID = 45168]
Correct Answer :-
Deduction [Option ID = 45168]
Someone experiencing physiological problems as a result of oxygen deprivation is
[Question ID = 18818]
61)
62)
63)
64)
65)
1. Polycythaemia [Option ID = 45263]
2. Hyperoxia [Option ID = 45266]
3. Barometric excess [Option ID = 45264]
4. Hypoxia [Option ID = 45265]
Correct Answer :-
Hypoxia [Option ID = 45265]
Who gave the concept of ?the survival of the fittest?? [Question ID = 18808]
1. Hugo De Vries [Option ID = 45225]
2. Charles Darwin [Option ID = 45223]
3. Carl Correns [Option ID = 45226]
4. Herbert Spencer [Option ID = 45224]
Correct Answer :-
Herbert Spencer [Option ID = 45224]
Prepared core technique is also known as
[Question ID = 18845]
1. Punching technique [Option ID = 45374]
2. Levalloisian technique [Option ID = 45373]
3. Clactonian technique [Option ID = 45371]
4. Cylinder Hammer technique [Option ID = 45372]
Correct Answer :-
Levalloisian technique [Option ID = 45373]
A syndrome characterised by physiological, behavioural and mental defects that are due to the presence of an extra copy of the
chromosome 21 is
[Question ID = 18848]
1. Klinefelter syndrome [Option ID = 45385]
2. Marfan syndrome [Option ID = 45386]
3. Turner syndrome [Option ID = 45383]
4. Down syndrome [Option ID = 45384]
Correct Answer :-
Down syndrome [Option ID = 45384]
The statement "Animals are good to think with" is from
[Question ID = 18804]
1. Sherry B. Ortner [Option ID = 45209]
2. Margaret Mead [Option ID = 45210]
3. Claude L?vi Strauss [Option ID = 45208]
4. Mellisa Liewyn-Davies [Option ID = 45207]
Correct Answer :-
Claude L?vi Strauss [Option ID = 45208]
The physiological process of being or becoming accustomed to a new physical environment is known as:
[Question ID = 18877]
1. Alteration [Option ID = 45502]
2. Adaptation [Option ID = 45501]
3. Acclimatization [Option ID = 45500]
4. Assimilation [Option ID = 45499]
Correct Answer :-
Acclimatization [Option ID = 45500]
66)
67)
68)
69)
70)
71)
The famous ?Fox Project? was guided by
[Question ID = 18821]
1. Marvin Harris. [Option ID = 45278]
2. Sol Tax [Option ID = 45275]
3. George P. Murdock [Option ID = 45276]
4. Margaret Mead [Option ID = 45277]
Correct Answer :-
Sol Tax [Option ID = 45275]
The first Director of Anthropological Survey of India was
[Question ID = 18851]
1. S.C. Roy [Option ID = 45396]
2. N. K. Bose. [Option ID = 45395]
3. B.S. Guha [Option ID = 45398]
4. G.S.Ghurye [Option ID = 45397]
Correct Answer :-
B.S. Guha [Option ID = 45398]
The Cross Cultural Cumulative Coding Center (The 5 C?s ) has been developed by
[Question ID = 18833]
1. Ruth Benedict [Option ID = 45323]
2. Franz Boas. [Option ID = 45324]
3. G. P. Murdock [Option ID = 45326]
4. Jack Goody [Option ID = 45325]
Correct Answer :-
G. P. Murdock [Option ID = 45326]
The advisory committee for the revision of the list of Scheduled Castes and Scheduled Tribal is also known as
[Question ID = 18861]
1. Ghurye committee [Option ID = 45438]
2. Shilu Ao committee [Option ID = 45437]
3. Elwin committee [Option ID = 45436]
4. Lokur committee [Option ID = 45435]
Correct Answer :-
Lokur committee [Option ID = 45435]
The period referring to the process of growth from birth up to 28 days of life is-
[Question ID = 18837]
1. Postnatal [Option ID = 45339]
2. Infancy [Option ID = 45340]
3. Neo-natal [Option ID = 45342]
4. Prenatal [Option ID = 45341]
Correct Answer :-
Neo-natal [Option ID = 45342]
The book ?Homo Hierarchicus? is authored by
[Question ID = 18791]
1. Louis Dumont [Option ID = 45155]
2. M.N.Srinivas [Option ID = 45156]
3. G.S. Ghurye [Option ID = 45158]
4. Gerald Berreman [Option ID = 45157]
72)
73)
74)
75)
76)
77)
Correct Answer :-
Louis Dumont [Option ID = 45155]
The study of methods under social control directed towards improving the hereditary constitution of future human generations is
[Question ID = 18847]
1. Ergonomics [Option ID = 45382]
2. Eugenics [Option ID = 45380]
3. Genetics [Option ID = 45379]
4. Physiology [Option ID = 45381]
Correct Answer :-
Eugenics [Option ID = 45380]
The energy giving foods which include starch of cereals and sugar from sugarcane and fruits are-
[Question ID = 18829]
1. Vitamins [Option ID = 45310]
2. Proteins [Option ID = 45308]
3. Carbohydrates [Option ID = 45307]
4. Minerals [Option ID = 45309]
Correct Answer :-
Carbohydrates [Option ID = 45307]
The mixture of homozygous and heterozygous genes of a population in a state of equilibrium is called:
[Question ID = 18878]
1. Multiple alleles [Option ID = 45505]
2. Transient polymorphism [Option ID = 45504]
3. Balanced polymorphism [Option ID = 45503]
4. Genetic equilibrium. [Option ID = 45506]
Correct Answer :-
Balanced polymorphism [Option ID = 45503]
The classical definition of culture by E. B. Tylor appeared in the year
[Question ID = 18830]
1. 1871 [Option ID = 45313]
2. 1879 [Option ID = 45311]
3. 1870 [Option ID = 45312]
4. 1872 [Option ID = 45314]
Correct Answer :-
1871 [Option ID = 45313]
The Fifth Schedule of the Constitution of India deals with
[Question ID = 18811]
1. Economic development and educational opportunities for Scheduled Tribes [Option ID = 45238]
2. Atrocities and crime against Scheduled Tribes [Option ID = 45237]
3. Administration of the Scheduled Areas [Option ID = 45236]
4. Reservation for the Scheduled Tribes [Option ID = 45235]
Correct Answer :-
Administration of the Scheduled Areas [Option ID = 45236]
The idea of ?original affluent society? for hunting and gathering subsistence was proposed by
[Question ID = 18840]
FirstRanker.com - FirstRanker's Choice
1)
2)
3)
4)
5)
DU MSc Anthropology
Topic:- DU_J18_MSC_ANTHRO
Increased labour investment in agriculture with no increase in per capita productivity is called as
[Question ID = 18814]
1. Agricultural recession [Option ID = 45250]
2. Agricultural stagnation [Option ID = 45247]
3. Agricultural relegation [Option ID = 45248]
4. Agricultural involution [Option ID = 45249]
Correct Answer :-
Agricultural involution [Option ID = 45249]
Which of the following is a hominid?
[Question ID = 18857]
1. Chimpanzee [Option ID = 45421]
2. Human [Option ID = 45422]
3. Macaque [Option ID = 45419]
4. Baboon [Option ID = 45420]
Correct Answer :-
Human [Option ID = 45422]
Which rule says that the best suited human body for a hot, tropical region would be one with long limbs and short trunk?
[Question ID = 18827]
1. Allen?s rule [Option ID = 45299]
2. Thompson?s rule [Option ID = 45302]
3. Gloger?s rule [Option ID = 45301]
4. Bergman?s rule [Option ID = 45300]
Correct Answer :-
Allen?s rule [Option ID = 45299]
Which Australopithecus fossil was nicknamed as Lucy?
[Question ID = 18867]
1. Australopithecus africanus [Option ID = 45459]
2. Australopithecus afarensis [Option ID = 45462]
3. Australopithecus anamensis [Option ID = 45460]
4. Australopithecus robustus [Option ID = 45461]
Correct Answer :-
Australopithecus afarensis [Option ID = 45462]
Which among the following does not constitute the elementary form of exchange?
[Question ID = 18820]
1. Exchange of goods [Option ID = 45274]
2. Exchange of women [Option ID = 45271]
3. Exchange of money [Option ID = 45272]
4. Exchange of messages [Option ID = 45273]
Correct Answer :-
Exchange of goods [Option ID = 45274]
6)
7)
8)
9)
10)
11)
12)
Which prehistoric site is known as Mode I Industry?
[Question ID = 18835]
1. Pirro Nord [Option ID = 45333]
2. Hunsgi [Option ID = 45331]
3. Tautavel [Option ID = 45332]
4. Somme valley [Option ID = 45334]
Correct Answer :-
Pirro Nord [Option ID = 45333]
Any quantitative increase in body size is known as [Question ID = 18788]
1. Development [Option ID = 45144]
2. Maturation [Option ID = 45146]
3. Differentiation [Option ID = 45145]
4. Growth [Option ID = 45143]
Correct Answer :-
Growth [Option ID = 45143]
The highest point on the head is known as [Question ID = 18807]
1. Opisthocranion [Option ID = 45222]
2. Bregma [Option ID = 45220]
3. Vertex [Option ID = 45221]
4. Lambda [Option ID = 45219]
Correct Answer :-
Vertex [Option ID = 45221]
Autosomes are
[Question ID = 18826]
1. Normal sex chromosomes [Option ID = 45296]
2. Abnormal sex chromosomes [Option ID = 45298]
3. Abnormal chromosomes [Option ID = 45297]
4. All chromosomes other than the sex chromosomes [Option ID = 45295]
Correct Answer :-
All chromosomes other than the sex chromosomes [Option ID = 45295]
?Puberty spurt? occurs in which one of the following stages of human growth [Question ID = 18798]
1. Infancy [Option ID = 45183]
2. Childhood [Option ID = 45185]
3. Adolescence [Option ID = 45186]
4. Adulthood [Option ID = 45184]
Correct Answer :-
Adolescence [Option ID = 45186]
Potlatch is associated with [Question ID = 18802]
1. Iroquois [Option ID = 45201]
2. Kwakiutl [Option ID = 45199]
3. Shoshonean [Option ID = 45202]
4. Arapesh [Option ID = 45200]
Correct Answer :-
Kwakiutl [Option ID = 45199]
Oldowan culture is dominated by
[Question ID = 18824]
1. Blade tools [Option ID = 45288]
2. Flake tools [Option ID = 45289]
13)
14)
15)
16)
17)
18)
3. Pebble tools [Option ID = 45287]
4. Microlithic tools [Option ID = 45290]
Correct Answer :-
Pebble tools [Option ID = 45287]
A higher fortified part of a city is called
[Question ID = 18823]
1. metropolis [Option ID = 45284]
2. megapolis [Option ID = 45285]
3. acropolis [Option ID = 45283]
4. necropolis [Option ID = 45286]
Correct Answer :-
acropolis [Option ID = 45283]
Yale Cambridge expedition to the Sohan Valley was led by
[Question ID = 18795]
1. R. B. Foote and V.D. Krishnaswamy [Option ID = 45171]
2. G. R. Sharma and V.D. Mishra [Option ID = 45174]
3. H. D. Sankalia and B. Subba Rao [Option ID = 45173]
4. H. De. Terra and T. T. Paterson [Option ID = 45172]
Correct Answer :-
H. De. Terra and T. T. Paterson [Option ID = 45172]
Two nuclear families in adjacent generations with one son/husband or daughter/wife who is a member of both the families is called a
[Question ID = 18870]
1. lineal family [Option ID = 45471]
2. supplemented nuclear family [Option ID = 45474]
3. stem family [Option ID = 45473]
4. compound family [Option ID = 45472]
Correct Answer :-
stem family [Option ID = 45473]
The lack, excess or imbalance of nutrients in the diet leading to ill health is known as [Question ID = 18790]
1. Malnutrition [Option ID = 45153]
2. Over nutrition [Option ID = 45151]
3. Under nutrition [Option ID = 45152]
4. Marasmus [Option ID = 45154]
Correct Answer :-
Malnutrition [Option ID = 45153]
Holocene starts around
[Question ID = 18834]
1. 20,000 B.P. [Option ID = 45329]
2. 12,000 B.P. [Option ID = 45328]
3. 14,000 B.P. [Option ID = 45330]
4. 10,000 B.P. [Option ID = 45327]
Correct Answer :-
10,000 B.P. [Option ID = 45327]
According to Levi-Strauss which of the following constitute one of the ?atoms of kinship? relationship?
[Question ID = 18879]
19)
20)
21)
22)
23)
24)
1. mother-son [Option ID = 45507]
2. mother-daughter [Option ID = 45508]
3. father-son [Option ID = 45509]
4. father-daughter [Option ID = 45510]
Correct Answer :-
mother-son [Option ID = 45507]
A karyotype is a:
[Question ID = 18819]
1. Picture of an individual's chromosomes arranged in a standardized way [Option ID = 45269]
2. Type of abnormal chromosome that is associated with Down's syndrome [Option ID = 45268]
3. General term for any type of chromosome [Option ID = 45267]
4. General term for sex chromosome [Option ID = 45270]
Correct Answer :-
Picture of an individual's chromosomes arranged in a standardized way [Option ID = 45269]
Theodicy is an
[Question ID = 18882]
1. attempt to explain theocracy [Option ID = 45521]
2. attempt to explain good [Option ID = 45519]
3. attempt to explain evil [Option ID = 45520]
4. attempt to explain lie and deceit [Option ID = 45522]
Correct Answer :-
attempt to explain evil [Option ID = 45520]
The word ?Somatotype? was coined by [Question ID = 18797]
1. Faulkner [Option ID = 45182]
2. Sheldon [Option ID = 45180]
3. Parnell [Option ID = 45181]
4. Heath and Carter [Option ID = 45179]
Correct Answer :-
Heath and Carter [Option ID = 45179]
Substantivism as an approach in economic anthropology was given by
[Question ID = 18853]
1. A. L. Kroeber [Option ID = 45406]
2. Karl Polanyi [Option ID = 45405]
3. Oscar Lewis [Option ID = 45403]
4. Robert Redfield [Option ID = 62620]
Correct Answer :-
Karl Polanyi [Option ID = 45405]
?Genetics and the Origin of Species? was written by :
[Question ID = 18859]
1. Thomas Malthus. [Option ID = 45430]
2. Theodosius Dobzhansky [Option ID = 45428]
3. Charles Darwin [Option ID = 45427]
4. T. H. Morgan [Option ID = 45429]
Correct Answer :-
Theodosius Dobzhansky [Option ID = 45428]
The ability of the living organisms to survive in a particular ecological set up is called [Question ID = 18809]
25)
26)
27)
28)
29)
30)
1. Acclimation [Option ID = 45228]
2. Acclimatisation [Option ID = 45229]
3. Adaptation [Option ID = 45227]
4. Selection [Option ID = 45230]
Correct Answer :-
Adaptation [Option ID = 45227]
___________is the physiological ability to reproduce an offspring.
[Question ID = 18876]
1. Fecundity [Option ID = 45496]
2. Fertility [Option ID = 45495]
3. Virility [Option ID = 45498]
4. Natality [Option ID = 45497]
Correct Answer :-
Fecundity [Option ID = 45496]
Charles Darwin's ideas concerning the causes of evolution were probably formulated in his mind
[Question ID = 18817]
1. During the late 1880's [Option ID = 45262]
2. During his voyage on H.M.S. Beagle, especially after he reached the Gal?pagos Islands [Option ID = 45261]
3. Before he began his voyage of exploration around the world on H.M.S. Beagle [Option ID = 45260]
4. While he was still a student at Cambridge University [Option ID = 45259]
Correct Answer :-
During his voyage on H.M.S. Beagle, especially after he reached the Gal?pagos Islands [Option ID = 45261]
First Himalayan Glaciation is known as
[Question ID = 18884]
1. Terrace I [Option ID = 45530]
2. Pinjaur [Option ID = 45527]
3. Boulder conglomerate [Option ID = 45529]
4. Tatrot [Option ID = 45528]
Correct Answer :-
Tatrot [Option ID = 45528]
The author of the book Hindus of the Himalayas is [Question ID = 18812]
1. S.C. Dube [Option ID = 45241]
2. D.N. Majumdar [Option ID = 45242]
3. B. K. Roy-Burman [Option ID = 45240]
4. G.D. Berreman [Option ID = 45239]
Correct Answer :-
G.D. Berreman [Option ID = 45239]
Who proposed the ?Three Age System? in pre-history?
[Question ID = 18883]
1. C. J. Thomsen [Option ID = 45525]
2. B. M. Fagan [Option ID = 45523]
3. H.D.Sankalia [Option ID = 45524]
4. F. Borde [Option ID = 45526]
Correct Answer :-
C. J. Thomsen [Option ID = 45525]
Who among the following worked on the complicity between colonial agencies and Anthropologists?
31)
32)
33)
34)
35)
36)
[Question ID = 18801]
1. Ibn Khaldun [Option ID = 45196]
2. Akbar Ahmed [Option ID = 45197]
3. Ralf Linto [Option ID = 45198]
4. Talal Asad [Option ID = 45195]
Correct Answer :-
Talal Asad [Option ID = 45195]
Who is the author of the book ?The Old Stone Age??
[Question ID = 18854]
1. M.C. Burkitt [Option ID = 45407]
2. J. N. Pal [Option ID = 45408]
3. P.C. Pant [Option ID = 45409]
4. H. De Lumley [Option ID = 45410]
Correct Answer :-
M.C. Burkitt [Option ID = 45407]
Who included the Proto-Australoid racial element in the classification of Indian races?
[Question ID = 18866]
1. S.S. Sarkar [Option ID = 45456]
2. H.H. Risley [Option ID = 45455]
3. B.S. Guha [Option ID = 45458]
4. A.C. Haddon [Option ID = 45457]
Correct Answer :-
B.S. Guha [Option ID = 45458]
In which State is the site Isampur located?
[Question ID = 18815]
1. Karnataka [Option ID = 45253]
2. Uttar Pradesh [Option ID = 45254]
3. Madhya Pradesh [Option ID = 45252]
4. Bihar [Option ID = 45251]
Correct Answer :-
Karnataka [Option ID = 45253]
The view of the skull from above is [Question ID = 18800]
1. Norma Basalis [Option ID = 45193]
2. Norma Occipitalis [Option ID = 45194]
3. Norma Frontalis [Option ID = 45192]
4. Norma Verticalis [Option ID = 45191]
Correct Answer :-
Norma Verticalis [Option ID = 45191]
?Childhood? is peculiar to [Question ID = 18789]
1. Nonhuman primates [Option ID = 45149]
2. Human and nonhuman primates [Option ID = 45147]
3. Humans [Option ID = 45148]
4. Mammals [Option ID = 45150]
Correct Answer :-
Humans [Option ID = 45148]
Children subjected to starvation for short periods may recover completely on getting adequate diet. This is referred to as
37)
38)
39)
40)
41)
[Question ID = 18838]
1. Growth spurt [Option ID = 45346]
2. Catch down growth [Option ID = 45345]
3. Catch-up growth [Option ID = 45343]
4. Canalization [Option ID = 45344]
Correct Answer :-
Catch-up growth [Option ID = 45343]
Attirampakkam is [Question ID = 18805]
1. Lower Paleolithic site [Option ID = 45211]
2. Mesolithic site [Option ID = 45214]
3. Indus Valley Civilization site [Option ID = 45213]
4. Upper Paleolithic site [Option ID = 45212]
Correct Answer :-
Lower Paleolithic site [Option ID = 45211]
Shivering
[Question ID = 18828]
1. Has no effect on body heat [Option ID = 45306]
2. Indirectly produces more body heat [Option ID = 45304]
3. Speeds up the loss of body heat [Option ID = 45303]
4. Delays up the loss of body heat [Option ID = 45305]
Correct Answer :-
Indirectly produces more body heat [Option ID = 45304]
A deficiency disease of the skeletal system caused by a lack of Vitamin D or Calcium or both, and often resulting in bone deformities is
[Question ID = 18849]
1. Arthritis [Option ID = 45390]
2. Scurvy [Option ID = 45387]
3. Rickets [Option ID = 45389]
4. Osteoporosis [Option ID = 45388]
Correct Answer :-
Rickets [Option ID = 45389]
A deficiency disease caused due to severe lack of protein in diet among children is
[Question ID = 18856]
1. Anaemia [Option ID = 45418]
2. Kwashiorkor [Option ID = 45417]
3. Scurvy [Option ID = 45416]
4. Rickets [Option ID = 45415]
Correct Answer :-
Kwashiorkor [Option ID = 45417]
Samples of Dendrochronology is taken from which of the following?
[Question ID = 18873]
1. Lake [Option ID = 45483]
2. Tree [Option ID = 45486]
3. River [Option ID = 45484]
4. sea [Option ID = 45485]
Correct Answer :-
Tree [Option ID = 45486]
42)
43)
44)
45)
46)
47)
48)
Swiddening is a form of
[Question ID = 18822]
1. terrace cultivation [Option ID = 45281]
2. intensive cultivation [Option ID = 45280]
3. commercial cultivation [Option ID = 45282]
4. slash and burn cultivation [Option ID = 45279]
Correct Answer :-
slash and burn cultivation [Option ID = 45279]
Osteology is the study of [Question ID = 18810]
1. The human skeleton [Option ID = 45234]
2. The human brain [Option ID = 45233]
3. Human survival strategies [Option ID = 45231]
4. Human languages [Option ID = 45232]
Correct Answer :-
The human skeleton [Option ID = 45234]
Extended Case Study method in anthropology is associated with the work of [Question ID = 18813]
1. Paul Radin [Option ID = 45246]
2. Julian Steward [Option ID = 45244]
3. Anthony Walker [Option ID = 45245]
4. Max Gluckman [Option ID = 45243]
Correct Answer :-
Max Gluckman [Option ID = 45243]
Celts are prepared by
[Question ID = 18863]
1. Levalloisian technique [Option ID = 45443]
2. Clactonian technique [Option ID = 45444]
3. Pressure technique [Option ID = 45446]
4. Grinding and polishing technique [Option ID = 45445]
Correct Answer :-
Grinding and polishing technique [Option ID = 45445]
Khallu is a form : [Question ID = 18792]
1. Fishing [Option ID = 45160]
2. Hunting [Option ID = 45159]
3. Cropping [Option ID = 45161]
4. Weaving [Option ID = 45162]
Correct Answer :-
Cropping [Option ID = 45161]
Lascaux is
[Question ID = 18844]
1. Cave art site [Option ID = 45367]
2. Mesolithic site [Option ID = 45368]
3. Chalcolithic site [Option ID = 45369]
4. Neolithic site [Option ID = 45370]
Correct Answer :-
Cave art site [Option ID = 45367]
A social fact is not characterized by
[Question ID = 18862]
49)
50)
51)
52)
53)
54)
1. Inheritance [Option ID = 45441]
2. Independent existence [Option ID = 45439]
3. Constraints [Option ID = 45442]
4. Exteriority [Option ID = 45440]
Correct Answer :-
Inheritance [Option ID = 45441]
Chopper and Chopping tool are
[Question ID = 18864]
1. blade tool [Option ID = 45447]
2. flake tool [Option ID = 45449]
3. core tool [Option ID = 45448]
4. Neolithic tools [Option ID = 45450]
Correct Answer :-
core tool [Option ID = 45448]
The theory of evolution by natural selection was independently developed by: [Question ID = 19246]
1. Darwin and Lamarck [Option ID = 46977]
2. Darwin and Wallace [Option ID = 46976]
3. Lyell and Darwin [Option ID = 46975]
4. Lyell and Hutton [Option ID = 46978]
Correct Answer :-
Darwin and Lamarck [Option ID = 46977]
? Linea aspera? responsible for the erect posture in man is present in
[Question ID = 18846]
1. Anterior side of tibia [Option ID = 45376]
2. Anterior side of femur [Option ID = 45378]
3. Posterior side of femur [Option ID = 45375]
4. Medial side of fibula [Option ID = 45377]
Correct Answer :-
Posterior side of femur [Option ID = 45375]
?Purity and Danger? is associated with the work of
[Question ID = 18842]
1. M.N. Srinivas [Option ID = 45362]
2. Mary Douglas [Option ID = 45360]
3. Victor Turner [Option ID = 45359]
4. Edmund R. Leach [Option ID = 45361]
Correct Answer :-
Mary Douglas [Option ID = 45360]
?Swaddling Hypothesis? was used by the
[Question ID = 18832]
1. Evolutionist [Option ID = 45320]
2. Conflict Theorist [Option ID = 45321]
3. Cultural Ecologist [Option ID = 45319]
4. Cultural Personality Thinker [Option ID = 45322]
Correct Answer :-
Cultural Personality Thinker [Option ID = 45322]
Trapeze is [Question ID = 18806]
55)
56)
57)
58)
59)
60)
1. Neolithic tool [Option ID = 45215]
2. Core tool [Option ID = 45216]
3. Flake tool [Option ID = 45217]
4. Microlithic tool [Option ID = 45218]
Correct Answer :-
Microlithic tool [Option ID = 45218]
?Melanin? pigment is produced in our [Question ID = 18799]
1. Blood [Option ID = 45188]
2. Pancreas [Option ID = 45190]
3. Liver [Option ID = 45189]
4. Skin [Option ID = 45187]
Correct Answer :-
Skin [Option ID = 45187]
What is the earliest date of Indian Lower Paleolithic culture?
[Question ID = 18825]
1. 2.4 mya [Option ID = 45294]
2. 1.8 mya [Option ID = 45292]
3. 1.51 mya [Option ID = 45293]
4. 1.71 mya [Option ID = 45291]
Correct Answer :-
1.51 mya [Option ID = 45293]
Mutation Theory was proposed by:
[Question ID = 18865]
1. Hugo de Vries [Option ID = 45451]
2. Gregor Mendel. [Option ID = 45454]
3. August Weismann [Option ID = 45453]
4. R. A. Fischer [Option ID = 45452]
Correct Answer :-
Hugo de Vries [Option ID = 45451]
Gonadotropins are responsible for the growth of the-
[Question ID = 18839]
1. Whole body [Option ID = 45350]
2. Brain [Option ID = 45347]
3. Ovaries and testis [Option ID = 45348]
4. Immune system [Option ID = 45349]
Correct Answer :-
Ovaries and testis [Option ID = 45348]
Attempts to account for facts by means of general hypothesis or research questions is known as: [Question ID = 18794]
1. Phenomenology [Option ID = 45169]
2. Reduction [Option ID = 45170]
3. Induction [Option ID = 45167]
4. Deduction [Option ID = 45168]
Correct Answer :-
Deduction [Option ID = 45168]
Someone experiencing physiological problems as a result of oxygen deprivation is
[Question ID = 18818]
61)
62)
63)
64)
65)
1. Polycythaemia [Option ID = 45263]
2. Hyperoxia [Option ID = 45266]
3. Barometric excess [Option ID = 45264]
4. Hypoxia [Option ID = 45265]
Correct Answer :-
Hypoxia [Option ID = 45265]
Who gave the concept of ?the survival of the fittest?? [Question ID = 18808]
1. Hugo De Vries [Option ID = 45225]
2. Charles Darwin [Option ID = 45223]
3. Carl Correns [Option ID = 45226]
4. Herbert Spencer [Option ID = 45224]
Correct Answer :-
Herbert Spencer [Option ID = 45224]
Prepared core technique is also known as
[Question ID = 18845]
1. Punching technique [Option ID = 45374]
2. Levalloisian technique [Option ID = 45373]
3. Clactonian technique [Option ID = 45371]
4. Cylinder Hammer technique [Option ID = 45372]
Correct Answer :-
Levalloisian technique [Option ID = 45373]
A syndrome characterised by physiological, behavioural and mental defects that are due to the presence of an extra copy of the
chromosome 21 is
[Question ID = 18848]
1. Klinefelter syndrome [Option ID = 45385]
2. Marfan syndrome [Option ID = 45386]
3. Turner syndrome [Option ID = 45383]
4. Down syndrome [Option ID = 45384]
Correct Answer :-
Down syndrome [Option ID = 45384]
The statement "Animals are good to think with" is from
[Question ID = 18804]
1. Sherry B. Ortner [Option ID = 45209]
2. Margaret Mead [Option ID = 45210]
3. Claude L?vi Strauss [Option ID = 45208]
4. Mellisa Liewyn-Davies [Option ID = 45207]
Correct Answer :-
Claude L?vi Strauss [Option ID = 45208]
The physiological process of being or becoming accustomed to a new physical environment is known as:
[Question ID = 18877]
1. Alteration [Option ID = 45502]
2. Adaptation [Option ID = 45501]
3. Acclimatization [Option ID = 45500]
4. Assimilation [Option ID = 45499]
Correct Answer :-
Acclimatization [Option ID = 45500]
66)
67)
68)
69)
70)
71)
The famous ?Fox Project? was guided by
[Question ID = 18821]
1. Marvin Harris. [Option ID = 45278]
2. Sol Tax [Option ID = 45275]
3. George P. Murdock [Option ID = 45276]
4. Margaret Mead [Option ID = 45277]
Correct Answer :-
Sol Tax [Option ID = 45275]
The first Director of Anthropological Survey of India was
[Question ID = 18851]
1. S.C. Roy [Option ID = 45396]
2. N. K. Bose. [Option ID = 45395]
3. B.S. Guha [Option ID = 45398]
4. G.S.Ghurye [Option ID = 45397]
Correct Answer :-
B.S. Guha [Option ID = 45398]
The Cross Cultural Cumulative Coding Center (The 5 C?s ) has been developed by
[Question ID = 18833]
1. Ruth Benedict [Option ID = 45323]
2. Franz Boas. [Option ID = 45324]
3. G. P. Murdock [Option ID = 45326]
4. Jack Goody [Option ID = 45325]
Correct Answer :-
G. P. Murdock [Option ID = 45326]
The advisory committee for the revision of the list of Scheduled Castes and Scheduled Tribal is also known as
[Question ID = 18861]
1. Ghurye committee [Option ID = 45438]
2. Shilu Ao committee [Option ID = 45437]
3. Elwin committee [Option ID = 45436]
4. Lokur committee [Option ID = 45435]
Correct Answer :-
Lokur committee [Option ID = 45435]
The period referring to the process of growth from birth up to 28 days of life is-
[Question ID = 18837]
1. Postnatal [Option ID = 45339]
2. Infancy [Option ID = 45340]
3. Neo-natal [Option ID = 45342]
4. Prenatal [Option ID = 45341]
Correct Answer :-
Neo-natal [Option ID = 45342]
The book ?Homo Hierarchicus? is authored by
[Question ID = 18791]
1. Louis Dumont [Option ID = 45155]
2. M.N.Srinivas [Option ID = 45156]
3. G.S. Ghurye [Option ID = 45158]
4. Gerald Berreman [Option ID = 45157]
72)
73)
74)
75)
76)
77)
Correct Answer :-
Louis Dumont [Option ID = 45155]
The study of methods under social control directed towards improving the hereditary constitution of future human generations is
[Question ID = 18847]
1. Ergonomics [Option ID = 45382]
2. Eugenics [Option ID = 45380]
3. Genetics [Option ID = 45379]
4. Physiology [Option ID = 45381]
Correct Answer :-
Eugenics [Option ID = 45380]
The energy giving foods which include starch of cereals and sugar from sugarcane and fruits are-
[Question ID = 18829]
1. Vitamins [Option ID = 45310]
2. Proteins [Option ID = 45308]
3. Carbohydrates [Option ID = 45307]
4. Minerals [Option ID = 45309]
Correct Answer :-
Carbohydrates [Option ID = 45307]
The mixture of homozygous and heterozygous genes of a population in a state of equilibrium is called:
[Question ID = 18878]
1. Multiple alleles [Option ID = 45505]
2. Transient polymorphism [Option ID = 45504]
3. Balanced polymorphism [Option ID = 45503]
4. Genetic equilibrium. [Option ID = 45506]
Correct Answer :-
Balanced polymorphism [Option ID = 45503]
The classical definition of culture by E. B. Tylor appeared in the year
[Question ID = 18830]
1. 1871 [Option ID = 45313]
2. 1879 [Option ID = 45311]
3. 1870 [Option ID = 45312]
4. 1872 [Option ID = 45314]
Correct Answer :-
1871 [Option ID = 45313]
The Fifth Schedule of the Constitution of India deals with
[Question ID = 18811]
1. Economic development and educational opportunities for Scheduled Tribes [Option ID = 45238]
2. Atrocities and crime against Scheduled Tribes [Option ID = 45237]
3. Administration of the Scheduled Areas [Option ID = 45236]
4. Reservation for the Scheduled Tribes [Option ID = 45235]
Correct Answer :-
Administration of the Scheduled Areas [Option ID = 45236]
The idea of ?original affluent society? for hunting and gathering subsistence was proposed by
[Question ID = 18840]
78)
79)
80)
81)
82)
1. Leslie A. White [Option ID = 45351]
2. Marshall D. Sahlins [Option ID = 45354]
3. Roy Rappaport [Option ID = 45353]
4. Julian Steward [Option ID = 45352]
Correct Answer :-
Marshall D. Sahlins [Option ID = 45354]
The concept ?Tribal-Rajput continuum? was given by
[Question ID = 18841]
1. S.S. Sarkar [Option ID = 45357]
2. L. P. Vidyarthi [Option ID = 45356]
3. B. S. Guha [Option ID = 45358]
4. Surjit Sinha [Option ID = 45355]
Correct Answer :-
Surjit Sinha [Option ID = 45355]
The concept ?modal personality? was popularized by
[Question ID = 18831]
1. Abram Kardiner [Option ID = 45316]
2. Ruth Benedict [Option ID = 45317]
3. Geoffrey Gorer [Option ID = 45318]
4. Cora Dubois [Option ID = 45315]
Correct Answer :-
Abram Kardiner [Option ID = 45316]
The concept of ?social network? was first used by
[Question ID = 18860]
1. A. L. Kroeber [Option ID = 45432]
2. G. P. Murdock [Option ID = 45434]
3. John Barnes [Option ID = 45431]
4. Kluckhohn [Option ID = 45433]
Correct Answer :-
John Barnes [Option ID = 45431]
The concept of ?complimentary filiations? was given by
[Question ID = 18881]
1. Raymond Firth [Option ID = 45518]
2. Evans-Pritchard [Option ID = 45516]
3. Radcliffe-Brown [Option ID = 45517]
4. Meyer Fortes [Option ID = 45515]
Correct Answer :-
Meyer Fortes [Option ID = 45515]
The concept of folk-urban continuum was proposed by
[Question ID = 18852]
1. Teodor Shanin [Option ID = 45400]
2. Eric R. Wolf [Option ID = 45399]
3. Milton Singer [Option ID = 45402]
4. Robert Redfield [Option ID = 45401]
Correct Answer :-
Robert Redfield [Option ID = 45401]
FirstRanker.com - FirstRanker's Choice
1)
2)
3)
4)
5)
DU MSc Anthropology
Topic:- DU_J18_MSC_ANTHRO
Increased labour investment in agriculture with no increase in per capita productivity is called as
[Question ID = 18814]
1. Agricultural recession [Option ID = 45250]
2. Agricultural stagnation [Option ID = 45247]
3. Agricultural relegation [Option ID = 45248]
4. Agricultural involution [Option ID = 45249]
Correct Answer :-
Agricultural involution [Option ID = 45249]
Which of the following is a hominid?
[Question ID = 18857]
1. Chimpanzee [Option ID = 45421]
2. Human [Option ID = 45422]
3. Macaque [Option ID = 45419]
4. Baboon [Option ID = 45420]
Correct Answer :-
Human [Option ID = 45422]
Which rule says that the best suited human body for a hot, tropical region would be one with long limbs and short trunk?
[Question ID = 18827]
1. Allen?s rule [Option ID = 45299]
2. Thompson?s rule [Option ID = 45302]
3. Gloger?s rule [Option ID = 45301]
4. Bergman?s rule [Option ID = 45300]
Correct Answer :-
Allen?s rule [Option ID = 45299]
Which Australopithecus fossil was nicknamed as Lucy?
[Question ID = 18867]
1. Australopithecus africanus [Option ID = 45459]
2. Australopithecus afarensis [Option ID = 45462]
3. Australopithecus anamensis [Option ID = 45460]
4. Australopithecus robustus [Option ID = 45461]
Correct Answer :-
Australopithecus afarensis [Option ID = 45462]
Which among the following does not constitute the elementary form of exchange?
[Question ID = 18820]
1. Exchange of goods [Option ID = 45274]
2. Exchange of women [Option ID = 45271]
3. Exchange of money [Option ID = 45272]
4. Exchange of messages [Option ID = 45273]
Correct Answer :-
Exchange of goods [Option ID = 45274]
6)
7)
8)
9)
10)
11)
12)
Which prehistoric site is known as Mode I Industry?
[Question ID = 18835]
1. Pirro Nord [Option ID = 45333]
2. Hunsgi [Option ID = 45331]
3. Tautavel [Option ID = 45332]
4. Somme valley [Option ID = 45334]
Correct Answer :-
Pirro Nord [Option ID = 45333]
Any quantitative increase in body size is known as [Question ID = 18788]
1. Development [Option ID = 45144]
2. Maturation [Option ID = 45146]
3. Differentiation [Option ID = 45145]
4. Growth [Option ID = 45143]
Correct Answer :-
Growth [Option ID = 45143]
The highest point on the head is known as [Question ID = 18807]
1. Opisthocranion [Option ID = 45222]
2. Bregma [Option ID = 45220]
3. Vertex [Option ID = 45221]
4. Lambda [Option ID = 45219]
Correct Answer :-
Vertex [Option ID = 45221]
Autosomes are
[Question ID = 18826]
1. Normal sex chromosomes [Option ID = 45296]
2. Abnormal sex chromosomes [Option ID = 45298]
3. Abnormal chromosomes [Option ID = 45297]
4. All chromosomes other than the sex chromosomes [Option ID = 45295]
Correct Answer :-
All chromosomes other than the sex chromosomes [Option ID = 45295]
?Puberty spurt? occurs in which one of the following stages of human growth [Question ID = 18798]
1. Infancy [Option ID = 45183]
2. Childhood [Option ID = 45185]
3. Adolescence [Option ID = 45186]
4. Adulthood [Option ID = 45184]
Correct Answer :-
Adolescence [Option ID = 45186]
Potlatch is associated with [Question ID = 18802]
1. Iroquois [Option ID = 45201]
2. Kwakiutl [Option ID = 45199]
3. Shoshonean [Option ID = 45202]
4. Arapesh [Option ID = 45200]
Correct Answer :-
Kwakiutl [Option ID = 45199]
Oldowan culture is dominated by
[Question ID = 18824]
1. Blade tools [Option ID = 45288]
2. Flake tools [Option ID = 45289]
13)
14)
15)
16)
17)
18)
3. Pebble tools [Option ID = 45287]
4. Microlithic tools [Option ID = 45290]
Correct Answer :-
Pebble tools [Option ID = 45287]
A higher fortified part of a city is called
[Question ID = 18823]
1. metropolis [Option ID = 45284]
2. megapolis [Option ID = 45285]
3. acropolis [Option ID = 45283]
4. necropolis [Option ID = 45286]
Correct Answer :-
acropolis [Option ID = 45283]
Yale Cambridge expedition to the Sohan Valley was led by
[Question ID = 18795]
1. R. B. Foote and V.D. Krishnaswamy [Option ID = 45171]
2. G. R. Sharma and V.D. Mishra [Option ID = 45174]
3. H. D. Sankalia and B. Subba Rao [Option ID = 45173]
4. H. De. Terra and T. T. Paterson [Option ID = 45172]
Correct Answer :-
H. De. Terra and T. T. Paterson [Option ID = 45172]
Two nuclear families in adjacent generations with one son/husband or daughter/wife who is a member of both the families is called a
[Question ID = 18870]
1. lineal family [Option ID = 45471]
2. supplemented nuclear family [Option ID = 45474]
3. stem family [Option ID = 45473]
4. compound family [Option ID = 45472]
Correct Answer :-
stem family [Option ID = 45473]
The lack, excess or imbalance of nutrients in the diet leading to ill health is known as [Question ID = 18790]
1. Malnutrition [Option ID = 45153]
2. Over nutrition [Option ID = 45151]
3. Under nutrition [Option ID = 45152]
4. Marasmus [Option ID = 45154]
Correct Answer :-
Malnutrition [Option ID = 45153]
Holocene starts around
[Question ID = 18834]
1. 20,000 B.P. [Option ID = 45329]
2. 12,000 B.P. [Option ID = 45328]
3. 14,000 B.P. [Option ID = 45330]
4. 10,000 B.P. [Option ID = 45327]
Correct Answer :-
10,000 B.P. [Option ID = 45327]
According to Levi-Strauss which of the following constitute one of the ?atoms of kinship? relationship?
[Question ID = 18879]
19)
20)
21)
22)
23)
24)
1. mother-son [Option ID = 45507]
2. mother-daughter [Option ID = 45508]
3. father-son [Option ID = 45509]
4. father-daughter [Option ID = 45510]
Correct Answer :-
mother-son [Option ID = 45507]
A karyotype is a:
[Question ID = 18819]
1. Picture of an individual's chromosomes arranged in a standardized way [Option ID = 45269]
2. Type of abnormal chromosome that is associated with Down's syndrome [Option ID = 45268]
3. General term for any type of chromosome [Option ID = 45267]
4. General term for sex chromosome [Option ID = 45270]
Correct Answer :-
Picture of an individual's chromosomes arranged in a standardized way [Option ID = 45269]
Theodicy is an
[Question ID = 18882]
1. attempt to explain theocracy [Option ID = 45521]
2. attempt to explain good [Option ID = 45519]
3. attempt to explain evil [Option ID = 45520]
4. attempt to explain lie and deceit [Option ID = 45522]
Correct Answer :-
attempt to explain evil [Option ID = 45520]
The word ?Somatotype? was coined by [Question ID = 18797]
1. Faulkner [Option ID = 45182]
2. Sheldon [Option ID = 45180]
3. Parnell [Option ID = 45181]
4. Heath and Carter [Option ID = 45179]
Correct Answer :-
Heath and Carter [Option ID = 45179]
Substantivism as an approach in economic anthropology was given by
[Question ID = 18853]
1. A. L. Kroeber [Option ID = 45406]
2. Karl Polanyi [Option ID = 45405]
3. Oscar Lewis [Option ID = 45403]
4. Robert Redfield [Option ID = 62620]
Correct Answer :-
Karl Polanyi [Option ID = 45405]
?Genetics and the Origin of Species? was written by :
[Question ID = 18859]
1. Thomas Malthus. [Option ID = 45430]
2. Theodosius Dobzhansky [Option ID = 45428]
3. Charles Darwin [Option ID = 45427]
4. T. H. Morgan [Option ID = 45429]
Correct Answer :-
Theodosius Dobzhansky [Option ID = 45428]
The ability of the living organisms to survive in a particular ecological set up is called [Question ID = 18809]
25)
26)
27)
28)
29)
30)
1. Acclimation [Option ID = 45228]
2. Acclimatisation [Option ID = 45229]
3. Adaptation [Option ID = 45227]
4. Selection [Option ID = 45230]
Correct Answer :-
Adaptation [Option ID = 45227]
___________is the physiological ability to reproduce an offspring.
[Question ID = 18876]
1. Fecundity [Option ID = 45496]
2. Fertility [Option ID = 45495]
3. Virility [Option ID = 45498]
4. Natality [Option ID = 45497]
Correct Answer :-
Fecundity [Option ID = 45496]
Charles Darwin's ideas concerning the causes of evolution were probably formulated in his mind
[Question ID = 18817]
1. During the late 1880's [Option ID = 45262]
2. During his voyage on H.M.S. Beagle, especially after he reached the Gal?pagos Islands [Option ID = 45261]
3. Before he began his voyage of exploration around the world on H.M.S. Beagle [Option ID = 45260]
4. While he was still a student at Cambridge University [Option ID = 45259]
Correct Answer :-
During his voyage on H.M.S. Beagle, especially after he reached the Gal?pagos Islands [Option ID = 45261]
First Himalayan Glaciation is known as
[Question ID = 18884]
1. Terrace I [Option ID = 45530]
2. Pinjaur [Option ID = 45527]
3. Boulder conglomerate [Option ID = 45529]
4. Tatrot [Option ID = 45528]
Correct Answer :-
Tatrot [Option ID = 45528]
The author of the book Hindus of the Himalayas is [Question ID = 18812]
1. S.C. Dube [Option ID = 45241]
2. D.N. Majumdar [Option ID = 45242]
3. B. K. Roy-Burman [Option ID = 45240]
4. G.D. Berreman [Option ID = 45239]
Correct Answer :-
G.D. Berreman [Option ID = 45239]
Who proposed the ?Three Age System? in pre-history?
[Question ID = 18883]
1. C. J. Thomsen [Option ID = 45525]
2. B. M. Fagan [Option ID = 45523]
3. H.D.Sankalia [Option ID = 45524]
4. F. Borde [Option ID = 45526]
Correct Answer :-
C. J. Thomsen [Option ID = 45525]
Who among the following worked on the complicity between colonial agencies and Anthropologists?
31)
32)
33)
34)
35)
36)
[Question ID = 18801]
1. Ibn Khaldun [Option ID = 45196]
2. Akbar Ahmed [Option ID = 45197]
3. Ralf Linto [Option ID = 45198]
4. Talal Asad [Option ID = 45195]
Correct Answer :-
Talal Asad [Option ID = 45195]
Who is the author of the book ?The Old Stone Age??
[Question ID = 18854]
1. M.C. Burkitt [Option ID = 45407]
2. J. N. Pal [Option ID = 45408]
3. P.C. Pant [Option ID = 45409]
4. H. De Lumley [Option ID = 45410]
Correct Answer :-
M.C. Burkitt [Option ID = 45407]
Who included the Proto-Australoid racial element in the classification of Indian races?
[Question ID = 18866]
1. S.S. Sarkar [Option ID = 45456]
2. H.H. Risley [Option ID = 45455]
3. B.S. Guha [Option ID = 45458]
4. A.C. Haddon [Option ID = 45457]
Correct Answer :-
B.S. Guha [Option ID = 45458]
In which State is the site Isampur located?
[Question ID = 18815]
1. Karnataka [Option ID = 45253]
2. Uttar Pradesh [Option ID = 45254]
3. Madhya Pradesh [Option ID = 45252]
4. Bihar [Option ID = 45251]
Correct Answer :-
Karnataka [Option ID = 45253]
The view of the skull from above is [Question ID = 18800]
1. Norma Basalis [Option ID = 45193]
2. Norma Occipitalis [Option ID = 45194]
3. Norma Frontalis [Option ID = 45192]
4. Norma Verticalis [Option ID = 45191]
Correct Answer :-
Norma Verticalis [Option ID = 45191]
?Childhood? is peculiar to [Question ID = 18789]
1. Nonhuman primates [Option ID = 45149]
2. Human and nonhuman primates [Option ID = 45147]
3. Humans [Option ID = 45148]
4. Mammals [Option ID = 45150]
Correct Answer :-
Humans [Option ID = 45148]
Children subjected to starvation for short periods may recover completely on getting adequate diet. This is referred to as
37)
38)
39)
40)
41)
[Question ID = 18838]
1. Growth spurt [Option ID = 45346]
2. Catch down growth [Option ID = 45345]
3. Catch-up growth [Option ID = 45343]
4. Canalization [Option ID = 45344]
Correct Answer :-
Catch-up growth [Option ID = 45343]
Attirampakkam is [Question ID = 18805]
1. Lower Paleolithic site [Option ID = 45211]
2. Mesolithic site [Option ID = 45214]
3. Indus Valley Civilization site [Option ID = 45213]
4. Upper Paleolithic site [Option ID = 45212]
Correct Answer :-
Lower Paleolithic site [Option ID = 45211]
Shivering
[Question ID = 18828]
1. Has no effect on body heat [Option ID = 45306]
2. Indirectly produces more body heat [Option ID = 45304]
3. Speeds up the loss of body heat [Option ID = 45303]
4. Delays up the loss of body heat [Option ID = 45305]
Correct Answer :-
Indirectly produces more body heat [Option ID = 45304]
A deficiency disease of the skeletal system caused by a lack of Vitamin D or Calcium or both, and often resulting in bone deformities is
[Question ID = 18849]
1. Arthritis [Option ID = 45390]
2. Scurvy [Option ID = 45387]
3. Rickets [Option ID = 45389]
4. Osteoporosis [Option ID = 45388]
Correct Answer :-
Rickets [Option ID = 45389]
A deficiency disease caused due to severe lack of protein in diet among children is
[Question ID = 18856]
1. Anaemia [Option ID = 45418]
2. Kwashiorkor [Option ID = 45417]
3. Scurvy [Option ID = 45416]
4. Rickets [Option ID = 45415]
Correct Answer :-
Kwashiorkor [Option ID = 45417]
Samples of Dendrochronology is taken from which of the following?
[Question ID = 18873]
1. Lake [Option ID = 45483]
2. Tree [Option ID = 45486]
3. River [Option ID = 45484]
4. sea [Option ID = 45485]
Correct Answer :-
Tree [Option ID = 45486]
42)
43)
44)
45)
46)
47)
48)
Swiddening is a form of
[Question ID = 18822]
1. terrace cultivation [Option ID = 45281]
2. intensive cultivation [Option ID = 45280]
3. commercial cultivation [Option ID = 45282]
4. slash and burn cultivation [Option ID = 45279]
Correct Answer :-
slash and burn cultivation [Option ID = 45279]
Osteology is the study of [Question ID = 18810]
1. The human skeleton [Option ID = 45234]
2. The human brain [Option ID = 45233]
3. Human survival strategies [Option ID = 45231]
4. Human languages [Option ID = 45232]
Correct Answer :-
The human skeleton [Option ID = 45234]
Extended Case Study method in anthropology is associated with the work of [Question ID = 18813]
1. Paul Radin [Option ID = 45246]
2. Julian Steward [Option ID = 45244]
3. Anthony Walker [Option ID = 45245]
4. Max Gluckman [Option ID = 45243]
Correct Answer :-
Max Gluckman [Option ID = 45243]
Celts are prepared by
[Question ID = 18863]
1. Levalloisian technique [Option ID = 45443]
2. Clactonian technique [Option ID = 45444]
3. Pressure technique [Option ID = 45446]
4. Grinding and polishing technique [Option ID = 45445]
Correct Answer :-
Grinding and polishing technique [Option ID = 45445]
Khallu is a form : [Question ID = 18792]
1. Fishing [Option ID = 45160]
2. Hunting [Option ID = 45159]
3. Cropping [Option ID = 45161]
4. Weaving [Option ID = 45162]
Correct Answer :-
Cropping [Option ID = 45161]
Lascaux is
[Question ID = 18844]
1. Cave art site [Option ID = 45367]
2. Mesolithic site [Option ID = 45368]
3. Chalcolithic site [Option ID = 45369]
4. Neolithic site [Option ID = 45370]
Correct Answer :-
Cave art site [Option ID = 45367]
A social fact is not characterized by
[Question ID = 18862]
49)
50)
51)
52)
53)
54)
1. Inheritance [Option ID = 45441]
2. Independent existence [Option ID = 45439]
3. Constraints [Option ID = 45442]
4. Exteriority [Option ID = 45440]
Correct Answer :-
Inheritance [Option ID = 45441]
Chopper and Chopping tool are
[Question ID = 18864]
1. blade tool [Option ID = 45447]
2. flake tool [Option ID = 45449]
3. core tool [Option ID = 45448]
4. Neolithic tools [Option ID = 45450]
Correct Answer :-
core tool [Option ID = 45448]
The theory of evolution by natural selection was independently developed by: [Question ID = 19246]
1. Darwin and Lamarck [Option ID = 46977]
2. Darwin and Wallace [Option ID = 46976]
3. Lyell and Darwin [Option ID = 46975]
4. Lyell and Hutton [Option ID = 46978]
Correct Answer :-
Darwin and Lamarck [Option ID = 46977]
? Linea aspera? responsible for the erect posture in man is present in
[Question ID = 18846]
1. Anterior side of tibia [Option ID = 45376]
2. Anterior side of femur [Option ID = 45378]
3. Posterior side of femur [Option ID = 45375]
4. Medial side of fibula [Option ID = 45377]
Correct Answer :-
Posterior side of femur [Option ID = 45375]
?Purity and Danger? is associated with the work of
[Question ID = 18842]
1. M.N. Srinivas [Option ID = 45362]
2. Mary Douglas [Option ID = 45360]
3. Victor Turner [Option ID = 45359]
4. Edmund R. Leach [Option ID = 45361]
Correct Answer :-
Mary Douglas [Option ID = 45360]
?Swaddling Hypothesis? was used by the
[Question ID = 18832]
1. Evolutionist [Option ID = 45320]
2. Conflict Theorist [Option ID = 45321]
3. Cultural Ecologist [Option ID = 45319]
4. Cultural Personality Thinker [Option ID = 45322]
Correct Answer :-
Cultural Personality Thinker [Option ID = 45322]
Trapeze is [Question ID = 18806]
55)
56)
57)
58)
59)
60)
1. Neolithic tool [Option ID = 45215]
2. Core tool [Option ID = 45216]
3. Flake tool [Option ID = 45217]
4. Microlithic tool [Option ID = 45218]
Correct Answer :-
Microlithic tool [Option ID = 45218]
?Melanin? pigment is produced in our [Question ID = 18799]
1. Blood [Option ID = 45188]
2. Pancreas [Option ID = 45190]
3. Liver [Option ID = 45189]
4. Skin [Option ID = 45187]
Correct Answer :-
Skin [Option ID = 45187]
What is the earliest date of Indian Lower Paleolithic culture?
[Question ID = 18825]
1. 2.4 mya [Option ID = 45294]
2. 1.8 mya [Option ID = 45292]
3. 1.51 mya [Option ID = 45293]
4. 1.71 mya [Option ID = 45291]
Correct Answer :-
1.51 mya [Option ID = 45293]
Mutation Theory was proposed by:
[Question ID = 18865]
1. Hugo de Vries [Option ID = 45451]
2. Gregor Mendel. [Option ID = 45454]
3. August Weismann [Option ID = 45453]
4. R. A. Fischer [Option ID = 45452]
Correct Answer :-
Hugo de Vries [Option ID = 45451]
Gonadotropins are responsible for the growth of the-
[Question ID = 18839]
1. Whole body [Option ID = 45350]
2. Brain [Option ID = 45347]
3. Ovaries and testis [Option ID = 45348]
4. Immune system [Option ID = 45349]
Correct Answer :-
Ovaries and testis [Option ID = 45348]
Attempts to account for facts by means of general hypothesis or research questions is known as: [Question ID = 18794]
1. Phenomenology [Option ID = 45169]
2. Reduction [Option ID = 45170]
3. Induction [Option ID = 45167]
4. Deduction [Option ID = 45168]
Correct Answer :-
Deduction [Option ID = 45168]
Someone experiencing physiological problems as a result of oxygen deprivation is
[Question ID = 18818]
61)
62)
63)
64)
65)
1. Polycythaemia [Option ID = 45263]
2. Hyperoxia [Option ID = 45266]
3. Barometric excess [Option ID = 45264]
4. Hypoxia [Option ID = 45265]
Correct Answer :-
Hypoxia [Option ID = 45265]
Who gave the concept of ?the survival of the fittest?? [Question ID = 18808]
1. Hugo De Vries [Option ID = 45225]
2. Charles Darwin [Option ID = 45223]
3. Carl Correns [Option ID = 45226]
4. Herbert Spencer [Option ID = 45224]
Correct Answer :-
Herbert Spencer [Option ID = 45224]
Prepared core technique is also known as
[Question ID = 18845]
1. Punching technique [Option ID = 45374]
2. Levalloisian technique [Option ID = 45373]
3. Clactonian technique [Option ID = 45371]
4. Cylinder Hammer technique [Option ID = 45372]
Correct Answer :-
Levalloisian technique [Option ID = 45373]
A syndrome characterised by physiological, behavioural and mental defects that are due to the presence of an extra copy of the
chromosome 21 is
[Question ID = 18848]
1. Klinefelter syndrome [Option ID = 45385]
2. Marfan syndrome [Option ID = 45386]
3. Turner syndrome [Option ID = 45383]
4. Down syndrome [Option ID = 45384]
Correct Answer :-
Down syndrome [Option ID = 45384]
The statement "Animals are good to think with" is from
[Question ID = 18804]
1. Sherry B. Ortner [Option ID = 45209]
2. Margaret Mead [Option ID = 45210]
3. Claude L?vi Strauss [Option ID = 45208]
4. Mellisa Liewyn-Davies [Option ID = 45207]
Correct Answer :-
Claude L?vi Strauss [Option ID = 45208]
The physiological process of being or becoming accustomed to a new physical environment is known as:
[Question ID = 18877]
1. Alteration [Option ID = 45502]
2. Adaptation [Option ID = 45501]
3. Acclimatization [Option ID = 45500]
4. Assimilation [Option ID = 45499]
Correct Answer :-
Acclimatization [Option ID = 45500]
66)
67)
68)
69)
70)
71)
The famous ?Fox Project? was guided by
[Question ID = 18821]
1. Marvin Harris. [Option ID = 45278]
2. Sol Tax [Option ID = 45275]
3. George P. Murdock [Option ID = 45276]
4. Margaret Mead [Option ID = 45277]
Correct Answer :-
Sol Tax [Option ID = 45275]
The first Director of Anthropological Survey of India was
[Question ID = 18851]
1. S.C. Roy [Option ID = 45396]
2. N. K. Bose. [Option ID = 45395]
3. B.S. Guha [Option ID = 45398]
4. G.S.Ghurye [Option ID = 45397]
Correct Answer :-
B.S. Guha [Option ID = 45398]
The Cross Cultural Cumulative Coding Center (The 5 C?s ) has been developed by
[Question ID = 18833]
1. Ruth Benedict [Option ID = 45323]
2. Franz Boas. [Option ID = 45324]
3. G. P. Murdock [Option ID = 45326]
4. Jack Goody [Option ID = 45325]
Correct Answer :-
G. P. Murdock [Option ID = 45326]
The advisory committee for the revision of the list of Scheduled Castes and Scheduled Tribal is also known as
[Question ID = 18861]
1. Ghurye committee [Option ID = 45438]
2. Shilu Ao committee [Option ID = 45437]
3. Elwin committee [Option ID = 45436]
4. Lokur committee [Option ID = 45435]
Correct Answer :-
Lokur committee [Option ID = 45435]
The period referring to the process of growth from birth up to 28 days of life is-
[Question ID = 18837]
1. Postnatal [Option ID = 45339]
2. Infancy [Option ID = 45340]
3. Neo-natal [Option ID = 45342]
4. Prenatal [Option ID = 45341]
Correct Answer :-
Neo-natal [Option ID = 45342]
The book ?Homo Hierarchicus? is authored by
[Question ID = 18791]
1. Louis Dumont [Option ID = 45155]
2. M.N.Srinivas [Option ID = 45156]
3. G.S. Ghurye [Option ID = 45158]
4. Gerald Berreman [Option ID = 45157]
72)
73)
74)
75)
76)
77)
Correct Answer :-
Louis Dumont [Option ID = 45155]
The study of methods under social control directed towards improving the hereditary constitution of future human generations is
[Question ID = 18847]
1. Ergonomics [Option ID = 45382]
2. Eugenics [Option ID = 45380]
3. Genetics [Option ID = 45379]
4. Physiology [Option ID = 45381]
Correct Answer :-
Eugenics [Option ID = 45380]
The energy giving foods which include starch of cereals and sugar from sugarcane and fruits are-
[Question ID = 18829]
1. Vitamins [Option ID = 45310]
2. Proteins [Option ID = 45308]
3. Carbohydrates [Option ID = 45307]
4. Minerals [Option ID = 45309]
Correct Answer :-
Carbohydrates [Option ID = 45307]
The mixture of homozygous and heterozygous genes of a population in a state of equilibrium is called:
[Question ID = 18878]
1. Multiple alleles [Option ID = 45505]
2. Transient polymorphism [Option ID = 45504]
3. Balanced polymorphism [Option ID = 45503]
4. Genetic equilibrium. [Option ID = 45506]
Correct Answer :-
Balanced polymorphism [Option ID = 45503]
The classical definition of culture by E. B. Tylor appeared in the year
[Question ID = 18830]
1. 1871 [Option ID = 45313]
2. 1879 [Option ID = 45311]
3. 1870 [Option ID = 45312]
4. 1872 [Option ID = 45314]
Correct Answer :-
1871 [Option ID = 45313]
The Fifth Schedule of the Constitution of India deals with
[Question ID = 18811]
1. Economic development and educational opportunities for Scheduled Tribes [Option ID = 45238]
2. Atrocities and crime against Scheduled Tribes [Option ID = 45237]
3. Administration of the Scheduled Areas [Option ID = 45236]
4. Reservation for the Scheduled Tribes [Option ID = 45235]
Correct Answer :-
Administration of the Scheduled Areas [Option ID = 45236]
The idea of ?original affluent society? for hunting and gathering subsistence was proposed by
[Question ID = 18840]
78)
79)
80)
81)
82)
1. Leslie A. White [Option ID = 45351]
2. Marshall D. Sahlins [Option ID = 45354]
3. Roy Rappaport [Option ID = 45353]
4. Julian Steward [Option ID = 45352]
Correct Answer :-
Marshall D. Sahlins [Option ID = 45354]
The concept ?Tribal-Rajput continuum? was given by
[Question ID = 18841]
1. S.S. Sarkar [Option ID = 45357]
2. L. P. Vidyarthi [Option ID = 45356]
3. B. S. Guha [Option ID = 45358]
4. Surjit Sinha [Option ID = 45355]
Correct Answer :-
Surjit Sinha [Option ID = 45355]
The concept ?modal personality? was popularized by
[Question ID = 18831]
1. Abram Kardiner [Option ID = 45316]
2. Ruth Benedict [Option ID = 45317]
3. Geoffrey Gorer [Option ID = 45318]
4. Cora Dubois [Option ID = 45315]
Correct Answer :-
Abram Kardiner [Option ID = 45316]
The concept of ?social network? was first used by
[Question ID = 18860]
1. A. L. Kroeber [Option ID = 45432]
2. G. P. Murdock [Option ID = 45434]
3. John Barnes [Option ID = 45431]
4. Kluckhohn [Option ID = 45433]
Correct Answer :-
John Barnes [Option ID = 45431]
The concept of ?complimentary filiations? was given by
[Question ID = 18881]
1. Raymond Firth [Option ID = 45518]
2. Evans-Pritchard [Option ID = 45516]
3. Radcliffe-Brown [Option ID = 45517]
4. Meyer Fortes [Option ID = 45515]
Correct Answer :-
Meyer Fortes [Option ID = 45515]
The concept of folk-urban continuum was proposed by
[Question ID = 18852]
1. Teodor Shanin [Option ID = 45400]
2. Eric R. Wolf [Option ID = 45399]
3. Milton Singer [Option ID = 45402]
4. Robert Redfield [Option ID = 45401]
Correct Answer :-
Robert Redfield [Option ID = 45401]
83)
84)
85)
86)
87)
88)
The distinction between general and specific evolution was given by
[Question ID = 18872]
1. Leslie White [Option ID = 45479]
2. Elman Service [Option ID = 45480]
3. Marshall Sahlins [Option ID = 45481]
4. Julian Steward [Option ID = 45482]
Correct Answer :-
Elman Service [Option ID = 45480]
The first Alpine glaciation is known as
[Question ID = 18874]
1. Gunz [Option ID = 45490]
2. Wurm [Option ID = 45489]
3. Riss [Option ID = 45488]
4. Mindel [Option ID = 45487]
Correct Answer :-
Gunz [Option ID = 45490]
The teaching of anthropology first began at
[Question ID = 18843]
1. University of Calcutta [Option ID = 45364]
2. University of Madras [Option ID = 45363]
3. University of Delhi [Option ID = 45365]
4. University of Lucknow [Option ID = 45366]
Correct Answer :-
University of Calcutta [Option ID = 45364]
The Blade tools were prepared by
[Question ID = 18855]
1. Punching technique [Option ID = 45413]
2. Stone hammer technique [Option ID = 45412]
3. Clactonian technique [Option ID = 45414]
4. Cylinder hammer technique [Option ID = 45411]
Correct Answer :-
Punching technique [Option ID = 45413]
The term Gemeinschaft refers to
[Question ID = 18793]
1. Association [Option ID = 45165]
2. Institution [Option ID = 45166]
3. Society [Option ID = 45164]
4. Community [Option ID = 45163]
Correct Answer :-
Community [Option ID = 45163]
The average cranial capacity of Homo erectus ranges from:
[Question ID = 18868]
1. 895-1040cc [Option ID = 45463]
2. 1200-1350cc. [Option ID = 45466]
3. 1120-1310cc [Option ID = 45465]
4. 1050-1200cc [Option ID = 45464]
FirstRanker.com - FirstRanker's Choice
1)
2)
3)
4)
5)
DU MSc Anthropology
Topic:- DU_J18_MSC_ANTHRO
Increased labour investment in agriculture with no increase in per capita productivity is called as
[Question ID = 18814]
1. Agricultural recession [Option ID = 45250]
2. Agricultural stagnation [Option ID = 45247]
3. Agricultural relegation [Option ID = 45248]
4. Agricultural involution [Option ID = 45249]
Correct Answer :-
Agricultural involution [Option ID = 45249]
Which of the following is a hominid?
[Question ID = 18857]
1. Chimpanzee [Option ID = 45421]
2. Human [Option ID = 45422]
3. Macaque [Option ID = 45419]
4. Baboon [Option ID = 45420]
Correct Answer :-
Human [Option ID = 45422]
Which rule says that the best suited human body for a hot, tropical region would be one with long limbs and short trunk?
[Question ID = 18827]
1. Allen?s rule [Option ID = 45299]
2. Thompson?s rule [Option ID = 45302]
3. Gloger?s rule [Option ID = 45301]
4. Bergman?s rule [Option ID = 45300]
Correct Answer :-
Allen?s rule [Option ID = 45299]
Which Australopithecus fossil was nicknamed as Lucy?
[Question ID = 18867]
1. Australopithecus africanus [Option ID = 45459]
2. Australopithecus afarensis [Option ID = 45462]
3. Australopithecus anamensis [Option ID = 45460]
4. Australopithecus robustus [Option ID = 45461]
Correct Answer :-
Australopithecus afarensis [Option ID = 45462]
Which among the following does not constitute the elementary form of exchange?
[Question ID = 18820]
1. Exchange of goods [Option ID = 45274]
2. Exchange of women [Option ID = 45271]
3. Exchange of money [Option ID = 45272]
4. Exchange of messages [Option ID = 45273]
Correct Answer :-
Exchange of goods [Option ID = 45274]
6)
7)
8)
9)
10)
11)
12)
Which prehistoric site is known as Mode I Industry?
[Question ID = 18835]
1. Pirro Nord [Option ID = 45333]
2. Hunsgi [Option ID = 45331]
3. Tautavel [Option ID = 45332]
4. Somme valley [Option ID = 45334]
Correct Answer :-
Pirro Nord [Option ID = 45333]
Any quantitative increase in body size is known as [Question ID = 18788]
1. Development [Option ID = 45144]
2. Maturation [Option ID = 45146]
3. Differentiation [Option ID = 45145]
4. Growth [Option ID = 45143]
Correct Answer :-
Growth [Option ID = 45143]
The highest point on the head is known as [Question ID = 18807]
1. Opisthocranion [Option ID = 45222]
2. Bregma [Option ID = 45220]
3. Vertex [Option ID = 45221]
4. Lambda [Option ID = 45219]
Correct Answer :-
Vertex [Option ID = 45221]
Autosomes are
[Question ID = 18826]
1. Normal sex chromosomes [Option ID = 45296]
2. Abnormal sex chromosomes [Option ID = 45298]
3. Abnormal chromosomes [Option ID = 45297]
4. All chromosomes other than the sex chromosomes [Option ID = 45295]
Correct Answer :-
All chromosomes other than the sex chromosomes [Option ID = 45295]
?Puberty spurt? occurs in which one of the following stages of human growth [Question ID = 18798]
1. Infancy [Option ID = 45183]
2. Childhood [Option ID = 45185]
3. Adolescence [Option ID = 45186]
4. Adulthood [Option ID = 45184]
Correct Answer :-
Adolescence [Option ID = 45186]
Potlatch is associated with [Question ID = 18802]
1. Iroquois [Option ID = 45201]
2. Kwakiutl [Option ID = 45199]
3. Shoshonean [Option ID = 45202]
4. Arapesh [Option ID = 45200]
Correct Answer :-
Kwakiutl [Option ID = 45199]
Oldowan culture is dominated by
[Question ID = 18824]
1. Blade tools [Option ID = 45288]
2. Flake tools [Option ID = 45289]
13)
14)
15)
16)
17)
18)
3. Pebble tools [Option ID = 45287]
4. Microlithic tools [Option ID = 45290]
Correct Answer :-
Pebble tools [Option ID = 45287]
A higher fortified part of a city is called
[Question ID = 18823]
1. metropolis [Option ID = 45284]
2. megapolis [Option ID = 45285]
3. acropolis [Option ID = 45283]
4. necropolis [Option ID = 45286]
Correct Answer :-
acropolis [Option ID = 45283]
Yale Cambridge expedition to the Sohan Valley was led by
[Question ID = 18795]
1. R. B. Foote and V.D. Krishnaswamy [Option ID = 45171]
2. G. R. Sharma and V.D. Mishra [Option ID = 45174]
3. H. D. Sankalia and B. Subba Rao [Option ID = 45173]
4. H. De. Terra and T. T. Paterson [Option ID = 45172]
Correct Answer :-
H. De. Terra and T. T. Paterson [Option ID = 45172]
Two nuclear families in adjacent generations with one son/husband or daughter/wife who is a member of both the families is called a
[Question ID = 18870]
1. lineal family [Option ID = 45471]
2. supplemented nuclear family [Option ID = 45474]
3. stem family [Option ID = 45473]
4. compound family [Option ID = 45472]
Correct Answer :-
stem family [Option ID = 45473]
The lack, excess or imbalance of nutrients in the diet leading to ill health is known as [Question ID = 18790]
1. Malnutrition [Option ID = 45153]
2. Over nutrition [Option ID = 45151]
3. Under nutrition [Option ID = 45152]
4. Marasmus [Option ID = 45154]
Correct Answer :-
Malnutrition [Option ID = 45153]
Holocene starts around
[Question ID = 18834]
1. 20,000 B.P. [Option ID = 45329]
2. 12,000 B.P. [Option ID = 45328]
3. 14,000 B.P. [Option ID = 45330]
4. 10,000 B.P. [Option ID = 45327]
Correct Answer :-
10,000 B.P. [Option ID = 45327]
According to Levi-Strauss which of the following constitute one of the ?atoms of kinship? relationship?
[Question ID = 18879]
19)
20)
21)
22)
23)
24)
1. mother-son [Option ID = 45507]
2. mother-daughter [Option ID = 45508]
3. father-son [Option ID = 45509]
4. father-daughter [Option ID = 45510]
Correct Answer :-
mother-son [Option ID = 45507]
A karyotype is a:
[Question ID = 18819]
1. Picture of an individual's chromosomes arranged in a standardized way [Option ID = 45269]
2. Type of abnormal chromosome that is associated with Down's syndrome [Option ID = 45268]
3. General term for any type of chromosome [Option ID = 45267]
4. General term for sex chromosome [Option ID = 45270]
Correct Answer :-
Picture of an individual's chromosomes arranged in a standardized way [Option ID = 45269]
Theodicy is an
[Question ID = 18882]
1. attempt to explain theocracy [Option ID = 45521]
2. attempt to explain good [Option ID = 45519]
3. attempt to explain evil [Option ID = 45520]
4. attempt to explain lie and deceit [Option ID = 45522]
Correct Answer :-
attempt to explain evil [Option ID = 45520]
The word ?Somatotype? was coined by [Question ID = 18797]
1. Faulkner [Option ID = 45182]
2. Sheldon [Option ID = 45180]
3. Parnell [Option ID = 45181]
4. Heath and Carter [Option ID = 45179]
Correct Answer :-
Heath and Carter [Option ID = 45179]
Substantivism as an approach in economic anthropology was given by
[Question ID = 18853]
1. A. L. Kroeber [Option ID = 45406]
2. Karl Polanyi [Option ID = 45405]
3. Oscar Lewis [Option ID = 45403]
4. Robert Redfield [Option ID = 62620]
Correct Answer :-
Karl Polanyi [Option ID = 45405]
?Genetics and the Origin of Species? was written by :
[Question ID = 18859]
1. Thomas Malthus. [Option ID = 45430]
2. Theodosius Dobzhansky [Option ID = 45428]
3. Charles Darwin [Option ID = 45427]
4. T. H. Morgan [Option ID = 45429]
Correct Answer :-
Theodosius Dobzhansky [Option ID = 45428]
The ability of the living organisms to survive in a particular ecological set up is called [Question ID = 18809]
25)
26)
27)
28)
29)
30)
1. Acclimation [Option ID = 45228]
2. Acclimatisation [Option ID = 45229]
3. Adaptation [Option ID = 45227]
4. Selection [Option ID = 45230]
Correct Answer :-
Adaptation [Option ID = 45227]
___________is the physiological ability to reproduce an offspring.
[Question ID = 18876]
1. Fecundity [Option ID = 45496]
2. Fertility [Option ID = 45495]
3. Virility [Option ID = 45498]
4. Natality [Option ID = 45497]
Correct Answer :-
Fecundity [Option ID = 45496]
Charles Darwin's ideas concerning the causes of evolution were probably formulated in his mind
[Question ID = 18817]
1. During the late 1880's [Option ID = 45262]
2. During his voyage on H.M.S. Beagle, especially after he reached the Gal?pagos Islands [Option ID = 45261]
3. Before he began his voyage of exploration around the world on H.M.S. Beagle [Option ID = 45260]
4. While he was still a student at Cambridge University [Option ID = 45259]
Correct Answer :-
During his voyage on H.M.S. Beagle, especially after he reached the Gal?pagos Islands [Option ID = 45261]
First Himalayan Glaciation is known as
[Question ID = 18884]
1. Terrace I [Option ID = 45530]
2. Pinjaur [Option ID = 45527]
3. Boulder conglomerate [Option ID = 45529]
4. Tatrot [Option ID = 45528]
Correct Answer :-
Tatrot [Option ID = 45528]
The author of the book Hindus of the Himalayas is [Question ID = 18812]
1. S.C. Dube [Option ID = 45241]
2. D.N. Majumdar [Option ID = 45242]
3. B. K. Roy-Burman [Option ID = 45240]
4. G.D. Berreman [Option ID = 45239]
Correct Answer :-
G.D. Berreman [Option ID = 45239]
Who proposed the ?Three Age System? in pre-history?
[Question ID = 18883]
1. C. J. Thomsen [Option ID = 45525]
2. B. M. Fagan [Option ID = 45523]
3. H.D.Sankalia [Option ID = 45524]
4. F. Borde [Option ID = 45526]
Correct Answer :-
C. J. Thomsen [Option ID = 45525]
Who among the following worked on the complicity between colonial agencies and Anthropologists?
31)
32)
33)
34)
35)
36)
[Question ID = 18801]
1. Ibn Khaldun [Option ID = 45196]
2. Akbar Ahmed [Option ID = 45197]
3. Ralf Linto [Option ID = 45198]
4. Talal Asad [Option ID = 45195]
Correct Answer :-
Talal Asad [Option ID = 45195]
Who is the author of the book ?The Old Stone Age??
[Question ID = 18854]
1. M.C. Burkitt [Option ID = 45407]
2. J. N. Pal [Option ID = 45408]
3. P.C. Pant [Option ID = 45409]
4. H. De Lumley [Option ID = 45410]
Correct Answer :-
M.C. Burkitt [Option ID = 45407]
Who included the Proto-Australoid racial element in the classification of Indian races?
[Question ID = 18866]
1. S.S. Sarkar [Option ID = 45456]
2. H.H. Risley [Option ID = 45455]
3. B.S. Guha [Option ID = 45458]
4. A.C. Haddon [Option ID = 45457]
Correct Answer :-
B.S. Guha [Option ID = 45458]
In which State is the site Isampur located?
[Question ID = 18815]
1. Karnataka [Option ID = 45253]
2. Uttar Pradesh [Option ID = 45254]
3. Madhya Pradesh [Option ID = 45252]
4. Bihar [Option ID = 45251]
Correct Answer :-
Karnataka [Option ID = 45253]
The view of the skull from above is [Question ID = 18800]
1. Norma Basalis [Option ID = 45193]
2. Norma Occipitalis [Option ID = 45194]
3. Norma Frontalis [Option ID = 45192]
4. Norma Verticalis [Option ID = 45191]
Correct Answer :-
Norma Verticalis [Option ID = 45191]
?Childhood? is peculiar to [Question ID = 18789]
1. Nonhuman primates [Option ID = 45149]
2. Human and nonhuman primates [Option ID = 45147]
3. Humans [Option ID = 45148]
4. Mammals [Option ID = 45150]
Correct Answer :-
Humans [Option ID = 45148]
Children subjected to starvation for short periods may recover completely on getting adequate diet. This is referred to as
37)
38)
39)
40)
41)
[Question ID = 18838]
1. Growth spurt [Option ID = 45346]
2. Catch down growth [Option ID = 45345]
3. Catch-up growth [Option ID = 45343]
4. Canalization [Option ID = 45344]
Correct Answer :-
Catch-up growth [Option ID = 45343]
Attirampakkam is [Question ID = 18805]
1. Lower Paleolithic site [Option ID = 45211]
2. Mesolithic site [Option ID = 45214]
3. Indus Valley Civilization site [Option ID = 45213]
4. Upper Paleolithic site [Option ID = 45212]
Correct Answer :-
Lower Paleolithic site [Option ID = 45211]
Shivering
[Question ID = 18828]
1. Has no effect on body heat [Option ID = 45306]
2. Indirectly produces more body heat [Option ID = 45304]
3. Speeds up the loss of body heat [Option ID = 45303]
4. Delays up the loss of body heat [Option ID = 45305]
Correct Answer :-
Indirectly produces more body heat [Option ID = 45304]
A deficiency disease of the skeletal system caused by a lack of Vitamin D or Calcium or both, and often resulting in bone deformities is
[Question ID = 18849]
1. Arthritis [Option ID = 45390]
2. Scurvy [Option ID = 45387]
3. Rickets [Option ID = 45389]
4. Osteoporosis [Option ID = 45388]
Correct Answer :-
Rickets [Option ID = 45389]
A deficiency disease caused due to severe lack of protein in diet among children is
[Question ID = 18856]
1. Anaemia [Option ID = 45418]
2. Kwashiorkor [Option ID = 45417]
3. Scurvy [Option ID = 45416]
4. Rickets [Option ID = 45415]
Correct Answer :-
Kwashiorkor [Option ID = 45417]
Samples of Dendrochronology is taken from which of the following?
[Question ID = 18873]
1. Lake [Option ID = 45483]
2. Tree [Option ID = 45486]
3. River [Option ID = 45484]
4. sea [Option ID = 45485]
Correct Answer :-
Tree [Option ID = 45486]
42)
43)
44)
45)
46)
47)
48)
Swiddening is a form of
[Question ID = 18822]
1. terrace cultivation [Option ID = 45281]
2. intensive cultivation [Option ID = 45280]
3. commercial cultivation [Option ID = 45282]
4. slash and burn cultivation [Option ID = 45279]
Correct Answer :-
slash and burn cultivation [Option ID = 45279]
Osteology is the study of [Question ID = 18810]
1. The human skeleton [Option ID = 45234]
2. The human brain [Option ID = 45233]
3. Human survival strategies [Option ID = 45231]
4. Human languages [Option ID = 45232]
Correct Answer :-
The human skeleton [Option ID = 45234]
Extended Case Study method in anthropology is associated with the work of [Question ID = 18813]
1. Paul Radin [Option ID = 45246]
2. Julian Steward [Option ID = 45244]
3. Anthony Walker [Option ID = 45245]
4. Max Gluckman [Option ID = 45243]
Correct Answer :-
Max Gluckman [Option ID = 45243]
Celts are prepared by
[Question ID = 18863]
1. Levalloisian technique [Option ID = 45443]
2. Clactonian technique [Option ID = 45444]
3. Pressure technique [Option ID = 45446]
4. Grinding and polishing technique [Option ID = 45445]
Correct Answer :-
Grinding and polishing technique [Option ID = 45445]
Khallu is a form : [Question ID = 18792]
1. Fishing [Option ID = 45160]
2. Hunting [Option ID = 45159]
3. Cropping [Option ID = 45161]
4. Weaving [Option ID = 45162]
Correct Answer :-
Cropping [Option ID = 45161]
Lascaux is
[Question ID = 18844]
1. Cave art site [Option ID = 45367]
2. Mesolithic site [Option ID = 45368]
3. Chalcolithic site [Option ID = 45369]
4. Neolithic site [Option ID = 45370]
Correct Answer :-
Cave art site [Option ID = 45367]
A social fact is not characterized by
[Question ID = 18862]
49)
50)
51)
52)
53)
54)
1. Inheritance [Option ID = 45441]
2. Independent existence [Option ID = 45439]
3. Constraints [Option ID = 45442]
4. Exteriority [Option ID = 45440]
Correct Answer :-
Inheritance [Option ID = 45441]
Chopper and Chopping tool are
[Question ID = 18864]
1. blade tool [Option ID = 45447]
2. flake tool [Option ID = 45449]
3. core tool [Option ID = 45448]
4. Neolithic tools [Option ID = 45450]
Correct Answer :-
core tool [Option ID = 45448]
The theory of evolution by natural selection was independently developed by: [Question ID = 19246]
1. Darwin and Lamarck [Option ID = 46977]
2. Darwin and Wallace [Option ID = 46976]
3. Lyell and Darwin [Option ID = 46975]
4. Lyell and Hutton [Option ID = 46978]
Correct Answer :-
Darwin and Lamarck [Option ID = 46977]
? Linea aspera? responsible for the erect posture in man is present in
[Question ID = 18846]
1. Anterior side of tibia [Option ID = 45376]
2. Anterior side of femur [Option ID = 45378]
3. Posterior side of femur [Option ID = 45375]
4. Medial side of fibula [Option ID = 45377]
Correct Answer :-
Posterior side of femur [Option ID = 45375]
?Purity and Danger? is associated with the work of
[Question ID = 18842]
1. M.N. Srinivas [Option ID = 45362]
2. Mary Douglas [Option ID = 45360]
3. Victor Turner [Option ID = 45359]
4. Edmund R. Leach [Option ID = 45361]
Correct Answer :-
Mary Douglas [Option ID = 45360]
?Swaddling Hypothesis? was used by the
[Question ID = 18832]
1. Evolutionist [Option ID = 45320]
2. Conflict Theorist [Option ID = 45321]
3. Cultural Ecologist [Option ID = 45319]
4. Cultural Personality Thinker [Option ID = 45322]
Correct Answer :-
Cultural Personality Thinker [Option ID = 45322]
Trapeze is [Question ID = 18806]
55)
56)
57)
58)
59)
60)
1. Neolithic tool [Option ID = 45215]
2. Core tool [Option ID = 45216]
3. Flake tool [Option ID = 45217]
4. Microlithic tool [Option ID = 45218]
Correct Answer :-
Microlithic tool [Option ID = 45218]
?Melanin? pigment is produced in our [Question ID = 18799]
1. Blood [Option ID = 45188]
2. Pancreas [Option ID = 45190]
3. Liver [Option ID = 45189]
4. Skin [Option ID = 45187]
Correct Answer :-
Skin [Option ID = 45187]
What is the earliest date of Indian Lower Paleolithic culture?
[Question ID = 18825]
1. 2.4 mya [Option ID = 45294]
2. 1.8 mya [Option ID = 45292]
3. 1.51 mya [Option ID = 45293]
4. 1.71 mya [Option ID = 45291]
Correct Answer :-
1.51 mya [Option ID = 45293]
Mutation Theory was proposed by:
[Question ID = 18865]
1. Hugo de Vries [Option ID = 45451]
2. Gregor Mendel. [Option ID = 45454]
3. August Weismann [Option ID = 45453]
4. R. A. Fischer [Option ID = 45452]
Correct Answer :-
Hugo de Vries [Option ID = 45451]
Gonadotropins are responsible for the growth of the-
[Question ID = 18839]
1. Whole body [Option ID = 45350]
2. Brain [Option ID = 45347]
3. Ovaries and testis [Option ID = 45348]
4. Immune system [Option ID = 45349]
Correct Answer :-
Ovaries and testis [Option ID = 45348]
Attempts to account for facts by means of general hypothesis or research questions is known as: [Question ID = 18794]
1. Phenomenology [Option ID = 45169]
2. Reduction [Option ID = 45170]
3. Induction [Option ID = 45167]
4. Deduction [Option ID = 45168]
Correct Answer :-
Deduction [Option ID = 45168]
Someone experiencing physiological problems as a result of oxygen deprivation is
[Question ID = 18818]
61)
62)
63)
64)
65)
1. Polycythaemia [Option ID = 45263]
2. Hyperoxia [Option ID = 45266]
3. Barometric excess [Option ID = 45264]
4. Hypoxia [Option ID = 45265]
Correct Answer :-
Hypoxia [Option ID = 45265]
Who gave the concept of ?the survival of the fittest?? [Question ID = 18808]
1. Hugo De Vries [Option ID = 45225]
2. Charles Darwin [Option ID = 45223]
3. Carl Correns [Option ID = 45226]
4. Herbert Spencer [Option ID = 45224]
Correct Answer :-
Herbert Spencer [Option ID = 45224]
Prepared core technique is also known as
[Question ID = 18845]
1. Punching technique [Option ID = 45374]
2. Levalloisian technique [Option ID = 45373]
3. Clactonian technique [Option ID = 45371]
4. Cylinder Hammer technique [Option ID = 45372]
Correct Answer :-
Levalloisian technique [Option ID = 45373]
A syndrome characterised by physiological, behavioural and mental defects that are due to the presence of an extra copy of the
chromosome 21 is
[Question ID = 18848]
1. Klinefelter syndrome [Option ID = 45385]
2. Marfan syndrome [Option ID = 45386]
3. Turner syndrome [Option ID = 45383]
4. Down syndrome [Option ID = 45384]
Correct Answer :-
Down syndrome [Option ID = 45384]
The statement "Animals are good to think with" is from
[Question ID = 18804]
1. Sherry B. Ortner [Option ID = 45209]
2. Margaret Mead [Option ID = 45210]
3. Claude L?vi Strauss [Option ID = 45208]
4. Mellisa Liewyn-Davies [Option ID = 45207]
Correct Answer :-
Claude L?vi Strauss [Option ID = 45208]
The physiological process of being or becoming accustomed to a new physical environment is known as:
[Question ID = 18877]
1. Alteration [Option ID = 45502]
2. Adaptation [Option ID = 45501]
3. Acclimatization [Option ID = 45500]
4. Assimilation [Option ID = 45499]
Correct Answer :-
Acclimatization [Option ID = 45500]
66)
67)
68)
69)
70)
71)
The famous ?Fox Project? was guided by
[Question ID = 18821]
1. Marvin Harris. [Option ID = 45278]
2. Sol Tax [Option ID = 45275]
3. George P. Murdock [Option ID = 45276]
4. Margaret Mead [Option ID = 45277]
Correct Answer :-
Sol Tax [Option ID = 45275]
The first Director of Anthropological Survey of India was
[Question ID = 18851]
1. S.C. Roy [Option ID = 45396]
2. N. K. Bose. [Option ID = 45395]
3. B.S. Guha [Option ID = 45398]
4. G.S.Ghurye [Option ID = 45397]
Correct Answer :-
B.S. Guha [Option ID = 45398]
The Cross Cultural Cumulative Coding Center (The 5 C?s ) has been developed by
[Question ID = 18833]
1. Ruth Benedict [Option ID = 45323]
2. Franz Boas. [Option ID = 45324]
3. G. P. Murdock [Option ID = 45326]
4. Jack Goody [Option ID = 45325]
Correct Answer :-
G. P. Murdock [Option ID = 45326]
The advisory committee for the revision of the list of Scheduled Castes and Scheduled Tribal is also known as
[Question ID = 18861]
1. Ghurye committee [Option ID = 45438]
2. Shilu Ao committee [Option ID = 45437]
3. Elwin committee [Option ID = 45436]
4. Lokur committee [Option ID = 45435]
Correct Answer :-
Lokur committee [Option ID = 45435]
The period referring to the process of growth from birth up to 28 days of life is-
[Question ID = 18837]
1. Postnatal [Option ID = 45339]
2. Infancy [Option ID = 45340]
3. Neo-natal [Option ID = 45342]
4. Prenatal [Option ID = 45341]
Correct Answer :-
Neo-natal [Option ID = 45342]
The book ?Homo Hierarchicus? is authored by
[Question ID = 18791]
1. Louis Dumont [Option ID = 45155]
2. M.N.Srinivas [Option ID = 45156]
3. G.S. Ghurye [Option ID = 45158]
4. Gerald Berreman [Option ID = 45157]
72)
73)
74)
75)
76)
77)
Correct Answer :-
Louis Dumont [Option ID = 45155]
The study of methods under social control directed towards improving the hereditary constitution of future human generations is
[Question ID = 18847]
1. Ergonomics [Option ID = 45382]
2. Eugenics [Option ID = 45380]
3. Genetics [Option ID = 45379]
4. Physiology [Option ID = 45381]
Correct Answer :-
Eugenics [Option ID = 45380]
The energy giving foods which include starch of cereals and sugar from sugarcane and fruits are-
[Question ID = 18829]
1. Vitamins [Option ID = 45310]
2. Proteins [Option ID = 45308]
3. Carbohydrates [Option ID = 45307]
4. Minerals [Option ID = 45309]
Correct Answer :-
Carbohydrates [Option ID = 45307]
The mixture of homozygous and heterozygous genes of a population in a state of equilibrium is called:
[Question ID = 18878]
1. Multiple alleles [Option ID = 45505]
2. Transient polymorphism [Option ID = 45504]
3. Balanced polymorphism [Option ID = 45503]
4. Genetic equilibrium. [Option ID = 45506]
Correct Answer :-
Balanced polymorphism [Option ID = 45503]
The classical definition of culture by E. B. Tylor appeared in the year
[Question ID = 18830]
1. 1871 [Option ID = 45313]
2. 1879 [Option ID = 45311]
3. 1870 [Option ID = 45312]
4. 1872 [Option ID = 45314]
Correct Answer :-
1871 [Option ID = 45313]
The Fifth Schedule of the Constitution of India deals with
[Question ID = 18811]
1. Economic development and educational opportunities for Scheduled Tribes [Option ID = 45238]
2. Atrocities and crime against Scheduled Tribes [Option ID = 45237]
3. Administration of the Scheduled Areas [Option ID = 45236]
4. Reservation for the Scheduled Tribes [Option ID = 45235]
Correct Answer :-
Administration of the Scheduled Areas [Option ID = 45236]
The idea of ?original affluent society? for hunting and gathering subsistence was proposed by
[Question ID = 18840]
78)
79)
80)
81)
82)
1. Leslie A. White [Option ID = 45351]
2. Marshall D. Sahlins [Option ID = 45354]
3. Roy Rappaport [Option ID = 45353]
4. Julian Steward [Option ID = 45352]
Correct Answer :-
Marshall D. Sahlins [Option ID = 45354]
The concept ?Tribal-Rajput continuum? was given by
[Question ID = 18841]
1. S.S. Sarkar [Option ID = 45357]
2. L. P. Vidyarthi [Option ID = 45356]
3. B. S. Guha [Option ID = 45358]
4. Surjit Sinha [Option ID = 45355]
Correct Answer :-
Surjit Sinha [Option ID = 45355]
The concept ?modal personality? was popularized by
[Question ID = 18831]
1. Abram Kardiner [Option ID = 45316]
2. Ruth Benedict [Option ID = 45317]
3. Geoffrey Gorer [Option ID = 45318]
4. Cora Dubois [Option ID = 45315]
Correct Answer :-
Abram Kardiner [Option ID = 45316]
The concept of ?social network? was first used by
[Question ID = 18860]
1. A. L. Kroeber [Option ID = 45432]
2. G. P. Murdock [Option ID = 45434]
3. John Barnes [Option ID = 45431]
4. Kluckhohn [Option ID = 45433]
Correct Answer :-
John Barnes [Option ID = 45431]
The concept of ?complimentary filiations? was given by
[Question ID = 18881]
1. Raymond Firth [Option ID = 45518]
2. Evans-Pritchard [Option ID = 45516]
3. Radcliffe-Brown [Option ID = 45517]
4. Meyer Fortes [Option ID = 45515]
Correct Answer :-
Meyer Fortes [Option ID = 45515]
The concept of folk-urban continuum was proposed by
[Question ID = 18852]
1. Teodor Shanin [Option ID = 45400]
2. Eric R. Wolf [Option ID = 45399]
3. Milton Singer [Option ID = 45402]
4. Robert Redfield [Option ID = 45401]
Correct Answer :-
Robert Redfield [Option ID = 45401]
83)
84)
85)
86)
87)
88)
The distinction between general and specific evolution was given by
[Question ID = 18872]
1. Leslie White [Option ID = 45479]
2. Elman Service [Option ID = 45480]
3. Marshall Sahlins [Option ID = 45481]
4. Julian Steward [Option ID = 45482]
Correct Answer :-
Elman Service [Option ID = 45480]
The first Alpine glaciation is known as
[Question ID = 18874]
1. Gunz [Option ID = 45490]
2. Wurm [Option ID = 45489]
3. Riss [Option ID = 45488]
4. Mindel [Option ID = 45487]
Correct Answer :-
Gunz [Option ID = 45490]
The teaching of anthropology first began at
[Question ID = 18843]
1. University of Calcutta [Option ID = 45364]
2. University of Madras [Option ID = 45363]
3. University of Delhi [Option ID = 45365]
4. University of Lucknow [Option ID = 45366]
Correct Answer :-
University of Calcutta [Option ID = 45364]
The Blade tools were prepared by
[Question ID = 18855]
1. Punching technique [Option ID = 45413]
2. Stone hammer technique [Option ID = 45412]
3. Clactonian technique [Option ID = 45414]
4. Cylinder hammer technique [Option ID = 45411]
Correct Answer :-
Punching technique [Option ID = 45413]
The term Gemeinschaft refers to
[Question ID = 18793]
1. Association [Option ID = 45165]
2. Institution [Option ID = 45166]
3. Society [Option ID = 45164]
4. Community [Option ID = 45163]
Correct Answer :-
Community [Option ID = 45163]
The average cranial capacity of Homo erectus ranges from:
[Question ID = 18868]
1. 895-1040cc [Option ID = 45463]
2. 1200-1350cc. [Option ID = 45466]
3. 1120-1310cc [Option ID = 45465]
4. 1050-1200cc [Option ID = 45464]
89)
90)
91)
92)
93)
94)
Correct Answer :-
895-1040cc [Option ID = 45463]
The method in which measurements are repeatedly done on the same individual or group or individuals at definite age intervals is-
[Question ID = 18836]
1. Longitudinal study [Option ID = 62613]
2. Cross-sectional study [Option ID = 45335]
3. Semi longitudinal [Option ID = 45338]
4. Mixed longitudinal [Option ID = 45337]
Correct Answer :-
Semi longitudinal [Option ID = 45338]
The author of the book Encounter with Anthropology is:
[Question ID = 18803]
1. Claude L?vi-Strauss [Option ID = 45203]
2. Bronislaw Malinowski [Option ID = 45205]
3. A.R. Radcliffe-Brown [Option ID = 45206]
4. Robin Fox [Option ID = 45204]
Correct Answer :-
Robin Fox [Option ID = 45204]
The term ?habitus? was given by
[Question ID = 18880]
1. Edward Sapir [Option ID = 45513]
2. Margaret Mead [Option ID = 45512]
3. Pierre Bourdieu [Option ID = 45511]
4. Jack Goody [Option ID = 45514]
Correct Answer :-
Pierre Bourdieu [Option ID = 45511]
The term ?thick description? in anthropology is associated with
[Question ID = 18871]
1. Marvin Harris [Option ID = 45478]
2. Clifford Geertz [Option ID = 48535]
3. Levi-Strauss [Option ID = 45475]
4. Roland Barthes [Option ID = 45477]
Correct Answer :-
Clifford Geertz [Option ID = 48535]
The fossil remains of Ramapithecus in India was discovered from the:
[Question ID = 18858]
1. Devagiri Hills [Option ID = 45425]
2. Siwalik Hills [Option ID = 45424]
3. Bateshwar Hills [Option ID = 45426]
4. Nilgiri Hills [Option ID = 45423]
Correct Answer :-
Siwalik Hills [Option ID = 45424]
The communities having matrilineal descent is found in
[Question ID = 18850]
FirstRanker.com - FirstRanker's Choice
1)
2)
3)
4)
5)
DU MSc Anthropology
Topic:- DU_J18_MSC_ANTHRO
Increased labour investment in agriculture with no increase in per capita productivity is called as
[Question ID = 18814]
1. Agricultural recession [Option ID = 45250]
2. Agricultural stagnation [Option ID = 45247]
3. Agricultural relegation [Option ID = 45248]
4. Agricultural involution [Option ID = 45249]
Correct Answer :-
Agricultural involution [Option ID = 45249]
Which of the following is a hominid?
[Question ID = 18857]
1. Chimpanzee [Option ID = 45421]
2. Human [Option ID = 45422]
3. Macaque [Option ID = 45419]
4. Baboon [Option ID = 45420]
Correct Answer :-
Human [Option ID = 45422]
Which rule says that the best suited human body for a hot, tropical region would be one with long limbs and short trunk?
[Question ID = 18827]
1. Allen?s rule [Option ID = 45299]
2. Thompson?s rule [Option ID = 45302]
3. Gloger?s rule [Option ID = 45301]
4. Bergman?s rule [Option ID = 45300]
Correct Answer :-
Allen?s rule [Option ID = 45299]
Which Australopithecus fossil was nicknamed as Lucy?
[Question ID = 18867]
1. Australopithecus africanus [Option ID = 45459]
2. Australopithecus afarensis [Option ID = 45462]
3. Australopithecus anamensis [Option ID = 45460]
4. Australopithecus robustus [Option ID = 45461]
Correct Answer :-
Australopithecus afarensis [Option ID = 45462]
Which among the following does not constitute the elementary form of exchange?
[Question ID = 18820]
1. Exchange of goods [Option ID = 45274]
2. Exchange of women [Option ID = 45271]
3. Exchange of money [Option ID = 45272]
4. Exchange of messages [Option ID = 45273]
Correct Answer :-
Exchange of goods [Option ID = 45274]
6)
7)
8)
9)
10)
11)
12)
Which prehistoric site is known as Mode I Industry?
[Question ID = 18835]
1. Pirro Nord [Option ID = 45333]
2. Hunsgi [Option ID = 45331]
3. Tautavel [Option ID = 45332]
4. Somme valley [Option ID = 45334]
Correct Answer :-
Pirro Nord [Option ID = 45333]
Any quantitative increase in body size is known as [Question ID = 18788]
1. Development [Option ID = 45144]
2. Maturation [Option ID = 45146]
3. Differentiation [Option ID = 45145]
4. Growth [Option ID = 45143]
Correct Answer :-
Growth [Option ID = 45143]
The highest point on the head is known as [Question ID = 18807]
1. Opisthocranion [Option ID = 45222]
2. Bregma [Option ID = 45220]
3. Vertex [Option ID = 45221]
4. Lambda [Option ID = 45219]
Correct Answer :-
Vertex [Option ID = 45221]
Autosomes are
[Question ID = 18826]
1. Normal sex chromosomes [Option ID = 45296]
2. Abnormal sex chromosomes [Option ID = 45298]
3. Abnormal chromosomes [Option ID = 45297]
4. All chromosomes other than the sex chromosomes [Option ID = 45295]
Correct Answer :-
All chromosomes other than the sex chromosomes [Option ID = 45295]
?Puberty spurt? occurs in which one of the following stages of human growth [Question ID = 18798]
1. Infancy [Option ID = 45183]
2. Childhood [Option ID = 45185]
3. Adolescence [Option ID = 45186]
4. Adulthood [Option ID = 45184]
Correct Answer :-
Adolescence [Option ID = 45186]
Potlatch is associated with [Question ID = 18802]
1. Iroquois [Option ID = 45201]
2. Kwakiutl [Option ID = 45199]
3. Shoshonean [Option ID = 45202]
4. Arapesh [Option ID = 45200]
Correct Answer :-
Kwakiutl [Option ID = 45199]
Oldowan culture is dominated by
[Question ID = 18824]
1. Blade tools [Option ID = 45288]
2. Flake tools [Option ID = 45289]
13)
14)
15)
16)
17)
18)
3. Pebble tools [Option ID = 45287]
4. Microlithic tools [Option ID = 45290]
Correct Answer :-
Pebble tools [Option ID = 45287]
A higher fortified part of a city is called
[Question ID = 18823]
1. metropolis [Option ID = 45284]
2. megapolis [Option ID = 45285]
3. acropolis [Option ID = 45283]
4. necropolis [Option ID = 45286]
Correct Answer :-
acropolis [Option ID = 45283]
Yale Cambridge expedition to the Sohan Valley was led by
[Question ID = 18795]
1. R. B. Foote and V.D. Krishnaswamy [Option ID = 45171]
2. G. R. Sharma and V.D. Mishra [Option ID = 45174]
3. H. D. Sankalia and B. Subba Rao [Option ID = 45173]
4. H. De. Terra and T. T. Paterson [Option ID = 45172]
Correct Answer :-
H. De. Terra and T. T. Paterson [Option ID = 45172]
Two nuclear families in adjacent generations with one son/husband or daughter/wife who is a member of both the families is called a
[Question ID = 18870]
1. lineal family [Option ID = 45471]
2. supplemented nuclear family [Option ID = 45474]
3. stem family [Option ID = 45473]
4. compound family [Option ID = 45472]
Correct Answer :-
stem family [Option ID = 45473]
The lack, excess or imbalance of nutrients in the diet leading to ill health is known as [Question ID = 18790]
1. Malnutrition [Option ID = 45153]
2. Over nutrition [Option ID = 45151]
3. Under nutrition [Option ID = 45152]
4. Marasmus [Option ID = 45154]
Correct Answer :-
Malnutrition [Option ID = 45153]
Holocene starts around
[Question ID = 18834]
1. 20,000 B.P. [Option ID = 45329]
2. 12,000 B.P. [Option ID = 45328]
3. 14,000 B.P. [Option ID = 45330]
4. 10,000 B.P. [Option ID = 45327]
Correct Answer :-
10,000 B.P. [Option ID = 45327]
According to Levi-Strauss which of the following constitute one of the ?atoms of kinship? relationship?
[Question ID = 18879]
19)
20)
21)
22)
23)
24)
1. mother-son [Option ID = 45507]
2. mother-daughter [Option ID = 45508]
3. father-son [Option ID = 45509]
4. father-daughter [Option ID = 45510]
Correct Answer :-
mother-son [Option ID = 45507]
A karyotype is a:
[Question ID = 18819]
1. Picture of an individual's chromosomes arranged in a standardized way [Option ID = 45269]
2. Type of abnormal chromosome that is associated with Down's syndrome [Option ID = 45268]
3. General term for any type of chromosome [Option ID = 45267]
4. General term for sex chromosome [Option ID = 45270]
Correct Answer :-
Picture of an individual's chromosomes arranged in a standardized way [Option ID = 45269]
Theodicy is an
[Question ID = 18882]
1. attempt to explain theocracy [Option ID = 45521]
2. attempt to explain good [Option ID = 45519]
3. attempt to explain evil [Option ID = 45520]
4. attempt to explain lie and deceit [Option ID = 45522]
Correct Answer :-
attempt to explain evil [Option ID = 45520]
The word ?Somatotype? was coined by [Question ID = 18797]
1. Faulkner [Option ID = 45182]
2. Sheldon [Option ID = 45180]
3. Parnell [Option ID = 45181]
4. Heath and Carter [Option ID = 45179]
Correct Answer :-
Heath and Carter [Option ID = 45179]
Substantivism as an approach in economic anthropology was given by
[Question ID = 18853]
1. A. L. Kroeber [Option ID = 45406]
2. Karl Polanyi [Option ID = 45405]
3. Oscar Lewis [Option ID = 45403]
4. Robert Redfield [Option ID = 62620]
Correct Answer :-
Karl Polanyi [Option ID = 45405]
?Genetics and the Origin of Species? was written by :
[Question ID = 18859]
1. Thomas Malthus. [Option ID = 45430]
2. Theodosius Dobzhansky [Option ID = 45428]
3. Charles Darwin [Option ID = 45427]
4. T. H. Morgan [Option ID = 45429]
Correct Answer :-
Theodosius Dobzhansky [Option ID = 45428]
The ability of the living organisms to survive in a particular ecological set up is called [Question ID = 18809]
25)
26)
27)
28)
29)
30)
1. Acclimation [Option ID = 45228]
2. Acclimatisation [Option ID = 45229]
3. Adaptation [Option ID = 45227]
4. Selection [Option ID = 45230]
Correct Answer :-
Adaptation [Option ID = 45227]
___________is the physiological ability to reproduce an offspring.
[Question ID = 18876]
1. Fecundity [Option ID = 45496]
2. Fertility [Option ID = 45495]
3. Virility [Option ID = 45498]
4. Natality [Option ID = 45497]
Correct Answer :-
Fecundity [Option ID = 45496]
Charles Darwin's ideas concerning the causes of evolution were probably formulated in his mind
[Question ID = 18817]
1. During the late 1880's [Option ID = 45262]
2. During his voyage on H.M.S. Beagle, especially after he reached the Gal?pagos Islands [Option ID = 45261]
3. Before he began his voyage of exploration around the world on H.M.S. Beagle [Option ID = 45260]
4. While he was still a student at Cambridge University [Option ID = 45259]
Correct Answer :-
During his voyage on H.M.S. Beagle, especially after he reached the Gal?pagos Islands [Option ID = 45261]
First Himalayan Glaciation is known as
[Question ID = 18884]
1. Terrace I [Option ID = 45530]
2. Pinjaur [Option ID = 45527]
3. Boulder conglomerate [Option ID = 45529]
4. Tatrot [Option ID = 45528]
Correct Answer :-
Tatrot [Option ID = 45528]
The author of the book Hindus of the Himalayas is [Question ID = 18812]
1. S.C. Dube [Option ID = 45241]
2. D.N. Majumdar [Option ID = 45242]
3. B. K. Roy-Burman [Option ID = 45240]
4. G.D. Berreman [Option ID = 45239]
Correct Answer :-
G.D. Berreman [Option ID = 45239]
Who proposed the ?Three Age System? in pre-history?
[Question ID = 18883]
1. C. J. Thomsen [Option ID = 45525]
2. B. M. Fagan [Option ID = 45523]
3. H.D.Sankalia [Option ID = 45524]
4. F. Borde [Option ID = 45526]
Correct Answer :-
C. J. Thomsen [Option ID = 45525]
Who among the following worked on the complicity between colonial agencies and Anthropologists?
31)
32)
33)
34)
35)
36)
[Question ID = 18801]
1. Ibn Khaldun [Option ID = 45196]
2. Akbar Ahmed [Option ID = 45197]
3. Ralf Linto [Option ID = 45198]
4. Talal Asad [Option ID = 45195]
Correct Answer :-
Talal Asad [Option ID = 45195]
Who is the author of the book ?The Old Stone Age??
[Question ID = 18854]
1. M.C. Burkitt [Option ID = 45407]
2. J. N. Pal [Option ID = 45408]
3. P.C. Pant [Option ID = 45409]
4. H. De Lumley [Option ID = 45410]
Correct Answer :-
M.C. Burkitt [Option ID = 45407]
Who included the Proto-Australoid racial element in the classification of Indian races?
[Question ID = 18866]
1. S.S. Sarkar [Option ID = 45456]
2. H.H. Risley [Option ID = 45455]
3. B.S. Guha [Option ID = 45458]
4. A.C. Haddon [Option ID = 45457]
Correct Answer :-
B.S. Guha [Option ID = 45458]
In which State is the site Isampur located?
[Question ID = 18815]
1. Karnataka [Option ID = 45253]
2. Uttar Pradesh [Option ID = 45254]
3. Madhya Pradesh [Option ID = 45252]
4. Bihar [Option ID = 45251]
Correct Answer :-
Karnataka [Option ID = 45253]
The view of the skull from above is [Question ID = 18800]
1. Norma Basalis [Option ID = 45193]
2. Norma Occipitalis [Option ID = 45194]
3. Norma Frontalis [Option ID = 45192]
4. Norma Verticalis [Option ID = 45191]
Correct Answer :-
Norma Verticalis [Option ID = 45191]
?Childhood? is peculiar to [Question ID = 18789]
1. Nonhuman primates [Option ID = 45149]
2. Human and nonhuman primates [Option ID = 45147]
3. Humans [Option ID = 45148]
4. Mammals [Option ID = 45150]
Correct Answer :-
Humans [Option ID = 45148]
Children subjected to starvation for short periods may recover completely on getting adequate diet. This is referred to as
37)
38)
39)
40)
41)
[Question ID = 18838]
1. Growth spurt [Option ID = 45346]
2. Catch down growth [Option ID = 45345]
3. Catch-up growth [Option ID = 45343]
4. Canalization [Option ID = 45344]
Correct Answer :-
Catch-up growth [Option ID = 45343]
Attirampakkam is [Question ID = 18805]
1. Lower Paleolithic site [Option ID = 45211]
2. Mesolithic site [Option ID = 45214]
3. Indus Valley Civilization site [Option ID = 45213]
4. Upper Paleolithic site [Option ID = 45212]
Correct Answer :-
Lower Paleolithic site [Option ID = 45211]
Shivering
[Question ID = 18828]
1. Has no effect on body heat [Option ID = 45306]
2. Indirectly produces more body heat [Option ID = 45304]
3. Speeds up the loss of body heat [Option ID = 45303]
4. Delays up the loss of body heat [Option ID = 45305]
Correct Answer :-
Indirectly produces more body heat [Option ID = 45304]
A deficiency disease of the skeletal system caused by a lack of Vitamin D or Calcium or both, and often resulting in bone deformities is
[Question ID = 18849]
1. Arthritis [Option ID = 45390]
2. Scurvy [Option ID = 45387]
3. Rickets [Option ID = 45389]
4. Osteoporosis [Option ID = 45388]
Correct Answer :-
Rickets [Option ID = 45389]
A deficiency disease caused due to severe lack of protein in diet among children is
[Question ID = 18856]
1. Anaemia [Option ID = 45418]
2. Kwashiorkor [Option ID = 45417]
3. Scurvy [Option ID = 45416]
4. Rickets [Option ID = 45415]
Correct Answer :-
Kwashiorkor [Option ID = 45417]
Samples of Dendrochronology is taken from which of the following?
[Question ID = 18873]
1. Lake [Option ID = 45483]
2. Tree [Option ID = 45486]
3. River [Option ID = 45484]
4. sea [Option ID = 45485]
Correct Answer :-
Tree [Option ID = 45486]
42)
43)
44)
45)
46)
47)
48)
Swiddening is a form of
[Question ID = 18822]
1. terrace cultivation [Option ID = 45281]
2. intensive cultivation [Option ID = 45280]
3. commercial cultivation [Option ID = 45282]
4. slash and burn cultivation [Option ID = 45279]
Correct Answer :-
slash and burn cultivation [Option ID = 45279]
Osteology is the study of [Question ID = 18810]
1. The human skeleton [Option ID = 45234]
2. The human brain [Option ID = 45233]
3. Human survival strategies [Option ID = 45231]
4. Human languages [Option ID = 45232]
Correct Answer :-
The human skeleton [Option ID = 45234]
Extended Case Study method in anthropology is associated with the work of [Question ID = 18813]
1. Paul Radin [Option ID = 45246]
2. Julian Steward [Option ID = 45244]
3. Anthony Walker [Option ID = 45245]
4. Max Gluckman [Option ID = 45243]
Correct Answer :-
Max Gluckman [Option ID = 45243]
Celts are prepared by
[Question ID = 18863]
1. Levalloisian technique [Option ID = 45443]
2. Clactonian technique [Option ID = 45444]
3. Pressure technique [Option ID = 45446]
4. Grinding and polishing technique [Option ID = 45445]
Correct Answer :-
Grinding and polishing technique [Option ID = 45445]
Khallu is a form : [Question ID = 18792]
1. Fishing [Option ID = 45160]
2. Hunting [Option ID = 45159]
3. Cropping [Option ID = 45161]
4. Weaving [Option ID = 45162]
Correct Answer :-
Cropping [Option ID = 45161]
Lascaux is
[Question ID = 18844]
1. Cave art site [Option ID = 45367]
2. Mesolithic site [Option ID = 45368]
3. Chalcolithic site [Option ID = 45369]
4. Neolithic site [Option ID = 45370]
Correct Answer :-
Cave art site [Option ID = 45367]
A social fact is not characterized by
[Question ID = 18862]
49)
50)
51)
52)
53)
54)
1. Inheritance [Option ID = 45441]
2. Independent existence [Option ID = 45439]
3. Constraints [Option ID = 45442]
4. Exteriority [Option ID = 45440]
Correct Answer :-
Inheritance [Option ID = 45441]
Chopper and Chopping tool are
[Question ID = 18864]
1. blade tool [Option ID = 45447]
2. flake tool [Option ID = 45449]
3. core tool [Option ID = 45448]
4. Neolithic tools [Option ID = 45450]
Correct Answer :-
core tool [Option ID = 45448]
The theory of evolution by natural selection was independently developed by: [Question ID = 19246]
1. Darwin and Lamarck [Option ID = 46977]
2. Darwin and Wallace [Option ID = 46976]
3. Lyell and Darwin [Option ID = 46975]
4. Lyell and Hutton [Option ID = 46978]
Correct Answer :-
Darwin and Lamarck [Option ID = 46977]
? Linea aspera? responsible for the erect posture in man is present in
[Question ID = 18846]
1. Anterior side of tibia [Option ID = 45376]
2. Anterior side of femur [Option ID = 45378]
3. Posterior side of femur [Option ID = 45375]
4. Medial side of fibula [Option ID = 45377]
Correct Answer :-
Posterior side of femur [Option ID = 45375]
?Purity and Danger? is associated with the work of
[Question ID = 18842]
1. M.N. Srinivas [Option ID = 45362]
2. Mary Douglas [Option ID = 45360]
3. Victor Turner [Option ID = 45359]
4. Edmund R. Leach [Option ID = 45361]
Correct Answer :-
Mary Douglas [Option ID = 45360]
?Swaddling Hypothesis? was used by the
[Question ID = 18832]
1. Evolutionist [Option ID = 45320]
2. Conflict Theorist [Option ID = 45321]
3. Cultural Ecologist [Option ID = 45319]
4. Cultural Personality Thinker [Option ID = 45322]
Correct Answer :-
Cultural Personality Thinker [Option ID = 45322]
Trapeze is [Question ID = 18806]
55)
56)
57)
58)
59)
60)
1. Neolithic tool [Option ID = 45215]
2. Core tool [Option ID = 45216]
3. Flake tool [Option ID = 45217]
4. Microlithic tool [Option ID = 45218]
Correct Answer :-
Microlithic tool [Option ID = 45218]
?Melanin? pigment is produced in our [Question ID = 18799]
1. Blood [Option ID = 45188]
2. Pancreas [Option ID = 45190]
3. Liver [Option ID = 45189]
4. Skin [Option ID = 45187]
Correct Answer :-
Skin [Option ID = 45187]
What is the earliest date of Indian Lower Paleolithic culture?
[Question ID = 18825]
1. 2.4 mya [Option ID = 45294]
2. 1.8 mya [Option ID = 45292]
3. 1.51 mya [Option ID = 45293]
4. 1.71 mya [Option ID = 45291]
Correct Answer :-
1.51 mya [Option ID = 45293]
Mutation Theory was proposed by:
[Question ID = 18865]
1. Hugo de Vries [Option ID = 45451]
2. Gregor Mendel. [Option ID = 45454]
3. August Weismann [Option ID = 45453]
4. R. A. Fischer [Option ID = 45452]
Correct Answer :-
Hugo de Vries [Option ID = 45451]
Gonadotropins are responsible for the growth of the-
[Question ID = 18839]
1. Whole body [Option ID = 45350]
2. Brain [Option ID = 45347]
3. Ovaries and testis [Option ID = 45348]
4. Immune system [Option ID = 45349]
Correct Answer :-
Ovaries and testis [Option ID = 45348]
Attempts to account for facts by means of general hypothesis or research questions is known as: [Question ID = 18794]
1. Phenomenology [Option ID = 45169]
2. Reduction [Option ID = 45170]
3. Induction [Option ID = 45167]
4. Deduction [Option ID = 45168]
Correct Answer :-
Deduction [Option ID = 45168]
Someone experiencing physiological problems as a result of oxygen deprivation is
[Question ID = 18818]
61)
62)
63)
64)
65)
1. Polycythaemia [Option ID = 45263]
2. Hyperoxia [Option ID = 45266]
3. Barometric excess [Option ID = 45264]
4. Hypoxia [Option ID = 45265]
Correct Answer :-
Hypoxia [Option ID = 45265]
Who gave the concept of ?the survival of the fittest?? [Question ID = 18808]
1. Hugo De Vries [Option ID = 45225]
2. Charles Darwin [Option ID = 45223]
3. Carl Correns [Option ID = 45226]
4. Herbert Spencer [Option ID = 45224]
Correct Answer :-
Herbert Spencer [Option ID = 45224]
Prepared core technique is also known as
[Question ID = 18845]
1. Punching technique [Option ID = 45374]
2. Levalloisian technique [Option ID = 45373]
3. Clactonian technique [Option ID = 45371]
4. Cylinder Hammer technique [Option ID = 45372]
Correct Answer :-
Levalloisian technique [Option ID = 45373]
A syndrome characterised by physiological, behavioural and mental defects that are due to the presence of an extra copy of the
chromosome 21 is
[Question ID = 18848]
1. Klinefelter syndrome [Option ID = 45385]
2. Marfan syndrome [Option ID = 45386]
3. Turner syndrome [Option ID = 45383]
4. Down syndrome [Option ID = 45384]
Correct Answer :-
Down syndrome [Option ID = 45384]
The statement "Animals are good to think with" is from
[Question ID = 18804]
1. Sherry B. Ortner [Option ID = 45209]
2. Margaret Mead [Option ID = 45210]
3. Claude L?vi Strauss [Option ID = 45208]
4. Mellisa Liewyn-Davies [Option ID = 45207]
Correct Answer :-
Claude L?vi Strauss [Option ID = 45208]
The physiological process of being or becoming accustomed to a new physical environment is known as:
[Question ID = 18877]
1. Alteration [Option ID = 45502]
2. Adaptation [Option ID = 45501]
3. Acclimatization [Option ID = 45500]
4. Assimilation [Option ID = 45499]
Correct Answer :-
Acclimatization [Option ID = 45500]
66)
67)
68)
69)
70)
71)
The famous ?Fox Project? was guided by
[Question ID = 18821]
1. Marvin Harris. [Option ID = 45278]
2. Sol Tax [Option ID = 45275]
3. George P. Murdock [Option ID = 45276]
4. Margaret Mead [Option ID = 45277]
Correct Answer :-
Sol Tax [Option ID = 45275]
The first Director of Anthropological Survey of India was
[Question ID = 18851]
1. S.C. Roy [Option ID = 45396]
2. N. K. Bose. [Option ID = 45395]
3. B.S. Guha [Option ID = 45398]
4. G.S.Ghurye [Option ID = 45397]
Correct Answer :-
B.S. Guha [Option ID = 45398]
The Cross Cultural Cumulative Coding Center (The 5 C?s ) has been developed by
[Question ID = 18833]
1. Ruth Benedict [Option ID = 45323]
2. Franz Boas. [Option ID = 45324]
3. G. P. Murdock [Option ID = 45326]
4. Jack Goody [Option ID = 45325]
Correct Answer :-
G. P. Murdock [Option ID = 45326]
The advisory committee for the revision of the list of Scheduled Castes and Scheduled Tribal is also known as
[Question ID = 18861]
1. Ghurye committee [Option ID = 45438]
2. Shilu Ao committee [Option ID = 45437]
3. Elwin committee [Option ID = 45436]
4. Lokur committee [Option ID = 45435]
Correct Answer :-
Lokur committee [Option ID = 45435]
The period referring to the process of growth from birth up to 28 days of life is-
[Question ID = 18837]
1. Postnatal [Option ID = 45339]
2. Infancy [Option ID = 45340]
3. Neo-natal [Option ID = 45342]
4. Prenatal [Option ID = 45341]
Correct Answer :-
Neo-natal [Option ID = 45342]
The book ?Homo Hierarchicus? is authored by
[Question ID = 18791]
1. Louis Dumont [Option ID = 45155]
2. M.N.Srinivas [Option ID = 45156]
3. G.S. Ghurye [Option ID = 45158]
4. Gerald Berreman [Option ID = 45157]
72)
73)
74)
75)
76)
77)
Correct Answer :-
Louis Dumont [Option ID = 45155]
The study of methods under social control directed towards improving the hereditary constitution of future human generations is
[Question ID = 18847]
1. Ergonomics [Option ID = 45382]
2. Eugenics [Option ID = 45380]
3. Genetics [Option ID = 45379]
4. Physiology [Option ID = 45381]
Correct Answer :-
Eugenics [Option ID = 45380]
The energy giving foods which include starch of cereals and sugar from sugarcane and fruits are-
[Question ID = 18829]
1. Vitamins [Option ID = 45310]
2. Proteins [Option ID = 45308]
3. Carbohydrates [Option ID = 45307]
4. Minerals [Option ID = 45309]
Correct Answer :-
Carbohydrates [Option ID = 45307]
The mixture of homozygous and heterozygous genes of a population in a state of equilibrium is called:
[Question ID = 18878]
1. Multiple alleles [Option ID = 45505]
2. Transient polymorphism [Option ID = 45504]
3. Balanced polymorphism [Option ID = 45503]
4. Genetic equilibrium. [Option ID = 45506]
Correct Answer :-
Balanced polymorphism [Option ID = 45503]
The classical definition of culture by E. B. Tylor appeared in the year
[Question ID = 18830]
1. 1871 [Option ID = 45313]
2. 1879 [Option ID = 45311]
3. 1870 [Option ID = 45312]
4. 1872 [Option ID = 45314]
Correct Answer :-
1871 [Option ID = 45313]
The Fifth Schedule of the Constitution of India deals with
[Question ID = 18811]
1. Economic development and educational opportunities for Scheduled Tribes [Option ID = 45238]
2. Atrocities and crime against Scheduled Tribes [Option ID = 45237]
3. Administration of the Scheduled Areas [Option ID = 45236]
4. Reservation for the Scheduled Tribes [Option ID = 45235]
Correct Answer :-
Administration of the Scheduled Areas [Option ID = 45236]
The idea of ?original affluent society? for hunting and gathering subsistence was proposed by
[Question ID = 18840]
78)
79)
80)
81)
82)
1. Leslie A. White [Option ID = 45351]
2. Marshall D. Sahlins [Option ID = 45354]
3. Roy Rappaport [Option ID = 45353]
4. Julian Steward [Option ID = 45352]
Correct Answer :-
Marshall D. Sahlins [Option ID = 45354]
The concept ?Tribal-Rajput continuum? was given by
[Question ID = 18841]
1. S.S. Sarkar [Option ID = 45357]
2. L. P. Vidyarthi [Option ID = 45356]
3. B. S. Guha [Option ID = 45358]
4. Surjit Sinha [Option ID = 45355]
Correct Answer :-
Surjit Sinha [Option ID = 45355]
The concept ?modal personality? was popularized by
[Question ID = 18831]
1. Abram Kardiner [Option ID = 45316]
2. Ruth Benedict [Option ID = 45317]
3. Geoffrey Gorer [Option ID = 45318]
4. Cora Dubois [Option ID = 45315]
Correct Answer :-
Abram Kardiner [Option ID = 45316]
The concept of ?social network? was first used by
[Question ID = 18860]
1. A. L. Kroeber [Option ID = 45432]
2. G. P. Murdock [Option ID = 45434]
3. John Barnes [Option ID = 45431]
4. Kluckhohn [Option ID = 45433]
Correct Answer :-
John Barnes [Option ID = 45431]
The concept of ?complimentary filiations? was given by
[Question ID = 18881]
1. Raymond Firth [Option ID = 45518]
2. Evans-Pritchard [Option ID = 45516]
3. Radcliffe-Brown [Option ID = 45517]
4. Meyer Fortes [Option ID = 45515]
Correct Answer :-
Meyer Fortes [Option ID = 45515]
The concept of folk-urban continuum was proposed by
[Question ID = 18852]
1. Teodor Shanin [Option ID = 45400]
2. Eric R. Wolf [Option ID = 45399]
3. Milton Singer [Option ID = 45402]
4. Robert Redfield [Option ID = 45401]
Correct Answer :-
Robert Redfield [Option ID = 45401]
83)
84)
85)
86)
87)
88)
The distinction between general and specific evolution was given by
[Question ID = 18872]
1. Leslie White [Option ID = 45479]
2. Elman Service [Option ID = 45480]
3. Marshall Sahlins [Option ID = 45481]
4. Julian Steward [Option ID = 45482]
Correct Answer :-
Elman Service [Option ID = 45480]
The first Alpine glaciation is known as
[Question ID = 18874]
1. Gunz [Option ID = 45490]
2. Wurm [Option ID = 45489]
3. Riss [Option ID = 45488]
4. Mindel [Option ID = 45487]
Correct Answer :-
Gunz [Option ID = 45490]
The teaching of anthropology first began at
[Question ID = 18843]
1. University of Calcutta [Option ID = 45364]
2. University of Madras [Option ID = 45363]
3. University of Delhi [Option ID = 45365]
4. University of Lucknow [Option ID = 45366]
Correct Answer :-
University of Calcutta [Option ID = 45364]
The Blade tools were prepared by
[Question ID = 18855]
1. Punching technique [Option ID = 45413]
2. Stone hammer technique [Option ID = 45412]
3. Clactonian technique [Option ID = 45414]
4. Cylinder hammer technique [Option ID = 45411]
Correct Answer :-
Punching technique [Option ID = 45413]
The term Gemeinschaft refers to
[Question ID = 18793]
1. Association [Option ID = 45165]
2. Institution [Option ID = 45166]
3. Society [Option ID = 45164]
4. Community [Option ID = 45163]
Correct Answer :-
Community [Option ID = 45163]
The average cranial capacity of Homo erectus ranges from:
[Question ID = 18868]
1. 895-1040cc [Option ID = 45463]
2. 1200-1350cc. [Option ID = 45466]
3. 1120-1310cc [Option ID = 45465]
4. 1050-1200cc [Option ID = 45464]
89)
90)
91)
92)
93)
94)
Correct Answer :-
895-1040cc [Option ID = 45463]
The method in which measurements are repeatedly done on the same individual or group or individuals at definite age intervals is-
[Question ID = 18836]
1. Longitudinal study [Option ID = 62613]
2. Cross-sectional study [Option ID = 45335]
3. Semi longitudinal [Option ID = 45338]
4. Mixed longitudinal [Option ID = 45337]
Correct Answer :-
Semi longitudinal [Option ID = 45338]
The author of the book Encounter with Anthropology is:
[Question ID = 18803]
1. Claude L?vi-Strauss [Option ID = 45203]
2. Bronislaw Malinowski [Option ID = 45205]
3. A.R. Radcliffe-Brown [Option ID = 45206]
4. Robin Fox [Option ID = 45204]
Correct Answer :-
Robin Fox [Option ID = 45204]
The term ?habitus? was given by
[Question ID = 18880]
1. Edward Sapir [Option ID = 45513]
2. Margaret Mead [Option ID = 45512]
3. Pierre Bourdieu [Option ID = 45511]
4. Jack Goody [Option ID = 45514]
Correct Answer :-
Pierre Bourdieu [Option ID = 45511]
The term ?thick description? in anthropology is associated with
[Question ID = 18871]
1. Marvin Harris [Option ID = 45478]
2. Clifford Geertz [Option ID = 48535]
3. Levi-Strauss [Option ID = 45475]
4. Roland Barthes [Option ID = 45477]
Correct Answer :-
Clifford Geertz [Option ID = 48535]
The fossil remains of Ramapithecus in India was discovered from the:
[Question ID = 18858]
1. Devagiri Hills [Option ID = 45425]
2. Siwalik Hills [Option ID = 45424]
3. Bateshwar Hills [Option ID = 45426]
4. Nilgiri Hills [Option ID = 45423]
Correct Answer :-
Siwalik Hills [Option ID = 45424]
The communities having matrilineal descent is found in
[Question ID = 18850]
95)
96)
97)
98)
99)
100)
1. Tripura [Option ID = 45392]
2. Meghalaya [Option ID = 45393]
3. Nagaland [Option ID = 45394]
4. Andaman Islands. [Option ID = 45391]
Correct Answer :-
Meghalaya [Option ID = 45393]
The norm whereby a couple joins the residence of bride?s father?s sister is called
[Question ID = 18869]
1. Uxorilocal [Option ID = 45467]
2. Amitalocal [Option ID = 45468]
3. Virilocal [Option ID = 45469]
4. Ambilocal [Option ID = 45470]
Correct Answer :-
Amitalocal [Option ID = 45468]
Phenomenon of Canalization was described by
[Question ID = 18787]
1. J.M Tanner [Option ID = 45140]
2. Barry Bogin [Option ID = 45142]
3. C.H Waddington [Option ID = 45139]
4. Ernst Haeckel [Option ID = 45141]
Correct Answer :-
C.H Waddington [Option ID = 45139]
The first Prehistoric tool was found at [Question ID = 18796]
1. Pallavaram [Option ID = 45176]
2. Adamgarh [Option ID = 45177]
3. Hunsgi [Option ID = 45178]
4. Kondapeta [Option ID = 45175]
Correct Answer :-
Pallavaram [Option ID = 45176]
Angle of the Clactonian Flake is [Question ID = 18816]
1. More than 45 degree [Option ID = 45258]
2. More than 90 degree [Option ID = 45256]
3. Less than 45 degree [Option ID = 45257]
4. Less than 90 degree [Option ID = 45255]
Correct Answer :-
More than 90 degree [Option ID = 45256]
One of the distinct characteristics of Prosimians is:
[Question ID = 18875]
1. Absence of tail [Option ID = 45494]
2. High degree of colour vision [Option ID = 45493]
3. Highly evolved brain [Option ID = 45492]
4. Highly developed sense of smell [Option ID = 45491]
Correct Answer :-
Highly developed sense of smell [Option ID = 45491]
One of the distinct characteristic of Prosimians is:
[Question ID = 30658]
FirstRanker.com - FirstRanker's Choice
1)
2)
3)
4)
5)
DU MSc Anthropology
Topic:- DU_J18_MSC_ANTHRO
Increased labour investment in agriculture with no increase in per capita productivity is called as
[Question ID = 18814]
1. Agricultural recession [Option ID = 45250]
2. Agricultural stagnation [Option ID = 45247]
3. Agricultural relegation [Option ID = 45248]
4. Agricultural involution [Option ID = 45249]
Correct Answer :-
Agricultural involution [Option ID = 45249]
Which of the following is a hominid?
[Question ID = 18857]
1. Chimpanzee [Option ID = 45421]
2. Human [Option ID = 45422]
3. Macaque [Option ID = 45419]
4. Baboon [Option ID = 45420]
Correct Answer :-
Human [Option ID = 45422]
Which rule says that the best suited human body for a hot, tropical region would be one with long limbs and short trunk?
[Question ID = 18827]
1. Allen?s rule [Option ID = 45299]
2. Thompson?s rule [Option ID = 45302]
3. Gloger?s rule [Option ID = 45301]
4. Bergman?s rule [Option ID = 45300]
Correct Answer :-
Allen?s rule [Option ID = 45299]
Which Australopithecus fossil was nicknamed as Lucy?
[Question ID = 18867]
1. Australopithecus africanus [Option ID = 45459]
2. Australopithecus afarensis [Option ID = 45462]
3. Australopithecus anamensis [Option ID = 45460]
4. Australopithecus robustus [Option ID = 45461]
Correct Answer :-
Australopithecus afarensis [Option ID = 45462]
Which among the following does not constitute the elementary form of exchange?
[Question ID = 18820]
1. Exchange of goods [Option ID = 45274]
2. Exchange of women [Option ID = 45271]
3. Exchange of money [Option ID = 45272]
4. Exchange of messages [Option ID = 45273]
Correct Answer :-
Exchange of goods [Option ID = 45274]
6)
7)
8)
9)
10)
11)
12)
Which prehistoric site is known as Mode I Industry?
[Question ID = 18835]
1. Pirro Nord [Option ID = 45333]
2. Hunsgi [Option ID = 45331]
3. Tautavel [Option ID = 45332]
4. Somme valley [Option ID = 45334]
Correct Answer :-
Pirro Nord [Option ID = 45333]
Any quantitative increase in body size is known as [Question ID = 18788]
1. Development [Option ID = 45144]
2. Maturation [Option ID = 45146]
3. Differentiation [Option ID = 45145]
4. Growth [Option ID = 45143]
Correct Answer :-
Growth [Option ID = 45143]
The highest point on the head is known as [Question ID = 18807]
1. Opisthocranion [Option ID = 45222]
2. Bregma [Option ID = 45220]
3. Vertex [Option ID = 45221]
4. Lambda [Option ID = 45219]
Correct Answer :-
Vertex [Option ID = 45221]
Autosomes are
[Question ID = 18826]
1. Normal sex chromosomes [Option ID = 45296]
2. Abnormal sex chromosomes [Option ID = 45298]
3. Abnormal chromosomes [Option ID = 45297]
4. All chromosomes other than the sex chromosomes [Option ID = 45295]
Correct Answer :-
All chromosomes other than the sex chromosomes [Option ID = 45295]
?Puberty spurt? occurs in which one of the following stages of human growth [Question ID = 18798]
1. Infancy [Option ID = 45183]
2. Childhood [Option ID = 45185]
3. Adolescence [Option ID = 45186]
4. Adulthood [Option ID = 45184]
Correct Answer :-
Adolescence [Option ID = 45186]
Potlatch is associated with [Question ID = 18802]
1. Iroquois [Option ID = 45201]
2. Kwakiutl [Option ID = 45199]
3. Shoshonean [Option ID = 45202]
4. Arapesh [Option ID = 45200]
Correct Answer :-
Kwakiutl [Option ID = 45199]
Oldowan culture is dominated by
[Question ID = 18824]
1. Blade tools [Option ID = 45288]
2. Flake tools [Option ID = 45289]
13)
14)
15)
16)
17)
18)
3. Pebble tools [Option ID = 45287]
4. Microlithic tools [Option ID = 45290]
Correct Answer :-
Pebble tools [Option ID = 45287]
A higher fortified part of a city is called
[Question ID = 18823]
1. metropolis [Option ID = 45284]
2. megapolis [Option ID = 45285]
3. acropolis [Option ID = 45283]
4. necropolis [Option ID = 45286]
Correct Answer :-
acropolis [Option ID = 45283]
Yale Cambridge expedition to the Sohan Valley was led by
[Question ID = 18795]
1. R. B. Foote and V.D. Krishnaswamy [Option ID = 45171]
2. G. R. Sharma and V.D. Mishra [Option ID = 45174]
3. H. D. Sankalia and B. Subba Rao [Option ID = 45173]
4. H. De. Terra and T. T. Paterson [Option ID = 45172]
Correct Answer :-
H. De. Terra and T. T. Paterson [Option ID = 45172]
Two nuclear families in adjacent generations with one son/husband or daughter/wife who is a member of both the families is called a
[Question ID = 18870]
1. lineal family [Option ID = 45471]
2. supplemented nuclear family [Option ID = 45474]
3. stem family [Option ID = 45473]
4. compound family [Option ID = 45472]
Correct Answer :-
stem family [Option ID = 45473]
The lack, excess or imbalance of nutrients in the diet leading to ill health is known as [Question ID = 18790]
1. Malnutrition [Option ID = 45153]
2. Over nutrition [Option ID = 45151]
3. Under nutrition [Option ID = 45152]
4. Marasmus [Option ID = 45154]
Correct Answer :-
Malnutrition [Option ID = 45153]
Holocene starts around
[Question ID = 18834]
1. 20,000 B.P. [Option ID = 45329]
2. 12,000 B.P. [Option ID = 45328]
3. 14,000 B.P. [Option ID = 45330]
4. 10,000 B.P. [Option ID = 45327]
Correct Answer :-
10,000 B.P. [Option ID = 45327]
According to Levi-Strauss which of the following constitute one of the ?atoms of kinship? relationship?
[Question ID = 18879]
19)
20)
21)
22)
23)
24)
1. mother-son [Option ID = 45507]
2. mother-daughter [Option ID = 45508]
3. father-son [Option ID = 45509]
4. father-daughter [Option ID = 45510]
Correct Answer :-
mother-son [Option ID = 45507]
A karyotype is a:
[Question ID = 18819]
1. Picture of an individual's chromosomes arranged in a standardized way [Option ID = 45269]
2. Type of abnormal chromosome that is associated with Down's syndrome [Option ID = 45268]
3. General term for any type of chromosome [Option ID = 45267]
4. General term for sex chromosome [Option ID = 45270]
Correct Answer :-
Picture of an individual's chromosomes arranged in a standardized way [Option ID = 45269]
Theodicy is an
[Question ID = 18882]
1. attempt to explain theocracy [Option ID = 45521]
2. attempt to explain good [Option ID = 45519]
3. attempt to explain evil [Option ID = 45520]
4. attempt to explain lie and deceit [Option ID = 45522]
Correct Answer :-
attempt to explain evil [Option ID = 45520]
The word ?Somatotype? was coined by [Question ID = 18797]
1. Faulkner [Option ID = 45182]
2. Sheldon [Option ID = 45180]
3. Parnell [Option ID = 45181]
4. Heath and Carter [Option ID = 45179]
Correct Answer :-
Heath and Carter [Option ID = 45179]
Substantivism as an approach in economic anthropology was given by
[Question ID = 18853]
1. A. L. Kroeber [Option ID = 45406]
2. Karl Polanyi [Option ID = 45405]
3. Oscar Lewis [Option ID = 45403]
4. Robert Redfield [Option ID = 62620]
Correct Answer :-
Karl Polanyi [Option ID = 45405]
?Genetics and the Origin of Species? was written by :
[Question ID = 18859]
1. Thomas Malthus. [Option ID = 45430]
2. Theodosius Dobzhansky [Option ID = 45428]
3. Charles Darwin [Option ID = 45427]
4. T. H. Morgan [Option ID = 45429]
Correct Answer :-
Theodosius Dobzhansky [Option ID = 45428]
The ability of the living organisms to survive in a particular ecological set up is called [Question ID = 18809]
25)
26)
27)
28)
29)
30)
1. Acclimation [Option ID = 45228]
2. Acclimatisation [Option ID = 45229]
3. Adaptation [Option ID = 45227]
4. Selection [Option ID = 45230]
Correct Answer :-
Adaptation [Option ID = 45227]
___________is the physiological ability to reproduce an offspring.
[Question ID = 18876]
1. Fecundity [Option ID = 45496]
2. Fertility [Option ID = 45495]
3. Virility [Option ID = 45498]
4. Natality [Option ID = 45497]
Correct Answer :-
Fecundity [Option ID = 45496]
Charles Darwin's ideas concerning the causes of evolution were probably formulated in his mind
[Question ID = 18817]
1. During the late 1880's [Option ID = 45262]
2. During his voyage on H.M.S. Beagle, especially after he reached the Gal?pagos Islands [Option ID = 45261]
3. Before he began his voyage of exploration around the world on H.M.S. Beagle [Option ID = 45260]
4. While he was still a student at Cambridge University [Option ID = 45259]
Correct Answer :-
During his voyage on H.M.S. Beagle, especially after he reached the Gal?pagos Islands [Option ID = 45261]
First Himalayan Glaciation is known as
[Question ID = 18884]
1. Terrace I [Option ID = 45530]
2. Pinjaur [Option ID = 45527]
3. Boulder conglomerate [Option ID = 45529]
4. Tatrot [Option ID = 45528]
Correct Answer :-
Tatrot [Option ID = 45528]
The author of the book Hindus of the Himalayas is [Question ID = 18812]
1. S.C. Dube [Option ID = 45241]
2. D.N. Majumdar [Option ID = 45242]
3. B. K. Roy-Burman [Option ID = 45240]
4. G.D. Berreman [Option ID = 45239]
Correct Answer :-
G.D. Berreman [Option ID = 45239]
Who proposed the ?Three Age System? in pre-history?
[Question ID = 18883]
1. C. J. Thomsen [Option ID = 45525]
2. B. M. Fagan [Option ID = 45523]
3. H.D.Sankalia [Option ID = 45524]
4. F. Borde [Option ID = 45526]
Correct Answer :-
C. J. Thomsen [Option ID = 45525]
Who among the following worked on the complicity between colonial agencies and Anthropologists?
31)
32)
33)
34)
35)
36)
[Question ID = 18801]
1. Ibn Khaldun [Option ID = 45196]
2. Akbar Ahmed [Option ID = 45197]
3. Ralf Linto [Option ID = 45198]
4. Talal Asad [Option ID = 45195]
Correct Answer :-
Talal Asad [Option ID = 45195]
Who is the author of the book ?The Old Stone Age??
[Question ID = 18854]
1. M.C. Burkitt [Option ID = 45407]
2. J. N. Pal [Option ID = 45408]
3. P.C. Pant [Option ID = 45409]
4. H. De Lumley [Option ID = 45410]
Correct Answer :-
M.C. Burkitt [Option ID = 45407]
Who included the Proto-Australoid racial element in the classification of Indian races?
[Question ID = 18866]
1. S.S. Sarkar [Option ID = 45456]
2. H.H. Risley [Option ID = 45455]
3. B.S. Guha [Option ID = 45458]
4. A.C. Haddon [Option ID = 45457]
Correct Answer :-
B.S. Guha [Option ID = 45458]
In which State is the site Isampur located?
[Question ID = 18815]
1. Karnataka [Option ID = 45253]
2. Uttar Pradesh [Option ID = 45254]
3. Madhya Pradesh [Option ID = 45252]
4. Bihar [Option ID = 45251]
Correct Answer :-
Karnataka [Option ID = 45253]
The view of the skull from above is [Question ID = 18800]
1. Norma Basalis [Option ID = 45193]
2. Norma Occipitalis [Option ID = 45194]
3. Norma Frontalis [Option ID = 45192]
4. Norma Verticalis [Option ID = 45191]
Correct Answer :-
Norma Verticalis [Option ID = 45191]
?Childhood? is peculiar to [Question ID = 18789]
1. Nonhuman primates [Option ID = 45149]
2. Human and nonhuman primates [Option ID = 45147]
3. Humans [Option ID = 45148]
4. Mammals [Option ID = 45150]
Correct Answer :-
Humans [Option ID = 45148]
Children subjected to starvation for short periods may recover completely on getting adequate diet. This is referred to as
37)
38)
39)
40)
41)
[Question ID = 18838]
1. Growth spurt [Option ID = 45346]
2. Catch down growth [Option ID = 45345]
3. Catch-up growth [Option ID = 45343]
4. Canalization [Option ID = 45344]
Correct Answer :-
Catch-up growth [Option ID = 45343]
Attirampakkam is [Question ID = 18805]
1. Lower Paleolithic site [Option ID = 45211]
2. Mesolithic site [Option ID = 45214]
3. Indus Valley Civilization site [Option ID = 45213]
4. Upper Paleolithic site [Option ID = 45212]
Correct Answer :-
Lower Paleolithic site [Option ID = 45211]
Shivering
[Question ID = 18828]
1. Has no effect on body heat [Option ID = 45306]
2. Indirectly produces more body heat [Option ID = 45304]
3. Speeds up the loss of body heat [Option ID = 45303]
4. Delays up the loss of body heat [Option ID = 45305]
Correct Answer :-
Indirectly produces more body heat [Option ID = 45304]
A deficiency disease of the skeletal system caused by a lack of Vitamin D or Calcium or both, and often resulting in bone deformities is
[Question ID = 18849]
1. Arthritis [Option ID = 45390]
2. Scurvy [Option ID = 45387]
3. Rickets [Option ID = 45389]
4. Osteoporosis [Option ID = 45388]
Correct Answer :-
Rickets [Option ID = 45389]
A deficiency disease caused due to severe lack of protein in diet among children is
[Question ID = 18856]
1. Anaemia [Option ID = 45418]
2. Kwashiorkor [Option ID = 45417]
3. Scurvy [Option ID = 45416]
4. Rickets [Option ID = 45415]
Correct Answer :-
Kwashiorkor [Option ID = 45417]
Samples of Dendrochronology is taken from which of the following?
[Question ID = 18873]
1. Lake [Option ID = 45483]
2. Tree [Option ID = 45486]
3. River [Option ID = 45484]
4. sea [Option ID = 45485]
Correct Answer :-
Tree [Option ID = 45486]
42)
43)
44)
45)
46)
47)
48)
Swiddening is a form of
[Question ID = 18822]
1. terrace cultivation [Option ID = 45281]
2. intensive cultivation [Option ID = 45280]
3. commercial cultivation [Option ID = 45282]
4. slash and burn cultivation [Option ID = 45279]
Correct Answer :-
slash and burn cultivation [Option ID = 45279]
Osteology is the study of [Question ID = 18810]
1. The human skeleton [Option ID = 45234]
2. The human brain [Option ID = 45233]
3. Human survival strategies [Option ID = 45231]
4. Human languages [Option ID = 45232]
Correct Answer :-
The human skeleton [Option ID = 45234]
Extended Case Study method in anthropology is associated with the work of [Question ID = 18813]
1. Paul Radin [Option ID = 45246]
2. Julian Steward [Option ID = 45244]
3. Anthony Walker [Option ID = 45245]
4. Max Gluckman [Option ID = 45243]
Correct Answer :-
Max Gluckman [Option ID = 45243]
Celts are prepared by
[Question ID = 18863]
1. Levalloisian technique [Option ID = 45443]
2. Clactonian technique [Option ID = 45444]
3. Pressure technique [Option ID = 45446]
4. Grinding and polishing technique [Option ID = 45445]
Correct Answer :-
Grinding and polishing technique [Option ID = 45445]
Khallu is a form : [Question ID = 18792]
1. Fishing [Option ID = 45160]
2. Hunting [Option ID = 45159]
3. Cropping [Option ID = 45161]
4. Weaving [Option ID = 45162]
Correct Answer :-
Cropping [Option ID = 45161]
Lascaux is
[Question ID = 18844]
1. Cave art site [Option ID = 45367]
2. Mesolithic site [Option ID = 45368]
3. Chalcolithic site [Option ID = 45369]
4. Neolithic site [Option ID = 45370]
Correct Answer :-
Cave art site [Option ID = 45367]
A social fact is not characterized by
[Question ID = 18862]
49)
50)
51)
52)
53)
54)
1. Inheritance [Option ID = 45441]
2. Independent existence [Option ID = 45439]
3. Constraints [Option ID = 45442]
4. Exteriority [Option ID = 45440]
Correct Answer :-
Inheritance [Option ID = 45441]
Chopper and Chopping tool are
[Question ID = 18864]
1. blade tool [Option ID = 45447]
2. flake tool [Option ID = 45449]
3. core tool [Option ID = 45448]
4. Neolithic tools [Option ID = 45450]
Correct Answer :-
core tool [Option ID = 45448]
The theory of evolution by natural selection was independently developed by: [Question ID = 19246]
1. Darwin and Lamarck [Option ID = 46977]
2. Darwin and Wallace [Option ID = 46976]
3. Lyell and Darwin [Option ID = 46975]
4. Lyell and Hutton [Option ID = 46978]
Correct Answer :-
Darwin and Lamarck [Option ID = 46977]
? Linea aspera? responsible for the erect posture in man is present in
[Question ID = 18846]
1. Anterior side of tibia [Option ID = 45376]
2. Anterior side of femur [Option ID = 45378]
3. Posterior side of femur [Option ID = 45375]
4. Medial side of fibula [Option ID = 45377]
Correct Answer :-
Posterior side of femur [Option ID = 45375]
?Purity and Danger? is associated with the work of
[Question ID = 18842]
1. M.N. Srinivas [Option ID = 45362]
2. Mary Douglas [Option ID = 45360]
3. Victor Turner [Option ID = 45359]
4. Edmund R. Leach [Option ID = 45361]
Correct Answer :-
Mary Douglas [Option ID = 45360]
?Swaddling Hypothesis? was used by the
[Question ID = 18832]
1. Evolutionist [Option ID = 45320]
2. Conflict Theorist [Option ID = 45321]
3. Cultural Ecologist [Option ID = 45319]
4. Cultural Personality Thinker [Option ID = 45322]
Correct Answer :-
Cultural Personality Thinker [Option ID = 45322]
Trapeze is [Question ID = 18806]
55)
56)
57)
58)
59)
60)
1. Neolithic tool [Option ID = 45215]
2. Core tool [Option ID = 45216]
3. Flake tool [Option ID = 45217]
4. Microlithic tool [Option ID = 45218]
Correct Answer :-
Microlithic tool [Option ID = 45218]
?Melanin? pigment is produced in our [Question ID = 18799]
1. Blood [Option ID = 45188]
2. Pancreas [Option ID = 45190]
3. Liver [Option ID = 45189]
4. Skin [Option ID = 45187]
Correct Answer :-
Skin [Option ID = 45187]
What is the earliest date of Indian Lower Paleolithic culture?
[Question ID = 18825]
1. 2.4 mya [Option ID = 45294]
2. 1.8 mya [Option ID = 45292]
3. 1.51 mya [Option ID = 45293]
4. 1.71 mya [Option ID = 45291]
Correct Answer :-
1.51 mya [Option ID = 45293]
Mutation Theory was proposed by:
[Question ID = 18865]
1. Hugo de Vries [Option ID = 45451]
2. Gregor Mendel. [Option ID = 45454]
3. August Weismann [Option ID = 45453]
4. R. A. Fischer [Option ID = 45452]
Correct Answer :-
Hugo de Vries [Option ID = 45451]
Gonadotropins are responsible for the growth of the-
[Question ID = 18839]
1. Whole body [Option ID = 45350]
2. Brain [Option ID = 45347]
3. Ovaries and testis [Option ID = 45348]
4. Immune system [Option ID = 45349]
Correct Answer :-
Ovaries and testis [Option ID = 45348]
Attempts to account for facts by means of general hypothesis or research questions is known as: [Question ID = 18794]
1. Phenomenology [Option ID = 45169]
2. Reduction [Option ID = 45170]
3. Induction [Option ID = 45167]
4. Deduction [Option ID = 45168]
Correct Answer :-
Deduction [Option ID = 45168]
Someone experiencing physiological problems as a result of oxygen deprivation is
[Question ID = 18818]
61)
62)
63)
64)
65)
1. Polycythaemia [Option ID = 45263]
2. Hyperoxia [Option ID = 45266]
3. Barometric excess [Option ID = 45264]
4. Hypoxia [Option ID = 45265]
Correct Answer :-
Hypoxia [Option ID = 45265]
Who gave the concept of ?the survival of the fittest?? [Question ID = 18808]
1. Hugo De Vries [Option ID = 45225]
2. Charles Darwin [Option ID = 45223]
3. Carl Correns [Option ID = 45226]
4. Herbert Spencer [Option ID = 45224]
Correct Answer :-
Herbert Spencer [Option ID = 45224]
Prepared core technique is also known as
[Question ID = 18845]
1. Punching technique [Option ID = 45374]
2. Levalloisian technique [Option ID = 45373]
3. Clactonian technique [Option ID = 45371]
4. Cylinder Hammer technique [Option ID = 45372]
Correct Answer :-
Levalloisian technique [Option ID = 45373]
A syndrome characterised by physiological, behavioural and mental defects that are due to the presence of an extra copy of the
chromosome 21 is
[Question ID = 18848]
1. Klinefelter syndrome [Option ID = 45385]
2. Marfan syndrome [Option ID = 45386]
3. Turner syndrome [Option ID = 45383]
4. Down syndrome [Option ID = 45384]
Correct Answer :-
Down syndrome [Option ID = 45384]
The statement "Animals are good to think with" is from
[Question ID = 18804]
1. Sherry B. Ortner [Option ID = 45209]
2. Margaret Mead [Option ID = 45210]
3. Claude L?vi Strauss [Option ID = 45208]
4. Mellisa Liewyn-Davies [Option ID = 45207]
Correct Answer :-
Claude L?vi Strauss [Option ID = 45208]
The physiological process of being or becoming accustomed to a new physical environment is known as:
[Question ID = 18877]
1. Alteration [Option ID = 45502]
2. Adaptation [Option ID = 45501]
3. Acclimatization [Option ID = 45500]
4. Assimilation [Option ID = 45499]
Correct Answer :-
Acclimatization [Option ID = 45500]
66)
67)
68)
69)
70)
71)
The famous ?Fox Project? was guided by
[Question ID = 18821]
1. Marvin Harris. [Option ID = 45278]
2. Sol Tax [Option ID = 45275]
3. George P. Murdock [Option ID = 45276]
4. Margaret Mead [Option ID = 45277]
Correct Answer :-
Sol Tax [Option ID = 45275]
The first Director of Anthropological Survey of India was
[Question ID = 18851]
1. S.C. Roy [Option ID = 45396]
2. N. K. Bose. [Option ID = 45395]
3. B.S. Guha [Option ID = 45398]
4. G.S.Ghurye [Option ID = 45397]
Correct Answer :-
B.S. Guha [Option ID = 45398]
The Cross Cultural Cumulative Coding Center (The 5 C?s ) has been developed by
[Question ID = 18833]
1. Ruth Benedict [Option ID = 45323]
2. Franz Boas. [Option ID = 45324]
3. G. P. Murdock [Option ID = 45326]
4. Jack Goody [Option ID = 45325]
Correct Answer :-
G. P. Murdock [Option ID = 45326]
The advisory committee for the revision of the list of Scheduled Castes and Scheduled Tribal is also known as
[Question ID = 18861]
1. Ghurye committee [Option ID = 45438]
2. Shilu Ao committee [Option ID = 45437]
3. Elwin committee [Option ID = 45436]
4. Lokur committee [Option ID = 45435]
Correct Answer :-
Lokur committee [Option ID = 45435]
The period referring to the process of growth from birth up to 28 days of life is-
[Question ID = 18837]
1. Postnatal [Option ID = 45339]
2. Infancy [Option ID = 45340]
3. Neo-natal [Option ID = 45342]
4. Prenatal [Option ID = 45341]
Correct Answer :-
Neo-natal [Option ID = 45342]
The book ?Homo Hierarchicus? is authored by
[Question ID = 18791]
1. Louis Dumont [Option ID = 45155]
2. M.N.Srinivas [Option ID = 45156]
3. G.S. Ghurye [Option ID = 45158]
4. Gerald Berreman [Option ID = 45157]
72)
73)
74)
75)
76)
77)
Correct Answer :-
Louis Dumont [Option ID = 45155]
The study of methods under social control directed towards improving the hereditary constitution of future human generations is
[Question ID = 18847]
1. Ergonomics [Option ID = 45382]
2. Eugenics [Option ID = 45380]
3. Genetics [Option ID = 45379]
4. Physiology [Option ID = 45381]
Correct Answer :-
Eugenics [Option ID = 45380]
The energy giving foods which include starch of cereals and sugar from sugarcane and fruits are-
[Question ID = 18829]
1. Vitamins [Option ID = 45310]
2. Proteins [Option ID = 45308]
3. Carbohydrates [Option ID = 45307]
4. Minerals [Option ID = 45309]
Correct Answer :-
Carbohydrates [Option ID = 45307]
The mixture of homozygous and heterozygous genes of a population in a state of equilibrium is called:
[Question ID = 18878]
1. Multiple alleles [Option ID = 45505]
2. Transient polymorphism [Option ID = 45504]
3. Balanced polymorphism [Option ID = 45503]
4. Genetic equilibrium. [Option ID = 45506]
Correct Answer :-
Balanced polymorphism [Option ID = 45503]
The classical definition of culture by E. B. Tylor appeared in the year
[Question ID = 18830]
1. 1871 [Option ID = 45313]
2. 1879 [Option ID = 45311]
3. 1870 [Option ID = 45312]
4. 1872 [Option ID = 45314]
Correct Answer :-
1871 [Option ID = 45313]
The Fifth Schedule of the Constitution of India deals with
[Question ID = 18811]
1. Economic development and educational opportunities for Scheduled Tribes [Option ID = 45238]
2. Atrocities and crime against Scheduled Tribes [Option ID = 45237]
3. Administration of the Scheduled Areas [Option ID = 45236]
4. Reservation for the Scheduled Tribes [Option ID = 45235]
Correct Answer :-
Administration of the Scheduled Areas [Option ID = 45236]
The idea of ?original affluent society? for hunting and gathering subsistence was proposed by
[Question ID = 18840]
78)
79)
80)
81)
82)
1. Leslie A. White [Option ID = 45351]
2. Marshall D. Sahlins [Option ID = 45354]
3. Roy Rappaport [Option ID = 45353]
4. Julian Steward [Option ID = 45352]
Correct Answer :-
Marshall D. Sahlins [Option ID = 45354]
The concept ?Tribal-Rajput continuum? was given by
[Question ID = 18841]
1. S.S. Sarkar [Option ID = 45357]
2. L. P. Vidyarthi [Option ID = 45356]
3. B. S. Guha [Option ID = 45358]
4. Surjit Sinha [Option ID = 45355]
Correct Answer :-
Surjit Sinha [Option ID = 45355]
The concept ?modal personality? was popularized by
[Question ID = 18831]
1. Abram Kardiner [Option ID = 45316]
2. Ruth Benedict [Option ID = 45317]
3. Geoffrey Gorer [Option ID = 45318]
4. Cora Dubois [Option ID = 45315]
Correct Answer :-
Abram Kardiner [Option ID = 45316]
The concept of ?social network? was first used by
[Question ID = 18860]
1. A. L. Kroeber [Option ID = 45432]
2. G. P. Murdock [Option ID = 45434]
3. John Barnes [Option ID = 45431]
4. Kluckhohn [Option ID = 45433]
Correct Answer :-
John Barnes [Option ID = 45431]
The concept of ?complimentary filiations? was given by
[Question ID = 18881]
1. Raymond Firth [Option ID = 45518]
2. Evans-Pritchard [Option ID = 45516]
3. Radcliffe-Brown [Option ID = 45517]
4. Meyer Fortes [Option ID = 45515]
Correct Answer :-
Meyer Fortes [Option ID = 45515]
The concept of folk-urban continuum was proposed by
[Question ID = 18852]
1. Teodor Shanin [Option ID = 45400]
2. Eric R. Wolf [Option ID = 45399]
3. Milton Singer [Option ID = 45402]
4. Robert Redfield [Option ID = 45401]
Correct Answer :-
Robert Redfield [Option ID = 45401]
83)
84)
85)
86)
87)
88)
The distinction between general and specific evolution was given by
[Question ID = 18872]
1. Leslie White [Option ID = 45479]
2. Elman Service [Option ID = 45480]
3. Marshall Sahlins [Option ID = 45481]
4. Julian Steward [Option ID = 45482]
Correct Answer :-
Elman Service [Option ID = 45480]
The first Alpine glaciation is known as
[Question ID = 18874]
1. Gunz [Option ID = 45490]
2. Wurm [Option ID = 45489]
3. Riss [Option ID = 45488]
4. Mindel [Option ID = 45487]
Correct Answer :-
Gunz [Option ID = 45490]
The teaching of anthropology first began at
[Question ID = 18843]
1. University of Calcutta [Option ID = 45364]
2. University of Madras [Option ID = 45363]
3. University of Delhi [Option ID = 45365]
4. University of Lucknow [Option ID = 45366]
Correct Answer :-
University of Calcutta [Option ID = 45364]
The Blade tools were prepared by
[Question ID = 18855]
1. Punching technique [Option ID = 45413]
2. Stone hammer technique [Option ID = 45412]
3. Clactonian technique [Option ID = 45414]
4. Cylinder hammer technique [Option ID = 45411]
Correct Answer :-
Punching technique [Option ID = 45413]
The term Gemeinschaft refers to
[Question ID = 18793]
1. Association [Option ID = 45165]
2. Institution [Option ID = 45166]
3. Society [Option ID = 45164]
4. Community [Option ID = 45163]
Correct Answer :-
Community [Option ID = 45163]
The average cranial capacity of Homo erectus ranges from:
[Question ID = 18868]
1. 895-1040cc [Option ID = 45463]
2. 1200-1350cc. [Option ID = 45466]
3. 1120-1310cc [Option ID = 45465]
4. 1050-1200cc [Option ID = 45464]
89)
90)
91)
92)
93)
94)
Correct Answer :-
895-1040cc [Option ID = 45463]
The method in which measurements are repeatedly done on the same individual or group or individuals at definite age intervals is-
[Question ID = 18836]
1. Longitudinal study [Option ID = 62613]
2. Cross-sectional study [Option ID = 45335]
3. Semi longitudinal [Option ID = 45338]
4. Mixed longitudinal [Option ID = 45337]
Correct Answer :-
Semi longitudinal [Option ID = 45338]
The author of the book Encounter with Anthropology is:
[Question ID = 18803]
1. Claude L?vi-Strauss [Option ID = 45203]
2. Bronislaw Malinowski [Option ID = 45205]
3. A.R. Radcliffe-Brown [Option ID = 45206]
4. Robin Fox [Option ID = 45204]
Correct Answer :-
Robin Fox [Option ID = 45204]
The term ?habitus? was given by
[Question ID = 18880]
1. Edward Sapir [Option ID = 45513]
2. Margaret Mead [Option ID = 45512]
3. Pierre Bourdieu [Option ID = 45511]
4. Jack Goody [Option ID = 45514]
Correct Answer :-
Pierre Bourdieu [Option ID = 45511]
The term ?thick description? in anthropology is associated with
[Question ID = 18871]
1. Marvin Harris [Option ID = 45478]
2. Clifford Geertz [Option ID = 48535]
3. Levi-Strauss [Option ID = 45475]
4. Roland Barthes [Option ID = 45477]
Correct Answer :-
Clifford Geertz [Option ID = 48535]
The fossil remains of Ramapithecus in India was discovered from the:
[Question ID = 18858]
1. Devagiri Hills [Option ID = 45425]
2. Siwalik Hills [Option ID = 45424]
3. Bateshwar Hills [Option ID = 45426]
4. Nilgiri Hills [Option ID = 45423]
Correct Answer :-
Siwalik Hills [Option ID = 45424]
The communities having matrilineal descent is found in
[Question ID = 18850]
95)
96)
97)
98)
99)
100)
1. Tripura [Option ID = 45392]
2. Meghalaya [Option ID = 45393]
3. Nagaland [Option ID = 45394]
4. Andaman Islands. [Option ID = 45391]
Correct Answer :-
Meghalaya [Option ID = 45393]
The norm whereby a couple joins the residence of bride?s father?s sister is called
[Question ID = 18869]
1. Uxorilocal [Option ID = 45467]
2. Amitalocal [Option ID = 45468]
3. Virilocal [Option ID = 45469]
4. Ambilocal [Option ID = 45470]
Correct Answer :-
Amitalocal [Option ID = 45468]
Phenomenon of Canalization was described by
[Question ID = 18787]
1. J.M Tanner [Option ID = 45140]
2. Barry Bogin [Option ID = 45142]
3. C.H Waddington [Option ID = 45139]
4. Ernst Haeckel [Option ID = 45141]
Correct Answer :-
C.H Waddington [Option ID = 45139]
The first Prehistoric tool was found at [Question ID = 18796]
1. Pallavaram [Option ID = 45176]
2. Adamgarh [Option ID = 45177]
3. Hunsgi [Option ID = 45178]
4. Kondapeta [Option ID = 45175]
Correct Answer :-
Pallavaram [Option ID = 45176]
Angle of the Clactonian Flake is [Question ID = 18816]
1. More than 45 degree [Option ID = 45258]
2. More than 90 degree [Option ID = 45256]
3. Less than 45 degree [Option ID = 45257]
4. Less than 90 degree [Option ID = 45255]
Correct Answer :-
More than 90 degree [Option ID = 45256]
One of the distinct characteristics of Prosimians is:
[Question ID = 18875]
1. Absence of tail [Option ID = 45494]
2. High degree of colour vision [Option ID = 45493]
3. Highly evolved brain [Option ID = 45492]
4. Highly developed sense of smell [Option ID = 45491]
Correct Answer :-
Highly developed sense of smell [Option ID = 45491]
One of the distinct characteristic of Prosimians is:
[Question ID = 30658]
1. Hingly evolved brain [Option ID = 62625]
2. Hingly developed sense of smell [Option ID = 62624]
3. Absence of tail [Option ID = 62627]
4. Highly degree of colour vision [Option ID = 62626]
Correct Answer :-
Hingly developed sense of smell [Option ID = 62624]
FirstRanker.com - FirstRanker's Choice

This post was last modified on 29 January 2020